Download RITE Answers 2007

Document related concepts

Neurogenomics wikipedia , lookup

Psychiatry wikipedia , lookup

Neuropsychopharmacology wikipedia , lookup

Alzheimer's disease wikipedia , lookup

Rheobase wikipedia , lookup

Biochemistry of Alzheimer's disease wikipedia , lookup

National Institute of Neurological Disorders and Stroke wikipedia , lookup

Allochiria wikipedia , lookup

Clinical neurochemistry wikipedia , lookup

Transcript
2007 RITE Discussion & Reference Manual
Anatomy
Question 1: Anatomy - Peripheral/Autonomic Nervous Systems
Discussion:
Entrapment of the suprascapular nerve may cause shoulder pain, weakness of shoulder abduction, and sparing of
sensation about the shoulder.
References:
Devinsky O, Feldmann E. Examination of the cranial and peripheral nerves. New York: Churchill Livingston,
1998.
Question 2: Anatomy - Peripheral/Autonomic Nervous Systems
Discussion:
The tibialis posterior muscle is derived from the L5 and S1 myotomes, but it is supplied by the tibial nerve. An
abnormality in this muscle would rule out a peroneal mononeuropathy at the fibular head. The peroneus longus
and extensor hallicus longus could both be involved in an L5 radiculopathy or peroneal mononeuropathy and
would therefore not distinguish the two. The gastrocnemius is an S1 muscle supplied by the tibial nerve, and the
vastus lateralis is an L3 and L4 muscle supplied by the femoral nerve.
References:
Perotto A. Anatomical guide for the electromyographer. Springfield: Charles C. Thomas, 1994.
Question 9: Anatomy - Cortex and Connections
Discussion:
Asomatognosia is a category of neglect in which the patient denies ownership of a limb contralateral to a lesion
of the supramarginal gyrus of the parietal lobe (usually nondominant).
References:
Brazis PW, Masdeu JC, Biller J. Localization in clinical neurology. 4th ed. Philadelphia: Lippincott, Williams &
Wilkins, 2001.
Question 10: Anatomy - Neuromuscular Junction and Muscle
Discussion:
The radial nerve supplies the extensor pollicis longus and brevis, abductor pollicis longus, and extensor
digitorum longus. The abductor pollicis brevis is innervated by the median nerve.
References:
No author. Aids to the examination of the peripheral nervous system. 2nd ed. London: Bailliere Tindall, 1986.
Brazis PW, Masdeu JC, Biller J. Localization in clinical neurology. 4th ed. Philadelphia: Lippincott, Williams &
Wilkins, 2001.
Question 11: Anatomy - Spinal Cord
Discussion:
The patient has a dissociated sensory loss, with hypesthesias involving pain and temperature but sparing
proprioception and vibration. The findings are most consistent with syringomyelia. A syrinx can involve the
spinothalamic tracts bilaterally as they cross in the anterior white commissure that lies just anterior to the central
canal. Lesions of the dorsal nerve roots would affect all sensory modalities. The fasciculus cuneatus and gracilis
convey proprioceptive and vibration sensation, which are not involved in this patient. The lesion is at the level of
1
2007 RITE Discussion & Reference Manual
convey proprioceptive and vibration sensation, which are not involved in this patient. The lesion is at the level of
the cervical spinal cord so would not directly involve the thalamus.
References:
Campbell WW. DeJong’s the neurologic examination. Philadelphia: Lippincott, Williams & Wilkins, 2005.
Question 27: Anatomy - Brainstem/Cerebellum
Discussion:
The findings are most consistent with a left dorsolateral medullary syndrome (Wallenberg syndrome) due to
ischemia in the distribution of the posterior inferior cerebellar artery. Ipsilateral limb ataxia in this case is due to
involvement of the left inferior cerebellar peduncle.
References:
Carpenter MB. Core text of neuroanatomy. Baltimore: William & Wilkins, 1991
Question 38: Anatomy - Spinal Cord
Discussion:
Anesthesia over the perineal area ("saddle anesthesia") in conjuction with loss of sphincter tone and erectile
dysfunction can be caused by a lesion of the lower cauda equina or the conus medularis. The region of sensory
abnormality is in the S2 to S5 dermatomes. There is no weakness noted indicating that the lumbar and
lumbosacral plexi are likely spared. Since this happend 3 weeks ago and his sphincter function remains flacid,
this is most likely to be a peripheral (lower motor neuron) problem. These factors implicate the bilateral S2-S5
roots as the most likely culprit. An intramedullary lesion occuring 3 weeks ago would be expected to have some
component of spasticity at this point. A medial frontal lesion would explain the bladder incontinence but would
not explain the sensory findings and there is no lower extremity weakness. Onuf's nucleus is a sphincteromotor
nucleus located at S2 and S4 and would only explain part the patient's urinary symptoms. Given the sesory and
bowel and bladder symptoms, this process is involving the distal rather than the proximal cauda equina. Since
the more distal roots of the cauda equina are located more medially, a central disc herniation would be entirely
consistent with this clinical vignette.
References:
Blumenfeld H. Neuroanatomy through clinical cases. 1st ed. Sunderland: Sinauer Associates, Inc., 2002.
Question 39: Anatomy - Basal Ganglia and Thalamus
Discussion:
The anterior nucleus of the thalamus receives information from the mammilothalamic tract and hippocampus
and sends its outputs to the cingulate gyrus. The dorsomedial (or mediodorsal) nucleus connects prefrontal,
olfactory, and limbic cortex with prefrontal cortical regions. These last two nuclei are the only two of those
listed that would be reasonably expected to affect multimodal attention and motivational drive. The ventral
anterior and ventral lateral nuclei receive input from the basal ganglia and cerebellum, respectively, and send
their outputs to the motor cortex. The ventral posterior medial and lateral nuclei receive primary somatosensory
information from the face and body on its way to primary sensory cortex. The pulvinar and lateral posterior
nuclei are both involved in visual processing and subcortical modulation of visual attention. The medial and
lateral geniculate receive primary auditory and visual information, respectively.
References:
Murray S, Guillery R. Exploring the thalamus. San Diego: Academic Press, 2001.
Question 41: Anatomy - Embryology
Discussion:
The preganglionic sympathetic neurons are located in the intermediolateral cell column in all thoracic and the
2
2007 RITE Discussion & Reference Manual
upper two lumbar spinal segments. Thus, they are formed from the neural tube.
References:
Haines DE. Fundamental neuroscience. 2nd ed. New York: WB Saunders, 2002.
Question 45: Anatomy - Brainstem/Cerebellum
Discussion:
The patient with multiple sclerosis and "rhythmic clicking" in her ears has palatal myoclonus. A lesion of the
pathway connecting the red nucleus, central tegmental tract, inferior olivary nucleus, and dentate nucleus
(Guillain-Mollaret triangle) results in palatal myoclonus. Of the choices given in the question the only correct
answer is dentate nucleus.
References:
Brazis PW, Masdeu JC, Biller J. Localization in clinical neurology. 4th ed. Philadelphia: Lippincott, Williams &
Wilkins, 2001.
Question 47: Anatomy - Blood Supply of Brain/Spinal Cord
Discussion:
This patient presents with a classic subcortical infarction located in or around the internal capsule and involving
fibers in the posterior limb and the genu. The anterior choroidal artery, a branch of the internal carotid artery,
supplies the posterior limb of the internal capsule and at least a portion of the genu. Hemorrhage from the
internal carotid, anterior cerebral, or middle cerebral arteries would be expected to produce cortical findings,
which are not seen in this patient. The posterior communicating artery connects the posterior cerebral artery to
the internal carotid artery.
References:
Blumenfeld H. Neuroanatomy through clinical cases. 1st ed. Sunderland: Sinauer Associates, Inc., 2002.
Question 56: Anatomy - Brainstem/Cerebellum
Discussion:
The patient has components of Parinaud's syndrome (conjugate upgaze paresis, nystagmus retractorious, and
unreactive pupils). A lesion producing these findings occurs in the midbrain tectal region. It often occurs by
extraaxial compression on the quadrigeminal plate (particularly the superior colliculi). Pineal region masses as
well as obstructive hydrocephalus may also cause the syndrome.
References:
Gilman S, Newman S. Manter & Gatz's essentials of clinical neuroanatomy and neurophysiology. 7th ed.
Philadelphia: FA Davis, 1987.
Question 64: Anatomy - Cranial Nerves, Roots, and Plexus
Discussion:
A lesion distal to the geniculate ganglion and proximal to the stapedius muscle would cause weakness in the
muscles of facial expression, alter sublingual and submandibular gland function, cause loss of taste to the
anterior two thirds of the tongue, and cause hyperacusis but would spare lacrimation.
References:
Brazis PW, Masdeu JC, Biller J. Localization in clinical neurology. 3rd ed. Boston: Little, Brown and Co., 1996.
Question 69: Anatomy - Brainstem/Cerebellum
Discussion:
3
2007 RITE Discussion & Reference Manual
A unilateral lesion of the ventrocaudal pons results in ipsilateral lateral rectus and facial paresis and a
contralateral facial sparing hemiparesis. This is known as the Millard-Gubler syndrome.
References:
Haines DE. Fundamental Neuroscience. 2nd ed. New York: WB Saunders, 2002.
Question 77: Anatomy - Cortex and Connections
Discussion:
The suprachiasmatic nucleus of the hypothalamus is the dominant circadian pacemaker of the mammalian brain.
The intergeniculate leaflet and raphe nuclei mediate photic entrainment of the suprachiasmatic nucleus--light is
the major entraining stimulus of the circadian system. The pineal gland is more important in seasonal rhythm
control. The neurohypophysis does not have a major role in circadian pacemaking.
References:
Kandel ER, Schwartz JH, Jessel TM. Principles of neural science. 4th ed. New York: McGraw-Hill, 2000.
Question 89: Anatomy - Cortex and Connections
Discussion:
The nucleus accumbens, a component of the basal ganglia, recieves extensive input from the limbic system and
the orbitofrontal cortex. It is involved in anticipating rewards and is therefore implicated in substance abuse and
addiction. Alzheimer's disease is a cortical dementia that does not primarily affect the basal gangila.
Creutzfeldt-Jakob disease, a spongiform encephalopathy caused by prions, does not have a predilection for deep
gray nuclei such as the nuclues accumbens. Epilepsy is a also a cortical process without basal ganglia
involvement. Although Huntington's disease involves the basal ganglia, it is primarily striatal neurons of the
caudate and putamen that degenerate.
References:
Blumenfeld H. Neuroanatomy through clinical cases. 1st ed. Sunderland: Sinauer Associates, Inc., 2002.
Schoenbaum G, Roesch M, Stalnaker T. Orbitofrontal cortex, decision-making and drug addiction. Trends
Neurosci 2006 Feb;29(2):116-124.
Question 99: Anatomy - Cortex and Connections
Discussion:
The symptoms experienced by the patient in question would localize to the frontal lobe (behavioral and
personality changes) as well as the temporal lobe (semantic memory). This would be a common presentation of
frontotemporal dementia. The posterior aspect of the superior temporal gyrus receives information from the
primary auditory cortices and is integral to the network of areas which decode the meaning of words. This area,
in the left lateral temporal cortex, is most likely responsible for the patient's semantic problems. The
hippocampus is involved in memory and is part of the mesial temporal lobe. The orbitofrontal cortex is more
likely involved in the patient's impulsive behavior and personality changes. The supplementary motor area is
involved in complex motor programming.
References:
Kirshner H. Neurology in clinical practice, volume 1. In: Bradley W, Daroff R, Fenichel G, Marsden C, editors.
Language disorders: aphasia. 3rd ed. Boston: Butterworth Heinemann, 2000; 141-159.
Question 125: Anatomy - Cortex and Connections
Discussion:
Prosody, the ability to produce and understand the emotional quality of speech, is processsed by the
non-dominant hemisphere. Production of prosody depends upon nondominant dorsolateral frontal lobe while
4
2007 RITE Discussion & Reference Manual
comprehension of prosody is a function of the nondominant temporal lobe.
References:
Mitchell R, Elliott R, Barry M, Cruttenden A, Woodruff P. The neural response to emotional prosody, as
revealed by functional magnetic resonance imaging. Neuropsychologia 2003;41(10):1410-1421.
Question 130: Anatomy - Cortex and Connections
Discussion:
The amygdala lies in the anterior pole of the temporal lobe just deep to the uncus and is an important structure in
processing the emotional significance of stimuli, including pain. The caudate is the principal basal ganglia
nucleus involved in processing oculomotor and prefrontal information. The nucleus acumbens is involved in
anticipating reward and habit formation rather than the emotional component of pain. The hippocampus is part
of the limbic system and is an essential component of memory formation. The pulvinar is an association nucleus
in the thalamus involved in visual processing.
References:
Blumenfeld H. Neuroanatomy through clinical cases. 1st ed. Sunderland: Sinauer Associates, Inc., 2002.
Nolte J. The Human Brain. 4th ed. St Louis: Mosby, 1999.
Question 134: Anatomy - Cranial Nerves, Roots, and Plexus
Discussion:
The patient has neurological evidence to support a left abducens palsy and a left Horner’s syndrome. The sixth
nerve and oculosympathetic fibers are close together in the cavernous sinus, near the internal carotid artery.
Therefore, among the choices listed, the diagnosis that best explains his findings is a cavernous sinus aneurysm.
A pontine glioma could produce similar symptoms, but one would also expect long tract signs. Vestibular
shwannomas are usually in the cerebellopontine angle and can injure the seventh and eighth cranial nerves. An
ecstatic basilar artery may compress the brainstem and injury multiple cranial nerves, but a Horner’s syndrome
would be atypical. Similarly, tuberculous meningitis can cause a basilar meningitis and injure multiple cranial
nerves, but the intraaxial location of the oculosympathetic fibers at that level would tend to be spared.
References:
Campbell WW. DeJong’s the neurologic examination. Philadelphia: Lippincott, Williams & Wilkins, 2005.
Question 137: Anatomy - Cranial Nerves, Roots, and Plexus
Discussion:
Both the superior orbital fissure and the cavernous sinus contain cranial nerves III, IV, VI, V1, and sympathetic
nerve fibers. V2 does not travel through the superior orbital fissure. A lesion in neither location would produce
facial anhidrosis.
References:
Brazis PW, Masdeu JC, Biller J. Localization in clinical neurology. 4th ed. Philadelphia: Lippincott, Williams &
Wilkins, 2001.
Question 140: Anatomy - Peripheral/Autonomic Nervous Systems
Discussion:
The anterior interosseous nerve is a pure motor branch of the median nerve after it passes between the two
heads of pronator teres. The anterior interosseous nerve innervates flexor pollicis longus, flexor digitorum
profundis to the index and middle finger, and pronator quadratus. A lesion of this nerve impairs the ability of the
patient to make an OK sign with the thumb and index finger producing instead a pinch attitude. There is mild
weakness of forearm pronation and pain located in the proximal forearm.
5
2007 RITE Discussion & Reference Manual
References:
Brazis PW, Masdeu JC, Biller J. Localization in Clinical Neurology. Boston: Little, Brown, 1996; 12-14.
Question 154: Anatomy - Cranial Nerves, Roots, and Plexus
Discussion:
The patient is presenting with a Bell’s palsy resulting in weakness of muscles supplied by the seventh cranial
nerve. The platysmus is one of these muscles. The masseter is supplied by the trigeminal nerve, the levator
palpebrae by the oculomotor nerve, the genioglossus by the hypoglossal nerve, and the stylopharyngeus by the
glossopharyngeal nerve.
References:
Campbell WW. DeJong’s the neurologic examination. Philadelphia: Lippincott, Williams & Wilkins, 2005.
Question 155: Anatomy - Neuromuscular Junction and Muscle
Discussion:
The tibialis anterior, which dorsiflexes and inverts the foot, is innervated by the deep peroneal nerve. The
gastrocnemius plantar flexes the foot and is innervated by the tibial nerve (a branch of the sciatic nerve). The
semitendinosus, one of the hamstring muscles, is innervated by the sciatic nerve. The tensor fasciae latae,
innervated by the superior gluteal nerve, abducts and medially rotates the thigh. The sartorius muscle inwardly
rotates the hip, and flexes the hip and knee and is innervated by the femoral nerve.
References:
No author. Aids to the examination of the peripheral nervous system. 2nd ed. London: Bailliere Tindall, 1986.
Blumenfeld H. Neuroanatomy through clinical cases. 1st ed. Sunderland: Sinauer Associates, Inc., 2002.
Question 157: Anatomy - Spinal Cord
Discussion:
A lesion of the accessory cuneate nucleus would spare pain and temperature. Cells of origin of the lateral
spinothalamic tract are present in laminae I, IV, and V of the dorsal horn. They project to ventral posterolateral
and intralaminar and posterior nuclei of the thalamus. Further projection to the cortex is to areas three, one, and
two and to the secondary somatic sensory area.
References:
Parent A. Carpenter's human neuroanatomy. 9th ed. Baltimore: Williams & Wilkins, 1996.
Question 158: Anatomy - Spinal Cord
Discussion:
On the side of a spinal cord hemisection there is an upper motor neuron syndrome, greatly impaired
discriminatory tactile sense, loss of kinesthetic sense, and reduced muscle tone. Contralateral to the lesion there
is loss of pain and temperature due to interruption of the ascending spinothalamic tracts.
References:
Brazis PW, Masdeu JC, Biller J. Localization in clinical neurology. 4th ed. Philadelphia: Lippincott, Williams &
Wilkins, 2001.
Question 166: Anatomy - Neuromuscular Junction and Muscle
Discussion:
The masseter, temporalis, medial and lateral pterygoids, tensor veli palati, tensor tympani, anterior belly of the
6
2007 RITE Discussion & Reference Manual
digastric, and mylohyoid are innervated by the trigeminal nerve. The stapedius, buccinator, posterior belly of the
digastric, frontalis, as well as other muscles of facial expression are all innervated by the facial nerve. The
stylopharyngeus is innervated by the glossopharyngeal nerve.
References:
Brazis PW, Masdeu JC, Biller J. Localization in clinical neurology. 4th ed. Philadelphia: Lippincott, Williams &
Wilkins, 2001.
Question 195: Anatomy - Basal Ganglia and Thalamus
Discussion:
The globus pallidus externa (GPe) is a part of the indirect pathway through the basal ganglia that projects
inhibitory fibers to the subthalamic nucleus. The GPe is not part of the direct pathway where fibers project
directly from the striatum to the globus pallidus interna (GPi).
References:
Blumenfeld H. Neuroanatomy through clinical cases. 1st ed. Sunderland: Sinauer Associates, Inc., 2002.
Question 331: Anatomy - Blood Supply of Brain/Spinal Cord
Discussion:
The lateral inferior or caudal pontine syndrome due to occlusion of the anterior inferior cerebellar artery (AICA
syndrome) involves lesions in the fascicles of cranial nerve VII, the spinal tract, and nucleus of cranial nerve V,
the lateral spinal thalamic tract, descending sympathetic fibers (lateral reticular nucleus), the middle cerebellar
peduncle, the inferior surface of the cerebellum, and, in addition, the inner ear and cochlear nerve due to
occlusion of the labyrinthine artery, a common branch of the AICA. Clinical findings include ipsilateral ataxia,
loss of pain and temperature sensation of the face, Horner’s syndrome, deafness, and contralateral pain and
temperature loss of the limbs.
References:
Campbell WW. DeJong’s the neurologic examination. Philadelphia: Lippincott, Williams & Wilkins, 2005.
Question 341: Anatomy - Blood Supply of Brain/Spinal Cord
Discussion:
The recurrent artery of Heubner, a branch of the anterior cerebral artery, supplies the anteromedial part of the
head of the caudate nucleus, adjacent parts of the internal capsule and putamen, and parts of the septal nuclei.
References:
Haines DE. Fundamental Neuroscience. 2nd ed. New York: WB Saunders, 2002.
Question 349: Anatomy - Basal Ganglia and Thalamus
Discussion:
The caudate and the putamen serve as the primary input nuclei for the basal ganglia while the globus palidus,
which projects to the ventral anterior nucleus of the thalamus, is the primary output nucleus. The substantia
nigra pars compacta, located in the midbrain, sends dopamanergic fibers to the putamen. The subthalamic
nucleus receives inhibitory input from the external part of the globus palidus and sends excitatory input to the
globus palidus pars interna.
References:
Carpenter M, Sutin J. Human neuroanatomy. 8th ed. Baltimore: Williams and Wilkins, 1983.
Blumenfeld H. Neuroanatomy through clinical cases. 1st ed. Sunderland: Sinauer Associates, Inc., 2002.
7
2007 RITE Discussion & Reference Manual
Question 369: Anatomy - Peripheral/Autonomic Nervous Systems
Discussion:
The medial antebrachial cutaneous nerve is a branch of the medial cord of the brachial plexus and would be
expected to be injured in the neurogenic thoracic outlet syndrome (TOS) and would be spared in an ulnar nerve
mononeuropathy at the elbow (UNE). The dorsal ulnar cutaneous nerve may be abnormal in both neurogenic
TOS or UNE. The sensory portions of the median nerve, the superficial radial nerve, and lateral antebrachial
cutaneous nerve would be spared in UNE and neurogenic TOS.
References:
Campbell WW. DeJong’s the neurologic examination. Philadelphia: Lippincott, Williams & Wilkins, 2005.
Question 373: Anatomy - Blood Supply of Brain/Spinal Cord
Discussion:
Contralateral hemianesthesia and hemiparesis followed by spontaneous pain in the affected limbs is due to
involvement of the thalamoperforate branches of the posterior cerebral artery. Some of these branches supply
portions of the posterior limb of the internal capsule and may produce contralateral hemiparesis in addition to
the sensory changes and a central (thalamic) pain syndrome.
References:
Gilman S, Newman S. Manter & Gatz's essentials of clinical neuroanatomy and neurophysiology. 7th ed.
Philadelphia: FA Davis, 1987.
Question 378: Anatomy - Peripheral/Autonomic Nervous Systems
Discussion:
The patient has a light-near dissociation, which is not seen in disorders of the oculomotor or optic nerve. The
absence of other neurological findings in this case would make a midbrain tectal or superior colliculus lesion
unlikely. The slow, nonuniform constriction of the pupil is consistent with an Adie’s pupil that is due to an
abnormality of the ciliary ganglion or short ciliary nerves.
References:
Campbell WW. DeJong’s the neurologic examination. Philadelphia: Lippincott, Williams & Wilkins, 2005.
Question 390: Anatomy - Basal Ganglia and Thalamus
Discussion:
The paraventricular nucleus of the hypothalamus provides the bulk of the direct innervation of the preganglionic
sympathetic neurons.
References:
Haines DE. Fundamental neuroscience. 2nd ed. New York: WB Saunders, 2002.
Parent A. Carpenter's human neuroanatomy. 9th ed. Baltimore: Williams & Wilkins, 1996.
Question 392: Anatomy - Basal Ganglia and Thalamus
Discussion:
The dorsomedial nucleus of the thalamus receives input from limbic structures and projects diffusely to the
frontal cortex. As the main relay for information passing to the frontal association areas, it is felt to be the cause
of the amnestic confabulation in Korsakoff's syndrome. The anterior nucleus, although it receives input from
limbic structures, projects to the cingulate gyrus. The lateral posterior nucleus is involved in visual processing
similar to the pulvinar. The ventral posteriolateral nucleus receives primary somatosensory information from the
body. The reticular nucleus does not project to the cortex but instead regulates the activity of other thalamic
8
2007 RITE Discussion & Reference Manual
nuclei.
References:
Carpenter M, Sutin J. Human neuroanatomy. 8th ed. Baltimore: Williams and Wilkins, 1983.
Blumenfeld H. Neuroanatomy through clinical cases. 1st ed. Sunderland: Sinauer Associates, Inc., 2002.
Question 397: Anatomy - Basal Ganglia and Thalamus
Discussion:
Fibers from the dentatorubrothalamic tract primarily synapse on the ventral lateral nucleus of the thalamus, but
also on the ventral posterolateral. Fibers from these thalamic nuclei then project to the primary motor cortex
(Brodmann area 4).
References:
Campbell WW. DeJong’s the neurologic examination. Philadelphia: Lippincott, Williams & Wilkins, 2005.
Question 401: Anatomy - Cortex and Connections
Discussion:
The anterior commissure interconnects olfactory areas as well as homologous regions of the temporal lobe. The
ansa lenticularis is an ipsilateral efferent pathway from the globus pallidus. The lateral olfactory stria arises in
the olfactory bulb and projects to the ipsilateral prepyriform cortex and the amygdala. The median forebrain
bundle projects to the hypothalamus and contains fibers from basal olfactory and periamygdaloid regions and
the septal nuclei. The crus cerebri, on the ventral surface of the midbrain, contains the corticospinal and
corticobulbar tracts. It does not connecct cortical regions.
References:
Carpenter M, Sutin J. Human neuroanatomy. 8th ed. Baltimore: Williams and Wilkins, 1983.
Question 412: Anatomy - Cortex and Connections
Discussion:
Fifty patients with elevations of serum cardiac troponin levels had strokes involving the right posterior, superior
medial insula, and the right inferior parietal lobule. Among patients with right middle cerebral artery strokes, the
insular cortex was involved in 88% of patients with elevated serum cardiac troponin but in only 33% of patients
without the elevation.
References:
Ay H, Koroshetz WJ, Benner T, et al. Neuroanatomic correlates of stroke-related myocardial injury. Neurology
2006; 66: 13256.
Question 419: Anatomy - Brainstem/Cerebellum
Discussion:
The patient has a crossed paresis with right arm and leg weakness as well as left facial paresis. This localizes to
the left pons. The sensory deficits of fine touch on the right arm, trunk, and leg are due to involvement of the
left medial lemniscus. The patient's diplopia with left lateral gaze is due to involvement of the left abducens
nucleus.
References:
Brazis PW, Masdeu JC, Biller J. Localization in clinical neurology. 4th ed. Philadelphia: Lippincott, Williams &
Wilkins, 2001.
Question 421: Anatomy - Cortex and Connections
9
2007 RITE Discussion & Reference Manual
Discussion:
Damage to the fornix can occur with transcallosal surgery to remove a colloid cyst of the third ventricle, which
interrupts Papez's circuit and results in loss of the ability to form new memories.
References:
Haines DE. Fundamental neuroscience. 2nd ed. New York: WB Saunders, 2002.
Question 423: Anatomy - Spinal Cord
Discussion:
The dorsal and ventral spinocerebellar tracts are the most lateral tracts in the spinal cord and therefore would be
expected to be affected first by an extrinsic lateral process. The lateral corticospinal tract lies just medial to the
dorsal spinocerebellar tract while the anterior corticospinal tract is in the anterior midline. The fasciculus
gracilis is the medial aspect of the dorsal columns. The tectospinal pathway lies just anterior to the anterior
commisure and the reticulospinal tract lies just anterior to the lateral corticospinal tract. The spinothalamic tracts
run just interal to the ventral spinocerebellar tract.
References:
Blumenfeld H. Neuroanatomy through clinical cases. 1st ed. Sunderland: Sinauer Associates, Inc., 2002.
Nolte J. The Human Brain. 4th ed. St Louis: Mosby, 1999.
Question 430: Anatomy - Cranial Nerves, Roots, and Plexus
Discussion:
The latissimus dorsi muscle is intervated by the thoracodorsal nerve, which is a branch of the posterior cord of
the brachial plexus.
References:
Aids to the Examination of the Peripheral Nervous System. 4th ed. Edinburgh: WB Saunders; 2000.
Question 433: Anatomy - Blood Supply of Brain/Spinal Cord
Discussion:
The thalamoperforating branches of the posterior cerebral arteries perfuse the medial and anterior regions of the
thalamus. The thalamogeniculate branches of the posterior cerebral arteries perfuse the pulvinar and lateral
nuclei. The inferior thalamic arteries arise from the posterior communicating arteries and perfuse the inferior
portions of the thalamus. The medial posterior choroidal artery supplies the superior and medial portions of the
thalamus.
References:
Haines DE. Fundamental Neuroscience. 2nd ed. New York: WB Saunders, 2002.
Question 440: Anatomy - Basal Ganglia and Thalamus
Discussion:
The dorsomedial nucleus connects prefrontal, limbic, and olfactory structures with prefrontal cortex. The
intralaminar nuclei project to the cerebral cortex and the basal ganglia. The lateral dorsal nucleus receives input
from the hippocampus and projects to the cingulate gyrus. The pulvinar is an associaton nucleus that receives
inputs from parietal, temporal, and occipital cortex and then projects to these same areas. The reticular nucleus
projects to other thalamic nuclei but not to the cortex.
References:
Nolte J. The Human Brain. 4th ed. St Louis: Mosby, 1999.
10
2007 RITE Discussion & Reference Manual
Behavioral/Psychiatry
Question 7: Behavioral/Psychiatry - Developmental Disorders
Discussion:
The most common cause of inherited mental retardation is fragile X syndrome. Nearly all affected boys manifest
attention deficit disorder and have learning disabilities. The most frequent neurocognitive symptoms are abstract
reasoning, complex problem solving, and expressive language. Many will also show manifestations of autism,
with 33% meeting criteria for autism. Female carriers can have a milder form of the disease with learning
disabilities, and about 50% will manifest attention deficit disorder. Characteristic physical features include a
long thin face, prominent forehead and jaw, protuberant ears, hip dislocation, and club feet.
References:
Rittey CD. Learning difficulties: what the neurologist needs to know. J Neurol Neurosurg Psychiatr
2003;74:30-36.
Question 12: Behavioral/Psychiatry - General Psychiatry
Discussion:
This woman has obsessive-compulsive disorder, and flurodeoxyglucose PET consistently shows hypermetabolic
activity in the caudate, anterior cingulate, and orbitofrontal cortex.
References:
Baxter LR, Phelps ME, Mazziotta JC, et al. Local cerebral glucose metabolic rates in obsessive-compulsive
disorder: a comparison with rates in unipolar depression and normal controls. Arch Gen Psychiatry
1987;44:211-218.
Question 14: Behavioral/Psychiatry - Behavioral Complications of Systemic Disease
Discussion:
Forced normalization refers to a psychosis occurring after achievement of good clinical seizure control or
resolution of interictal epileptiform discharges.
References:
Paraiso J, Devinsky D. Neurobehavioral Aspects of Epilepsy. In: Feinberg TE, Farah MJ, editors. Behavioral
neurology and neuropsychology. 2nd ed. New York: McGraw-Hill, 2003.
Question 15: Behavioral/Psychiatry - Psychopharmacology
Discussion:
Acetylcholine, vital to the formation and encoding of new memories is one of many neurotransmitters deficient
in Alzheimer's disease. Medications including tricyclic antidepressants, antihistamines, and antiemetics with
strong anticholinergic properties can worsen memory loss as well as cause confusion.
References:
Cummings JL, Mega MS. Neuropsychiatry and behavioral neuroscience. New York: Oxford University Press,
2003.
Question 16: Behavioral/Psychiatry - Occipital Syndromes
Discussion:
There are numerous types of reading disorders seen after focal lesions and in neurodegenerative disorders.
Surface dyslexia is characterized by impairment linking the visual word form system with the phonological
11
2007 RITE Discussion & Reference Manual
Surface dyslexia is characterized by impairment linking the visual word form system with the phonological
output lexicon. Therefore, patients are unable to access the visual word image to link to proper pronounciation.
Patients have to rely on "print to sound conversion" and cannot read words that do not sound the way they are
spelled.
References:
Cummings JL, Mega MS. Neuropsychiatry and behavioral neuroscience. New York: Oxford University Press,
2003.
Question 19: Behavioral/Psychiatry - Dementia
Discussion:
Patients with frontotemporal dementia have been shown to manifest a variety of behavioral changes, including
hoarding of items and nascent musical and/or artistic expression. This combination of behaviors is usually not
seen in other degenerative dementias.
References:
Miller BL, Cummings JL, Boone K, et al. Emergence of artistic talent in frontotemporal dementia. Neurology
1998;51:978-981.
Question 31: Behavioral/Psychiatry - Behavioral Complications of Systemic Disease
Discussion:
Wilson’s disease is an autosomal recessive disorder caused by a mutation in the ATP7B gene on chromosome
13. Clinical symptoms are neurological, psychiatric, hepatic, or ocular. Other disorders affecting basal ganglia
circuitry can mimic Wilson’s disease; however, only Wilson’s disease will manifest with copper abnormalities
on laboratory screens. These syndromes include pantothenate kinase disease (caused by a mutation in the
PANK2 gene) and Huntington’s disease (trinucleotide repeat disorder on chromosome 4).
References:
Online Mendelian Inheritance in Man (OMIM) TM [homepage on the Internet]. Baltimore: McKusick-Nathans
Institute for Genetic Medicine, Johns Hopkins University; Bethesda: National Center for Biotechnology
Information, National Library of Medicine; c2006 [cited 2006 Aug 4;]. Available from:
www.ncbi.nlm.nih.gov/omim/.
Kitzberger R, Madl C, Ferenci P. Wilson disease. Metab Brain Dis 2005;20:295-302.
Question 35: Behavioral/Psychiatry - Behavioral Complications of Systemic Disease
Discussion:
Paroxysmal autonomic instability with dystonia (PAID) is a common symptom cluster similar to malignant
hyperthermia and neuroleptic malignant syndrome. It commonly appears following severe traumatic or hypoxic
brain injury. Treatment generally consists of beta-adrenergic blockers, opioid analgesia, dopamine agonists, and
benzodiazepines. Dopamine antagonists can precipitate symptoms similar to PAID. Drugs acting on cholinergic
and serotonin systems have not been found to be effective.
References:
Blackman JA, Patrick PD, Buck ML, Rust RS. Paroxysmal autonomic instability with dystonia after brain
injury. Arch Neurol 2004;61:321-328.
Question 62: Behavioral/Psychiatry - Behavioral Complications of Systemic Disease
Discussion:
Bilateral hilar adenopathy, bilateral cranial nerve palsies, and an encephalopathy due to a basilar
meningoencephalitis is typical of neurosarcoid. Progressive multifocal leukoencephalopathy due to the JC virus,
12
2007 RITE Discussion & Reference Manual
herpes simplex virus encephalitis, and HIV dementia all are characterized by white matter or cortical lesions on
MRI. The case is inconsistent with strokes due to cardiac thrombi.
References:
Stern BJ. Neurological complications of sarcoidosis. Curr Opin Neurol 2004;17:311-316.
Question 67: Behavioral/Psychiatry - Behavioral Complications of Systemic Disease
Discussion:
This patient with traumatic brain injury would most likely have frontal lobe impairment given the nature of his
accident. Psychosis, visual hallucinations, delusions, and mania would all be atypical. Road rage, episodic
dyscontrol, impulsivity, intrusiveness, apathy and aggression would be more common findings.
References:
Fann JR, Katon WJ, Uomoto JM, Esselman PC. Psychiatric disorders and functional disability in outpatients
with traumatic brain injuries. Am J Psychiatry 1995;152:1493-1499.
Question 70: Behavioral/Psychiatry - Temporal-Limbic Syndromes
Discussion:
Klüver-Bucy syndrome results from bilateral temporal lesions involving the amydala nuclei. Clinical features
include hypermetamorphosis, hyperorality, hypersexuality, visual agnosia, and blunted emotional affect.
Aggression is not a component of the syndrome. Hypermetamorphosis occurs when an individual is overly
sensitive or acutely aware of minute stimuli in the environment, such as a speck of lint on someone's shirt or a
scrap of paper on the floor. Patient's with Klüver-Bucy syndrome may become preoccupied with these stimuli
by touching, picking or examining them, symptoms that are described as hypermetamorphosis.
References:
Mendez MF. Pick's disease. In: Feinberg TE, Farah MJ, editors. Behavioral neurology and neuropsychology.
2nd ed. New York: McGraw-Hill, 2003.
Question 75: Behavioral/Psychiatry - Language/Speech Abnormalities
Discussion:
Speech remains intact as language deteriorates with advancing Alzheimer's dementia, eventually producing an
aphasia in which the patient is fluent and paraphasic, their speech is empty, and they have limited
comprehension but repeat well, which is typical of transcortical sensory aphasia.
References:
Cummings JL, Darkins A, Mendez M, et al. Alzheimer's disease and Parkinson's disease: comparison of speech
and language alterations. Arch Neurol 1988;38:680-684.
Question 84: Behavioral/Psychiatry - Parietal Syndromes
Discussion:
Anosognosia (unawareness of deficit or illness) is usually seen associated with nondominant parietal lobe
lesions. Achromatopsia is found after lesions of the inferior lip of the occipital lobe. Limb kinetic apraxia is seen
after lesions of the anterior corpus callosum. Expressive aprosodia is seen after right frontal lesions. Semantic
aphasia is seen after dominant hemisphere lesions.
References:
Feinberg TE, Farah MJ. Behavioral neurology and neuropsychology. 2nd ed. New York: McGraw-Hill, 2003.
Question 86: Behavioral/Psychiatry - Occipital Syndromes
13
2007 RITE Discussion & Reference Manual
Discussion:
Disturbance with the recognition of color in one visual field, hemiachromatopsia, occurs only with inferior,
posterior occipital lesions.
References:
Damasio A, Tranel D, Rizzo M. Disorders of complex visual processing. In: Mesulam MM, editor. Principles of
behavioral and cognitive neurology. New York: Oxford University Press, 2000.
Question 96: Behavioral/Psychiatry - Neurobiology of Behavior
Discussion:
Depression is often associated with left anterior frontal vascular lesions.
References:
Cummings JL, Mega MS. Neuropsychiatry and behavioral neuroscience. New York: Oxford University Press,
2003.
Question 105: Behavioral/Psychiatry - Neurobehavioral/Neuropsychological Exam
Discussion:
Multiple system atrophy is a relatively uncommon disorder with a constellation of symptoms that include
parkinsonism and autonomic and/or cerebellar dysfunction. Cognitive impairment can be a frequent
manifestation of the syndrome and usually manifests with frontal executive dysfunction.
References:
Mendez MF, Cummings JL. Dementia: a clinical approach. 3rd ed. Philadelphia: Butterworth-Heinemann,
2003;260-263.
Question 110: Behavioral/Psychiatry - Dementia
Discussion:
Amyloid starts as an amyloid precursor protein. It is normally cleaved by a series of enzymes into a short
version that can easily be excreted by the body. In pathological conditions such as Alzheimer’s disease, amyloid
is incorrectly cleaved by beta-secretase and gamma-secretase. Presenilin-1 assists gamma-secretase in cleaving
the amyloid precursor protein. This abnormal cleaving results in amyloid aggregation that ultimately leads to
plaque formation.
References:
Hardy J, Selkoe DJ. The amyloid hypothesis of Alzheimer’s disease: progress and problems on the road to
therapeutics. Science 2002;297:353-356.
Question 111: Behavioral/Psychiatry - General Psychiatry
Discussion:
The loss of remote memory, including autobiographical memory, in the face of intact new learning ability is
consistent with psychogenic amnesia.
References:
Mendez MF, Cummings JL. Dementia: a clinical approach. 3rd ed. Philadelphia: Butterworth-Heinemann, 2003.
Question 112: Behavioral/Psychiatry - Dementia
Discussion:
Any patient with Alzheimer's disease who acutely develops symptoms of a confusional state and behavioral
14
2007 RITE Discussion & Reference Manual
Any patient with Alzheimer's disease who acutely develops symptoms of a confusional state and behavioral
changes first warrants a workup to look for the underlying cause. Even mild changes in metabolic status,
medications, or an infection such as of the urinary tract may precipitate confusion and behavioral changes.
References:
Mendez MF, Cummings JL. Dementia: a clinical approach. 3rd ed. Philadelphia: Butterworth-Heinemann,
2003;260-263.
Question 116: Behavioral/Psychiatry - Language/Speech Abnormalities
Discussion:
Conduction aphasia, also called associative aphasia, is a relatively rare form of aphasia caused by damage to the
nerve fibers in the arcuate fasciculus, which connects Wernicke's and Broca's areas. Patients with conduction
aphasia show the following characteristics: speech is fluent, comprehension remains good, oral reading is poor,
repetition is poor, transpositions of sounds within a word (television/velitision) are common. To understand the
symptoms, recall that Broca's area controls expression whereas Wernicke's area is responsible for
comprehension. When both areas are intact but the neural connections between them is broken, there is the
curious condition in which the patient can understand what is being said but cannot repeat it (or repeats it
incorrectly). Such a patient will also end up saying something inappropriate or wrong, realize his/her mistake,
but continue making further mistakes while trying to correct it.
References:
Cummings JL, Mega MS. Neuropsychiatry and behavioral neuroscience. New York: Oxford University Press,
2003.
Question 122: Behavioral/Psychiatry - Behavioral Complications of Systemic Disease
Discussion:
Korsakoff amnestic syndrome causes impairment in declarative memory (anterograde amnesia) and forgetting
of recent events (retrograde amnesia) with sparing of motor memory and semantic memory (memory for
meaning of words). Digit span remains normal in this syndrome.
References:
Cummings JL, Mega MS. Neuropsychiatry and behavioral neuroscience. New York: Oxford University Press,
2003.
Question 123: Behavioral/Psychiatry - Neurobehavioral/Neuropsychological Exam
Discussion:
The Clinical Dementia Rating (CDR) Scale is a dementia staging instrument used to rate cognitive function
along five levels of impairment from none to maximal (rated as 0, 0.5, 1, 2, or 3) in each of six domains: (1)
memory, (2) orientation, (3) judgment and problem solving, (4) function in community affairs, (5) home and
hobbies, and (6) personal care. (Personal care has no 0.5 impairment level.) Only impairment caused by
cognitive dysfunction is rated. Community affairs and home and hobbies assess instrumental activities of daily
living relevant to the individual and hence vary according to that person’s accustomed activities; examples
include job performance for those who still are employed and skills in driving, home repairs, household
finances, shopping, cooking, and card games. Personal care represents basic activities of daily living common to
almost all individuals (dressing, bathing and grooming, eating, and continence). Based on the collateral source
and participant interviews, a global CDR score is derived from individual ratings in each domain such that CDR
0 indicates no dementia and CDR 0.5, 1, 2, and 3 represent very mild--also referred to as mild cognitive
impairment (MCI), mild, moderate, and severe dementia, respectively. Interrater reliability for the CDR has
been established at about 88%. Not all domains need be rated at the same level of impairment as the global CDR
score; for example, a participant may merit a box score of 1 for memory but scores of 0.5 or 0 for other domains
and still have a global CDR of 0.5. The individual ratings can be totaled to yield the sum boxes, a more
quantitative rating that ranges from 0 (or no impairment in any of the 6 domains) to 18 (or maximal impairment
in each of the 6 domains).
15
2007 RITE Discussion & Reference Manual
References:
Morris JC, Ernesto C, Schaefer K, et al. Clinical dementia rating (CDR) training and reliability protocol: the
Alzheimer Disease Cooperative Study Unit experience. Neurology 1997;48:1508-1510.
Question 141: Behavioral/Psychiatry - Dementia
Discussion:
The Dementia with Lewy bodies (DLB) Consortium has revised criteria for the clinical and pathologic diagnosis
of DLB incorporating new information about the core clinical features and suggesting improved methods to
assess them. REM sleep behavior disorder, severe neuroleptic sensitivity, and reduced striatal dopamine
transporter activity on functional neuroimaging are given greater diagnostic weighting as features suggestive of
a DLB diagnosis. When any of these are present with one of the primary findings of visual hallucinations,
parkinsonism, or fluctuating attention, then the diagnosis of probable DLB is supported.
References:
McKeith IG, Dickson DW, Lowe J, et al. Diagnosis and management of dementia with Lewy bodies: third
report of the DLB consortium. Neurology 2005;65:1863–1872.
Question 146: Behavioral/Psychiatry - Dementia
Discussion:
Pick's disease is usually manifested by disinhibition, socially inappropriate behavior, and sometimes the
Kluver-Bucy syndrome. Although some patients may develop depression, it is far less likely than the incidence
of depression in Parkinson's, Wilson's, Huntington's disease, and multiple sclerosis.
References:
Mendez MF, Cummings JL. Dementia: a clinical approach. 3rd ed. Philadelphia: Butterworth-Heinemann, 2003.
Question 147: Behavioral/Psychiatry - Psychopharmacology
Discussion:
Bupropion has had a low incidence of erectile dysfunction associated with its use. All of the selective serotonin
reuptake inhibitors (SSRIs) have been reported to have erectile dysfunction as a side effect. Amitriptyline and
venlafaxine also cause erectile dysfunction.
References:
Arana GW, Rosenbaum JF. Handbook of psychiatric drug therapy. 5th ed. Philadelphia: Lippincott, Williams &
Wilkins, 2005.
Question 149: Behavioral/Psychiatry - Language/Speech Abnormalities
Discussion:
An aphasia is considered fluent if: word output per minute is high; there are five or more words per phrase;
content per phrase is low; paraphasias are present; and speech is nondysarthric with normal prosody.
References:
Benson DF, Ardila A. Aphasia: a clinical perspective. Oxford: Oxford University Press, 1996.
Question 167: Behavioral/Psychiatry - Parietal Syndromes
Discussion:
Asomatognosia is a form of neglect in which patients deny ownership of their limbs; it frequently accompanies
anosognosia. The lesion is generally located in the nondominant supramarginal gyrus.
16
2007 RITE Discussion & Reference Manual
References:
Meador KJ, Loring DW, Feinberg TE, et al. Anosognosia and asomatognosia during intracarotid amobarbital
inactivation. Neurology 2000;55:816-820.
Feinberg TE, Laber LD, Needs NE. Verbal asomatognosia. Neurology 1990;40:1391-1394.
Question 178: Behavioral/Psychiatry - Neurobiology of Behavior
Discussion:
Several neurotransmitters and hormones have been implicated in the modulation of violent behavior. Most recent
evidence has found low levels of CSF 5-HIAA in patients who have attempted suicide via violent means as well
as in alcoholics with impulsive violent behavior. Norepinephrine and COMT have also been implicated in
aggressive behavior.
References:
Volavka J. The neurobiology of violence: an update. J Neuropsychiatry Clin Neurosci 1999;11:307-314.
Question 179: Behavioral/Psychiatry - Behavioral Complications of Systemic Disease
Discussion:
Numerous neuropsychiatric symptoms have been associated with epilepsy, particularly with temporal lobe
epilepsy. The symptoms include psychosis, fear, anxiety, hypergraphia, hypermorality, and altered sexual
function. The most commonly reported and readily treatable symptom is depression.
References:
Bortz JJ. Neuropsychiatric and memory issues in epilepsy. Mayo Clin Proc 2003;78:781-787.
Jones JE, Hermann BP, Barry JJ, et al. Clinical assessment of Axis I psychiatric morbidity in chronic epilepsy: a
multicenter investigation. J Neuropsychiatry Clin Neurosci 2005;17:172-179.
Question 194: Behavioral/Psychiatry - Frontal Systems Syndromes
Discussion:
The thalamus is a major relay station for most inherent functions of the brain to include cognition. This nuclear
structure can be subdivided into regions based on functional relationships. Lesions in specific subnuclei can lead
to different clinical manifestations. For example, lesions in the anterior group are more likely to manifest
amnesia, confabulation, anomia and preserved visuospatial function. Paramedian thalamic lesions can manifest
wtih acute decreased consciousness followed by vertical gaze paresis, disinhibition and at times amnesia.
References:
Carrera E, Bogousslavsky J. The thalamus and behavior: effects of anatomically distinct strokes. Neurology
2006;66(12):1817-1823.
Question 197: Behavioral/Psychiatry - Frontal Systems Syndromes
Discussion:
Personality changes associated with dorsolateral frontal dysfunction include apathy, self-absorption,
perseveration, neurovegetative symptoms (eg, eating and sleeping disturbances), irritability, and agitation.
Psychosis is usually seen with temporal lobe dysfunction. Obsessive-compulsive traits, disinhibition,
hypersexuality, and intrusiveness are typical of lesions affecting the orbitofrontal cortex.
References:
Miller BL, Cummings JL, editors. The human frontal lobes: Functions and disorders. New York: The Guilford
Press, 1999.
17
2007 RITE Discussion & Reference Manual
Question 198: Behavioral/Psychiatry - General Psychiatry
Discussion:
Factitious disorder is defined as a syndrome of intentional production of psychological or physical symptoms in
the absence of external incentives but in the presence of a psychological need to assume the sick role. When
there are external incentives for the behavior, then malingering is the likely diagnosis. Amnestic disorder is
when the individual has difficulties learning new things. Conversion symptoms are subconscious and not
intentionally produced. Somatoform disorder refers to individuals who have recurrent and multiple somatic
complaints not due to any physical disorder.
References:
American Psychiatric Association. Diagnostic and Statistical Manual of Mental Disorders. 4th ed., text revision.
Washington DC: American Psychiatric Association, 2000.
Question 317: Behavioral/Psychiatry - Dementia
Discussion:
Numerous studies have found a serotonergic deficit in patients with frontotemporal dementia (FTD). Experts in
the field will often treat these patients with selective serotonin reuptake inhibitors (SSRIs) even in the absence of
depression. There is no evidence of a cholinergic deficit in FTD, and studies evaluating the efficacy of
cholinesterase inhibitors have been largely neutral or negative. Purported neuroprotective and antioxidant
compounds have also not been found to be beneficial.
References:
Graff-Radford N, Woodruff B. Frontotemporal dementia. Continuum 2004;10:58-80.
Litvan I. Therapy and management of frontal lobe dementia patients. Neurol 2001;56(Suppl 4):S41-S45.
Question 320: Behavioral/Psychiatry - Developmental Disorders
Discussion:
Attention deficit hyperactivity disorder (ADHD) is a highly heritable, disruptive, childhood-onset condition, the
etiology and pathogenesis of which is poorly understood. There have been relatively few genome-wide linkage
studies, and no chromosomal region has yet been unequivocally implicated. In contrast, evidence from
pharmacological, neuroimaging, and animal studies has suggested the involvement of specific neurotransmitter
systems, notably dopaminergic pathways. Meta-analyses or pooled data analyses have supported association
between ADHD and polymorphisms in DRD4, DRD5, and SLC6A3, which encode dopamine D4 and D5
receptors and the dopamine transporter, respectively.
References:
Waldman ID, Gizer IR. The genetics of attention deficit hyperactivity disorder. Clin Psychol Rev
2006;26:396-432.
Question 324: Behavioral/Psychiatry - Psychopharmacology
Discussion:
Olanzapine is an atypical antipsychotic that frequently causes significant weight gain. Quetiapine, risperidone,
haloperidol, and molindone are less likely to do so.
References:
Puzantian T, Stimmel G. Review of psychotropic drugs. New York: McMahon, 2001.
Question 328: Behavioral/Psychiatry - Behavioral Complications of Systemic Disease
Discussion:
18
2007 RITE Discussion & Reference Manual
Prion proteins cause spongiform encephalopathy characterized by rapidly progressive dementia with myoclonus
and seizures. Progressive multifocal leukoencephalopathy caused by the JC virus traditionally presents with
encephalopathy, vision loss, paralysis, and ataxia. This syndrome, like human immunodeficiency virus (HIV),
usually presents in patients who are immunocompromised. The neurological manifestations of HIV are many
and include myelopathy, sensory neuropathy, and dementia. Tropheryma whippelii is the organism identified as
the cause of Whipple’s disease. Patients typically present with gastrointestinal complaints including diarrhea,
malabsorption, and weight loss. Other symptoms may include lymphadenopathy, hyperpigmented skin,
movement disorders, oculomasticatory myodysrhythmia, and dementia. Herpes simplex virus typically presents
with changes in personality and seizures; gastrointestinal symptoms are uncommon.
References:
Manzel K, Tranel D, Cooper G. Cognitive and behavioral abnormalities in a case of central nervous system
Whipple disease. Arch Neurol 2000;57:399-403.
Question 334: Behavioral/Psychiatry - Dementia
Discussion:
Most patients with Alzheimer’s disease (AD) have a sporadic late-onset form of the disease. A small percentage
of patients, however, have familial disease produced by one of three autosomal dominant genes. Among familial
AD patients, 50% to 70% have the presenilin-1 mutation or an associated mutation on chromosome 14. Five
percent to 10% have the presenilin-2 mutation. A small percentage will have a mutation in the gene that codes
for amyloid precursor protein.
References:
Bird TD. Genetic factors in Alzheimer’s disease. N Eng J Med 2005;352:862-864.
Question 339: Behavioral/Psychiatry - Temporal-Limbic Syndromes
Discussion:
Often, pseudoseizures are considered when bilateral rhythmic motor output occurs without loss of
consciousness, especially when an EEG captures the “spell” and no electrophysiological correlate is found with
scalp or sphenoidal leads. However, the midportion of the anterior cingulate (adjacent to the supplementary
motor cortex) has a bilateral motor homunculus that, when affected by an ictal focus, will produce bilateral
rhythmic motor output (without loss of consciousness) detectable only with subdural brain surface electrodes.
This limbic focus often manifests as a primary psychiatric disorder escaping routine neurological surveillance.
References:
Devinsky O, Morrell MJ, Vogt BA. Contributions of anterior cingulate cortex to behavior. Brain
1995;118:279-306.
Question 343: Behavioral/Psychiatry - Language/Speech Abnormalities
Discussion:
Numerous deficits of recognition following right hemisphere damage have been described. These syndromes
generally do not involve deficits in discrimination and result from lesions outside of unimodal cortex.
Prosopagnosia, a deficit in facial recognition, is probably the most well known. Lesser known syndromes
include auditory agnosia, a deficit in recognition of verbal and nonverbal sounds; autotopagnosia, which
represents the inability to localize stimuli on the affected side of the body; and phonagnosia, which is the
inability to recognize familiar voices. Pure word deafness, unlike the others, results from lesions of the dominant
hemisphere and leaves patients with the inability to recognize spoken language with spared nonverbal
communication and fluency.
References:
Shah NJ, Marshall JC, Zafiris O, et al. The neural correlates of person familiarity. A functional magnetic
resonance imaging study with clinical implications. Brain 2001;124:804-815.
19
2007 RITE Discussion & Reference Manual
Van Lancker DR, Krieman J, Cummings J. Voice perception deficits: neuroanatomical correlates of
phonagnosia. J Clin Exp Neuropsychol 1989;11:665-674.
Question 358: Behavioral/Psychiatry - Dementia
Discussion:
Although chromosome 3 and 9 have been linked to kindreds with frontotemporal dementia (FTD), the
overwhelming majority of patients with FTD have been found to have mutations on chromosome 17. They may
also demonstrate features of parkinsonism. Chromosome 17 harbors the gene for tau and at least 30 mutations
have been discovered in this gene among 100+ families. Chromosomes 12 and 19 have been implicated in
Alzheimer’s disease.
References:
Mendez MF, Cummings JL. Dementia: a clinical approach. 3rd ed. Philadelphia: Butterworth-Heinemann, 2003.
Sobrido MJ, Wiedau-Pazos M, Geschwind DH. The genetics of frontotemporal dementia and related disorders.
Current Genomics 2000;1:339-352.
Question 363: Behavioral/Psychiatry - Behavioral Complications of Systemic Disease
Discussion:
Numerous neurological symptoms can be seen associated with systemic lupus erythematosis (SLE). These
manifestations include peripheral neuropathy and cerebritis as well as neuropsychiatric symptoms such as
depression, mania, and psychosis. SLE is more frequently associated with psychosis than are Bechet's syndrome,
Ehlers-Danlos syndrome, rheumatoid arthritis, or Sjogren's syndrome.
References:
Cummings JL, Mega MS. Neuropsychiatry and behavioral neuroscience. New York: Oxford University Press,
2003.
Question 364: Behavioral/Psychiatry - Neurobehavioral/Neuropsychological Exam
Discussion:
Numerous neuropsychological tests have been devised that target predominantly one cognitive domains. These
include tests of visuospatial abilities for parietal lobe functions, memory for mesial temporal structures and tests
of sustained and complex attention for frontal lobe/executive function.
References:
Cummings JL, Mega MS. Neuropsychiatry and behavioral neuroscience. New York: Oxford University Press,
2003.
Question 370: Behavioral/Psychiatry - Behavioral Complications of Systemic Disease
Discussion:
HIV infection can result in minor cognitive and motor disorder, HIV-associated mild neurocognitive disorder,
and HIV-associated dementia. The earliest symptoms revolve around mental slowing and processing speed.
Tests that assess processing speed, including trails A and B, grooved pegboard, Symbol Digit Modalities Test,
and the HIV Dementia Scale are likely to be abnormal early in the disease course.
References:
Mendez MF, Cummings JL. Dementia: a clinical approach. 3rd ed. Philadelphia: Butterworth-Heinemann, 2003.
Question 371: Behavioral/Psychiatry - Language/Speech Abnormalities
Discussion:
20
2007 RITE Discussion & Reference Manual
Several disorders of language can result following damage to the dominant hemisphere. Detailed examination of
the six components of the language examination (fluency, comprehension, repetition, reading, writing, and
naming) is essential to help distinguish them. Wernicke’s aphasia is characterized by fluent speech with
impaired comprehension of written and spoken words. Patient’s with aphemia are relatively nonfluent in spoken
language; however, comprehension and written communication are much better preserved. The hallmark of
conduction aphasia is impaired repetition with relative sparing of other components of language. Patients who
suffer from pure word deafness are unable to repeat or comprehend spoken language; however, they can still
communicate effectively via writing.
References:
Benson DF, Ardila A. Aphasia: a clinical perspective. New York: Oxford University Press, 1996.
Question 374: Behavioral/Psychiatry - General Psychiatry
Discussion:
Catatonia is a syndrome manifested by a number of motor and neurobehavioral features. It may have a
"retarded-stuporous" form or an "excited-delirious" form. It may be seen in over 10% of inpatient psychiatric
patients. Catatonia is more prevalent in mood disorders than in schizophrenia. The most common mood disorder
in which it is seen is bipolar. Catalepsy, waxy flexibility, echophenomena, and negativism including mutism are
common. Many neurological and systemic illnesses may also present as catatonia. Treatments include
benzodiazepines, barbiturates, and electroconvulsive therapy. Dopamine antagonists as well as baclofen may
worsen the condition.
References:
Taylor MA, Fink M. Catatonia in psychiatric classification: a home of its own. Am J Psychiatry
2003;160:1233-1241.
Question 381: Behavioral/Psychiatry - Neurobiology of Behavior
Discussion:
Akinetic mutism may result from anterior cingulate lesions or a disconnection of the limbic connections
projecting from the anterior cingulate through subcortical circuits. Based on nonhuman primate primate tracer
studies, ventral pallidal lesions should disrupt the anterior cingulate frontal-subcortical circuit. A patient will
develop a rigid akinetic mute state caused by bilateral lesions of the globus pallidus interna with ventral
extension.
References:
Cummings JL, Mega MS. Neuropsychiatry and behavioral neuroscience. New York: Oxford University Press,
2003.
Question 393: Behavioral/Psychiatry - Dementia
Discussion:
A posterior parietal-occipital functional defect is often seen in dementia with Lewy bodies, unlike Alzheimer's
disease, which typically has a temporal parietal functional defect.
References:
McKeith IG, Dickson DW, Lowe J, et al. Diagnosis and management of dementia with Lewy bodies: third
report of the DLB consortium. Neurology 2005;65:1863–1872.
Question 402: Behavioral/Psychiatry - Developmental Disorders
Discussion:
Tuberous sclerosis is a genetic disorder associated with numeorus skin and systemic manifestations as well as
21
2007 RITE Discussion & Reference Manual
intracranial tubers, mental retardation, and seizures. Although numerous neuropsychiatric symptoms have been
reported to be associated with tuberous sclerosis, autism spectrum disorder is the most common.
References:
Wiznitzer M. Autism and tuberous sclerosis. Child Neurol 2004;19:675-679.
Question 411: Behavioral/Psychiatry - Psychopharmacology
Discussion:
The serotonin syndrome results from concomitant administration of medications that enhance serotonin
transmission via decreased breakdown or increased production. Medication combinations to use cautiously
include monoamine oxidase inhibitor agents with selective serotonin reuptake inhibitors, tricyclic
antidepressants, or dextromethorphan. The serotonin syndrome can be differentiated from neuroleptic malignant
syndrome by the presence of shivering and myoclonus in the former.
References:
Boyer EW, Shannon M. Current Concepts: the serotonin syndrome. New Eng J Med 2005;352:1112-1120.
Christensen RC. Identifying serotonin syndrome in the emergency department. Am J Emerg Med
2005;23:406-408.
Question 436: Behavioral/Psychiatry - Dementia
Discussion:
Subcortical dementia is characterized clinically by psychomotor slowing, forgetfulness, cognitive decline,
visuospatial impairment, and personality changes, especially in mood. Bradyphrenia (slowness of mental
processing) is very common.
References:
Mendez MF, Cummings JL. Dementia: a clinical approach. 3rd ed. Philadelphia: Butterworth-Heinemann, 2003.
Clinical Adult
Question 3: Clinical Adult - Neuromuscular Disorders
Discussion:
Charcot-Marie-Tooth disease (CMT) 1A typically shows distal weakness, nerve hypertrophy, and pes cavus
associated with a duplication of the PMP22 gene. In hereditary neuropathy with liability to pressure palsies
(HNPP), there is a deletion of the PMP22 gene. CMT 2 is the axonal phenotype.
References:
Nicholson GA. The dominantly inherited motor and sensory neuropathies: clinical and molecular advances.
Muscle Nerve 2006;33:589-597.
Question 4: Clinical Adult - Dementia
Discussion:
According to the guidelines by the NINCDS-ADRDA, the routine evaluation of the patient with dementia
should include the following laboratory tests: (1) complete blood count, (2) serum electrolytes, (3) glucose, (4)
blood urea nitrogen/creatinine, (5) serum B12 levels, (6) depression screening, (7) liver function test, and (8)
thyroid function test. Venereal Disease Research Laboratory (VDRL), HIV, lumbar puncture, and heavy metal
screen are not recommended in routine dementia screening without clinical indication.
References:
22
2007 RITE Discussion & Reference Manual
Dubinsky RM, Stein AC, Lyons K. Practice parameter: risk of driving and Alzheimer's disease (an
evidence-based review): report of the Quality Standards Subcommittee of the American Academy of Neurology.
Neurology 2000;54(12):2205-2211.
Question 23: Clinical Adult - Movement Disorders
Discussion:
The onset of orofacial dyskinesias with lingual and oral dystonia in a 30-year-old patient is characteristic of
neuroacanthocytosis, which may also be associated with chorea and peripheral polyneuropathy.
References:
Bradley WG, Daroff RB, Fenichel GM, et al, editors. Neurology in clinical practice. 3rd ed. Boston:
Butterworth-Heinemann, 2000.
Question 33: Clinical Adult - Neurology of Systemic Disease
Discussion:
Anticoagulation may produce spontaneous hemorrhage into the psoas or iliacus muscle and produce and acute
iliofemoral neuropathy. CT of the abdomen an pelvis is most useful for imaging acute blood in the
retroperitoneum.
References:
Seijo-Martinez M. Acute femoral neuropathy secondary to an iliacus muscle hematoma. J Neurol Sci 2003;
209(1-2): 119-122.
Question 40: Clinical Adult - Headache
Discussion:
Valproic acid, amytriptilline, and propranolol have the best proven benefit as prophylactic agents. Drug choice
should be balanced with the patient's comorbidities. AAN practice guidelines have evaluated evidence-based
data for acute and prophylactic therapy based upon stastical and clinical benefit in published clinical trials.
References:
Silberstein S. Practice parameter: Evidence-based guidelines for migraine headache (an evidence-based review).
Report of the Quality Standards Subcommittee of the American Academy of Neurology. Neurology 2000; 55:
754-763.
Question 49: Clinical Adult - Cerebrovascular Disease
Discussion:
In the syndrome of alexia without agraphia, a complete right homonymous hemianopsia is present in many
cases, but there are exceptions. Impaired naming and understanding of color names in the presence of intact or
nearly intact color vision is common, although some patients suffer from an actual impairment of color vision.
Mild anomia is common but not always present. In most cases, there are no other aphasic disturbances or
abnormalities of the primary motor or sensory systems. The most frequently reported pathology is occlusion of
the dominant (left) posterior cerebral artery.
References:
Brazis PW, Masdeu JC, Biller J. Localization in clinical neurology. 4th ed. Philadelphia: Lippincott, Williams &
Wilkins, 2001.
Question 53: Clinical Adult - Motor Neuron/Nerve
Discussion:
23
2007 RITE Discussion & Reference Manual
This patient has peroneal compression neuropathy due to significant weight loss and leg crossing. The
appropriate management would be to caution him against leg crossing.
References:
Rubin DI, Kimmel DW, Cascino TL. Outcome of peroneal neuropathies in patients with systemic malignant
disease. Cancer 1998;83:1602-1606.
Question 58: Clinical Adult - Critical Care/Trauma
Discussion:
This patient has raised intracranial pressure most probably due to a developing epidural hematoma. Pending
definitive treatment with a neurosurgical procedure, intracranial pressure should be managed by putting the
patient’s head up 40 degrees, intubation and hyperventilation, and administering IV mannitol.
References:
Wijdicks, EF. Neurologic Catastophies in the Emergency Department. Woburn, Mass: Butterworth-Heinemann,
2000.
Question 65: Clinical Adult - Spinal and Root Disorders
Discussion:
This patient has a C5 to C6 disc herniation producing a relatively mild acute C6 radiculopathy. The diagnosis is
clear from the information provided, and no additional diagnostic testing (EMG, myelography) should be
required. In most cases the symptoms will resolve spontaneously without need for cervical discectomy. Surgery
would be required if the patient had significant radicular weakness, cervical myelopathy, or intractable radicular
pain. Epidural steroid injection does not have a defined role in the treatment of radiculopathy but may improve
pain control in patients with radicular pain that is severe or does not respond to oral analgesics.
References:
Noseworthy J. Neurological therapeutics principles and practice. 2nd ed. London: Informa Healthcare, 2006.
Question 68: Clinical Adult - Cerebrovascular Disease
Discussion:
Cerebral amyloid angiopathy usually affects the elderly and accounts for up to 10% of intracranial hemorrhages.
The typical location of the hemorrhages is in the lobar areas. The deposition of beta amyloid protein in the
media and adventitia of small meningeal and cortical vessels result in lobar hemorrhages that frequently recur.
References:
Victor M, Ropper A. Adams and Victor's principles of neurology. 7th ed. New York: McGraw-Hill, 2001.
Question 71: Clinical Adult - Neuromuscular Disorders
Discussion:
Polymyositis often presents as an acquired progressive proximal weakness with elevated creatine kinase levels
and EMG demonstrating fibrillations and small polyphasic potentials.
References:
Engel AG, Franzini-Armstrong C. Myology. 3rd ed. New York: McGraw-Hill, 2004.
Question 72: Clinical Adult - Neuro-ophthalmology/Neuro-otology
Discussion:
A lesion of the right upper bank of the calcarine fissure results in a homonymous left inferior quadrantanopsia.
24
2007 RITE Discussion & Reference Manual
References:
Haines DE. Fundamental Neuroscience. 3rd ed. Philadelphia: Churchill Livingstone/Elsevier 2006; 251.
Question 95: Clinical Adult - Neuromuscular Disorders
Discussion:
When the examiner is testing the biceps strength, the subject's forearm should be held in supination to eliminate
elbow flexion force produced by the brachioradialis muscle.
References:
Haerer AF. Dejong's The neurologic examination. 5th ed. Philadelphia: Lippincott, Williams & Wilkins, 1992.
Question 100: Clinical Adult - Movement Disorders
Discussion:
This patient has drug-induced parkinsonism due to use of metoclopramide, a dopamine receptor antagonist. The
other medications would not produce parkinsonism.
References:
Noseworthy J. Neurological therapeutics principles and practice. 2nd ed. London: Informa Healthcare, 2006.
Question 102: Clinical Adult - Cerebrovascular Disease
Discussion:
This patient has a cerebral infarct in the distribution of the right anterior cerebral artery. If contrast is
administered, subacute infarcts commonly enhance, usually with a gyriform pattern.
References:
Ropper AH, Brown RH. Adams and Victor's principles of neurology. 8th ed. New York: McGraw-Hill, 2005;
261-264.
Question 108: Clinical Adult - Demyelinating Disease
Discussion:
Based on the history and examination, this patient most likely has optic neuritis. By history, she has had
multiple episodes of neurological dysfunction separated temporally and anatomically. This patient most likely
has demyelinating disease. The CSF immunoglobulin G index and oligoclonal bands are typically elevated in
the CSF of a patient with acute optic neuritis. Sarcoidosis and Lyme disease can also produce optic neuritis but
are less common in a patient of this age and gender.
References:
Noseworthy J. Neurological therapeutics principles and practice. 2nd ed. London: Informa Healthcare, 2006.
Question 113: Clinical Adult - Neuromuscular Disorders
Discussion:
The pronator quadratus is responsible for pronation of the forearm when the elbow is flexed. It is supplied by the
anterior interosseous nerve, a branch of the median nerve.
References:
Daube JR, editor. Clinical neurophysiology. Philadelphia: FA Davis, 1996;81-84.
Question 114: Clinical Adult - Demyelinating Disease
25
2007 RITE Discussion & Reference Manual
Question 114: Clinical Adult - Demyelinating Disease
Discussion:
The daughter of a parent with multiple sclerosis has about a 5% chance of developing the disease. This
percentage is higher than siblings, with the exception of monozygotic twins.
References:
Miller A, editor. Multiple Sclerosis. Continuum: Lifelong Learning Neurology 1999;5(5):8(part B).
Question 118: Clinical Adult - Epilepsy
Discussion:
Levetiracetam, gabapentin, tiagabine, vigabatrin, zonisamide, and topiramate (<200 mg) all appear to have no
effect on steroid hormone concentration, such as oral contraceptive concentration. Contraceptive failure is
higher (6% to 20% per year) when lamotrigine, phenytoin, phenobarbital, or carbamazepine is used.
References:
Rationale for the Clinical Use of the New Over the Older Antiepileptic Drugs:. Proceedings of a Symposium at
the 32nd Annual Meeting of the Southern Clinical Neurological Society. January 15, 2005; Oaxaca, Mexico.
Question 124: Clinical Adult - Critical Care/Trauma
Discussion:
The formation of an epidural hematoma may be delayed by minutes to 16 days after injury. Epidural hematomas
are more common in younger than in elderly patients because the dura is more adherent to the inner table of the
skull in elderly patients.
References:
Narayan RK, Wilberger JE Jr, Povlishock JT. Neurotrauma. New York: McGraw-Hill, 1996.
Question 128: Clinical Adult - Movement Disorders
Discussion:
Episodic ataxia type 2 is an autosomal dominant disorder characterized by recurrent attacks of unsteady gait,
limb ataxia, and dysarthria lasting minutes to hours. Between attacks, patients may have nystagmus and mild gait
ataxia, but the neurological examination is otherwise normal. Oral acetazolamide usually prevents or reduces the
frequency of the attacks. It is associated with a mutation of the CACNA1A gene on chromosome 19.
References:
Jen J, Kim GW, Baloh RW. Clinical spectrum of episodic ataxia type 2. Neurology 2004;62:17–22.
Question 129: Clinical Adult - Neuro-ophthalmology/Neuro-otology
Discussion:
The patient presents with symptoms and laboratory abnormalities consistent with the diagnosis of acute optic
neuritis. Based on the Optic Neuritis Treatment Trial, IV methylprednisolone 1 g/kg/d for 3 days followed by
an oral prednisone taper has been shown to be effective in accelerating the recovery of vision and reducing
ocular pain and discomfort.
References:
Dubinsky RM, Stein AC, Lyons K. Practice parameter: risk of driving and Alzheimer's disease (an
evidence-based review): report of the Quality Standards Subcommittee of the American Academy of Neurology.
Neurology 2000;54(12):2205-2211.
26
2007 RITE Discussion & Reference Manual
Question 143: Clinical Adult - Headache
Discussion:
Divalproex sodium is considered an effective prophylactic medication for migraines. Carbamazepine has not
proven to be as effective and is reserved more for the treatment of painful peripheral neuropathies. The other
medication choices (naratriptan, naproxen, and cyproheptadine) are used in the acute therapy of migraines.
References:
Silberstein S. Practice Parameter: Evidence-based guidelines for migraine headache (an evidence-based review):
Report of the Quality Standards Subcommittee of the American Academy of Neurology. Neurology
2000;55:754-762.
Question 144: Clinical Adult - Neurotoxicology
Discussion:
This patient most likely has been exposed to an anticholinesterase toxin in liquid or gas form. The fact that the
intoxication occurred in a crowded, enclosed, public space suggests terrorism. The clinical findings would be
different with carbon monoxide, nitrous oxide, methane, or cyanide poisoning.
References:
Newmark J. Nerve agents. Neurol Clin 2005;23(2):623-641.
Question 153: Clinical Adult - Headache
Discussion:
Paroxysmal hemicrania is a disorder, more common in women, characterized by frequent (7 to 22 per day)
episodes of unilateral, severe, but short-lasting (5 to 45 minutes) headaches associated with ipsilateral
autonomic manifestations. Indomethacin is the treatment of choice.
References:
Goadsby PJ, Lipton RB. A review of paroxysmal hemicranias, SUNCT syndrome and other short-lasting
headaches with autonomic features, including new cases. Brain 1997;120:193-209.
Question 161: Clinical Adult - Neuro-oncology
Discussion:
A seizure with postictal hemiparesis in a patient with known cancer should be presumed to be related to brain
metastases until proven otherwise. Paraneoplastic limbic encephalitis would be much less likely, and
hypertensive encephalopathy, hypoglycemia, and hyponatremia would be unlikely to produce focal deficits.
Moreover, the patient does not have severe enough hypertension, hypoglycemia, or hyponatremia to produce a
seizure.
References:
Noseworthy J. Neurological therapeutics principles and practice. 2nd ed. London: Informa Healthcare, 2006.
Question 163: Clinical Adult - Epilepsy
Discussion:
This is a single unprovoked generalized seizure in a patient with a normal neurological examination and a
normal evaluation. The etiology is likely idiopathic, and his risk for recurrent seizure is low (about 30% over the
next 5 years); therefore, observation is warranted.
References:
Sirven J. Antiepileptic Drug Therapy for Adults: When to Initiate and How to Choose. Mayo Clinic Proceedings
2002; 77(12):1367-1375.
27
2007 RITE Discussion & Reference Manual
2002; 77(12):1367-1375.
Marson A, Jacoby A, Johnson A, et al. Medical Research Council MESS Study Group. Immediate versus
deferred antiepileptic drug treatment for early epilepsy and single seizures: a randomized controlled trial. Lancet
2005;365(9476):2007- 2013.
Question 169: Clinical Adult - Neuro-oncology
Discussion:
This patient has Foster-Kennedy syndrome. Unilateral anosmia and optic neuropathy in conjunction with signs
of raised ICP (papilledema in the opposite eye) are the clinical features of this disorder, which is most
commonly due to a planum sphenoidale meningioma that could involve both the olfactory groove and optic
canal. The other disorders listed would not typically have this constellation of signs and symptoms.
References:
Demetriades AK. Victor Horsley's contribution to the Foster Kennedy syndrome. Brit J Neurosurg
2004;18(4):371-374.
Question 170: Clinical Adult - Spinal and Root Disorders
Discussion:
Saddle anesthesia, sphincter loss, and loss of ankle reflexes after a fall signify midline disk herniation with
compression of the caudal equina.
References:
Noseworthy J. Neurological therapeutics principles and practice. 2nd ed. London: Informa Healthcare, 2006.
Question 172: Clinical Adult - Headache
Discussion:
This patient has a history of migraine, a straightforward current history, and a normal neurological examination.
No features suggest elevated ICP or meningeal irritation. No testing is necessary in this setting.
References:
Edmeads J. Migraine. In: Noseworthy JH, editor. Neurologic therapeutics: principles and practice. New York:
Martin Dunitz, 2003;73-88.
Question 173: Clinical Adult - Cerebrovascular Disease
Discussion:
This patient most likely has a vertebral dissection. This disorder is more common is women. It often follows
neck trauma. When occurring spontaneously, it is often associated with a disorder of connective tissue, such as
fibromuscular dysplasia. The other disorders listed (disseminated intravascular coagulation, factor V Leiden
mutation, patent foramen ovale, and vasculitis) are less likely, particularly in view of the associated neck pain.
References:
Noseworthy J. Neurological therapeutics principles and practice. 2nd ed. London: Informa Healthcare, 2006.
Question 176: Clinical Adult - Other Pain Syndromes
Discussion:
Allodynia is the perception of a non-noxious stimulus as being painful. This is an associated feature common in
neuropathic pain. Incident pain refers to pain that occurs with movement rather than with touch. Paresthesia
refers to an abnormal spontaneous sensation that has no objective cause.
28
2007 RITE Discussion & Reference Manual
References:
Kanner R. Pain management secrets. Philadelphia: Hanley and Belfus, 1997;2.
Question 182: Clinical Adult - Critical Care/Trauma
Discussion:
Practice guidelines have been established for concussion in sports through the AAN. This athlete has
experienced one Grade II concussion (no loss of consciousness but concussion symptoms greater than 15
minutes) and a more recent Grade III concussion(any loss of consciousness). Guidelines state he should be out
of play for 1 week in this setting of recurrent concussion.
References:
Practice Parameter: Managing Concussion in Sports. Report of the Quality Standards Subcommittee. Neurology
1997;48:581-585.
Question 185: Clinical Adult - Spinal and Root Disorders
Discussion:
Patients with transverse myelitis are at risk, but may not necessarily develop, multiple sclerosis. Presence of a
partial rather than complete myelitis gives a higher risk for progression to multiple sclerosis. The presence of
subclinical evidence of demyelination on brain imaging at the time of initial event is the strongest predictor of
subsequent demyelinating disease. Multiple sclerosis remains a clinical diagnosis, and exclusion of other genetic
or autoimmune mimics is important.
References:
Lublin F, Tullman M, editors. Multiple sclerosis. Continuum: Lifelong Learning in Neurology 2004;10(6).
Question 189: Clinical Adult - Neuromuscular Disorders
Discussion:
Diazepam is the mainstay of treatment for the stiff person syndrome, which occurs about as often in women as
in men. Onset in adult life, proximal distribution of stiffness, development of lordosis, and precipitation by
motion or emotion are typical. Diabetes and organ-specific autoimmune disorders are common. Seizures
sometimes occur.
References:
Noseworthy J. Neurological therapeutics principles and practice. 2nd ed. London: Informa Healthcare, 2006.
Question 190: Clinical Adult - Cerebrovascular Disease
Discussion:
This patient has Anton's syndrome (denial of blindness). It is a result of severe acute bilateral injury to the
medial occipital lobes and adjacent association cortex, usually due to ischemia in the distribution of the
posterior cerebral arteries. The syndrome has also occured after traumatic injury to the optic nerve with
associated bifrontal lobe contusions.
References:
McDaniel KD, McDaniel LD. Anton's syndrome in a patient with posttraumatic optic neuropathy and bifrontal
contusions. Arch Neurol 1991;48 (1):101-105.
Brazis PW, Masdeu JC, Biller J. Localization in clinical neurology. 4th ed. Philadelphia: Lippincott, Williams &
Wilkins, 2001.
Question 196: Clinical Adult - Demyelinating Disease
29
2007 RITE Discussion & Reference Manual
Discussion:
After an episode of optic neuritis, the best predictor for subsequent MS is an abnormal MRI of the brain. CSF
findings do not add to the prognostic information available from brain MRI data.
References:
Jacobson D. Optic neuritis. In: Samuels MA, Feske SK, editors. Office practice of neurology. Philadelphia:
Churchill-Livingstone, 2003;416-420.
Question 200: Clinical Adult - Demyelinating Disease
Discussion:
Mitoxantrone is USFDA-approved for the treatment of secondary progressive multiple sclerosis and for
relapsing-progressive multiple sclerosis. The other treatments have no proven benefit for these stages of multiple
sclerosis.
References:
Goodin DS, Arnason BG, Coyle PK, et al. The use of mitoxantrone (Novantrone) for the treatment of multiple
sclerosis: Report of the Therapeutics and Technology Assessment Subcommittee of the American Academy of
Neurology. Neurology 2003;61:1332-1338.
Questions 204 - 208: Clinical Adult - Epilepsy
Discussion:
Choice of anticonvulsant therapy is based upon effectiveness of each drug in the reduction of specific seizure
types (ie, generalized vs complex partial vs absence) and also least risk of side effects. Lamotrigine has recently
been given a pregnancy category C rating, making it the safest selection for patients with refractory epilepsy in
pregnancy, providing the seizure frequency is a greater risk to the pregnancy and medication risk. Valproic acid
has a particular efficacy in absence seizures and juvenile myoclonic epilepsy. It also has the least risk of skin
hypersensitivity. Levetriacetam is considered the safest in the setting of liver disease. Phenobarbital is used in
the acute treatment of neonatal seizures. Carbamazepine is the initial monotherapy for a child with cryptogenic
complex partial seizures.
References:
Asconape J. Some common issues in the use of antiepileptic drugs. Semin Neurol 2002;22:27-40.
Wheless JW, Clarke DF, Carpenter D. Treatment of pediatric epilepsy: expert opinion. J Child Neurol
2005;20(Suppl 1):S1-S56.
Questions 219 - 223: Clinical Adult - Dementia
Discussion:
A subacute encephalopathy with multiple-enhancing MRI lesions and a high antinuclear antibody titer are
consistent with lupus cerebritis. Creutzfeldt-Jakob disease is a prion disorder that can present subacutely with a
syndrome of rapidly progressive dementia, ataxia, and stimulus-sensitive myoclonus. Joint laxity, ectopic lens,
and aortic dissection can be seen as a complication of Marfan syndrome, which also carries a typical phenotype
of tall stature and long arms. CADASIL is a genetic syndrome producing an arteriopathy that can present with
migraine, dementia, and characteristic white matter infarcts on MRI imaging. Vascular dementia is
distinguished from Alzheimer-type dementia primarily by stepwise decline in cognitive function.
References:
Gallagher C. Neurogenetics Review. Hospital Physician 2005;9(1):1-16.
Question 330: Clinical Adult - Headache
Discussion:
30
2007 RITE Discussion & Reference Manual
This patient most likely has orthostatic headache secondary to spontaneous intracranial hypotension. The usual
etiology is a CSF leak located in the spinal meninges. The leak is most readily identified with a myelogram. The
other tests listed (meningeal biopsy, serum angiotensin converting enzyme, slit lamp examination, and CSF
cytology) would not assist in the diagnosis of this syndrome.
References:
Schievink WI. Spontaneous spinal cerebrospinal fluid leaks and intracranial hypotension. JAMA 2006;
295(19):2286-2296.
Question 333: Clinical Adult - Neuromuscular Disorders
Discussion:
The clinical history and examination are most consistent with Lambert-Eaton myasthenic syndrome, which is
associated with malignancy (usually small cell lung cancer) in two thirds of patients.
References:
Noseworthy J. Neurological therapeutics principles and practice. 2nd ed. London: Informa Healthcare, 2006.
Question 335: Clinical Adult - Infectious Disease
Discussion:
This patient most likely has cerebral malaria, which is due to sludging of infected erythrocytes in the cerebral
capillaries. The CSF examination is not consistent with viral encephalitis nor has she been in a geographic area
in which Japanese encephalitis is endemic. A Giemsa-stained peripheral blood smear will demonstrate the
malaria parasite.
References:
Noseworthy J. Neurological therapeutics principles and practice. 2nd ed. London: Informa Healthcare, 2006.
Question 337: Clinical Adult - Cerebrovascular Disease
Discussion:
This patient has arteritic acute ischemic optic neuropathy, in which the findings are typically a swollen disc and
retinal pallor. The mechanism is likely to be giant cell arteritis based on his clinical history.
References:
Noseworthy J. Neurological therapeutics principles and practice. 2nd ed. London: Informa Healthcare, 2006.
Question 354: Clinical Adult - Headache
Discussion:
This patient has pseudotumor cerebri, which has been linked to use of isotretinoin and other vitamin
A–containing compounds. The other medications listed should not cause pseudotumor cerebri.
References:
Wall M. Papilledema and idiopathic intracranial hypertension (pseudotumor cerebri). In: Noseworthy JH, editor.
Neurologic therapeutics: principles and practice. New York: Martin Dunitz, 2003;1730-1742.
Question 361: Clinical Adult - Neuromuscular Disorders
Discussion:
This patients has symptoms and signs consistent with Guillain-Barre syndrome (GBS). IV immunoglobulin is
recommended for GBS in nonambulant patients within 2 to 4 weeks of onset of neuropathic symptoms.
References:
31
2007 RITE Discussion & Reference Manual
References:
Dubinsky RM, Stein AC, Lyons K. Practice parameter: risk of driving and Alzheimer's disease (an
evidence-based review): report of the Quality Standards Subcommittee of the American Academy of Neurology.
Neurology 2000;54(12):2205-2211.
Question 368: Clinical Adult - Headache
Discussion:
This patient has medication overuse headache related to daily use of an acetaminophen/caffeine/butalbital
medication. Although the other interventions might help her headache problem, they would be less likely to do
so than tapering the medication.
References:
Boes CJ, Black DF, Dodick DW. Pathophysiology and management of transformed migraine and medication
overuse headache. Seminars in Neurology 2006;26(2):232-241.
Question 376: Clinical Adult - Movement Disorders
Discussion:
In patients with Lewy body dementia, severe akinesia, dystonias, and neuroleptic malignant syndrome are
common reactions to even very low doses of the older, typical antipsychotics.
References:
Ballard C, Grace J, McKeith I, et al. Neuroleptic sensitivity in dementia with Lewy bodies and Alzheimer's
disease. Lancet 1998;351:1032-1033.
Question 384: Clinical Adult - Spinal and Root Disorders
Discussion:
Disturbance of the S1 nerve root results in pain radiating down the posterior aspect of the lower extremity,
sensory disturbance of the little toe, lateral foot, and most of the sole of the foot, weakness in several muscles
including the gastrocnemius and soleus muscles, and reduction or absence of the ankle jerk.
References:
Brazis PW, Masdeu JC, Biller J. Localization in clinical neurology. 4th ed. Philadelphia: Lippincott, Williams &
Wilkins, 2001.
Question 387: Clinical Adult - Neurogenetics
Discussion:
This patient most likely has Duchenne's muscular dystrophy, which is characterized by X-linked recessive
inheritance and a mutation of the dystrophin gene located on Xp21.2
References:
Noseworthy J. Neurological therapeutics principles and practice. 2nd ed. London: Informa Healthcare, 2006.
Question 398: Clinical Adult - Sleep
Discussion:
This patient most likely has restless legs syndrome. Medications that are most helpful for this disorder are
dopamine agonists, opioids, and gabapentin. Dopaminergic agents (including pramipexole, ropinarole, and
carbidopa-levadopa) usually work best with fewest side effects. Opioids are not the preferred agents, as they
may produce drug dependency and tolerance, requiring dose escalation to maintain drug effect.
32
2007 RITE Discussion & Reference Manual
References:
Noseworthy J. Neurological therapeutics principles and practice. 2nd ed. London: Informa Healthcare, 2006.
Question 403: Clinical Adult - Neurology of Systemic Disease
Discussion:
The most common neurological complication of scleroderma is carpal tunnel syndrome.
References:
Rolak LA. Neurological complications of progressive systemic sclerosis. In: Noseworthy J, editor. Neurological
therapeutics. London: Martin Dunitz, 2003.
Question 406: Clinical Adult - Infectious Disease
Discussion:
This is a case of progressive multifocal leukoencephalopathy (PML), caused by the JC virus. Highly active
antiretroviral therapy (HAART) is the only proven treatment of HIV-associated PML. Pyrimethamine is used to
treat toxoplasmosis. Amphotericin B is an antifungal agent. The other drugs (AraC and interferon alpha) are not
effective for HIV-associated PML.
References:
Evaluation and Management of Intracranial Mass Lesions in AIDS. Report of the Quality Standards
Subcommittee of the American Academy of Neurology. Neurology 1998;50:21-26.
Question 410: Clinical Adult - Dementia
Discussion:
The patient has many clinical features consistent with a frontal dementia. The trails B test is a measure of
executive function. It assesses sustained attention, visual scanning, and the ability to change sets (inhibit certain
stimuli while facilitating others). It is a timed task, so speed of performance is also important. Finger tapping
assesses motor speed. The Wechsler Memory Scale and California Verbal Learning Test assess memory. The
Boston Naming Test evaluates language.
References:
Bartleson JD, Schierman B, Edlund W, editors. Summaries of AAN Clinical Practice Guidelines for Adult
Neurology: Neuropsychological Testing. 2006-2006 ed. American Academy of Neurology
Feinberg TE, Farah MJ. Behavioral neurology and neuropsychology. 2nd ed. New York: McGraw-Hill, 2003.
Question 413: Clinical Adult - Demyelinating Disease
Discussion:
Adrenoleukodystrophy is an X-linked disorder characterized by defective beta oxidation and accumulations of
very-long chain fatty acids. This metabolic defect is detectable in heterozygotes and prenatally. The adult-onset
phenotype (progressive spastic paraparesis) is called adrenomyeloneuropathy.
References:
Moser HW, Raymond GV, Dubey P. Adrenoleukodystrophy: new approaches to a neurodegenerative disease.
JAMA 2005;294(24):3131-3134.
Question 424: Clinical Adult - Neurorehabilitation
Discussion:
Although the pattern of recovery following a stroke may be variable, the earliest evidence of a return of
33
2007 RITE Discussion & Reference Manual
neurological function is usually an increase in tone. This generally occurs before improvement in strength or a
return of reflexes.
References:
DeLisa JA, Gans BM. Rehabilitation medicine: Principles in Practice. 2nd ed. Philadelphia: Lippincott,
1993;458.
Question 427: Clinical Adult - Spinal and Root Disorders
Discussion:
This represents a Brown-Sequard syndrome with ipsilateral vibration/position sense involvement and
contralateral pain/temperature abnormality below the level of the lesion.
References:
Haines DE, editor. Fundamental Neuroscience. 3rd ed. Philadelphia: Churchill Livingstone, 2006;156.
Question 428: Clinical Adult - Neurogenetics
Discussion:
Examples of X-linked inheritance include Duchenne/Becker muscular dystrophy, Kennedy’s disease, fragile X
syndrome, adrenoleukodystrophy, and Fabry lipid storage disease. Characteristics of X-linked inheritance
include that males are almost exclusively affected but symptomatic female carriers can have a mild phenotype as
seen in Duchenne dystrophy. Transmission occurs through carrier females to their sons, each of whom have a
50% chance of being affected. Daughters of affected males are obligate carriers, and male-to-male transmission
does not occur.
References:
Gallagher C. Hospital physician neurology board review manual: neurogenetics review. Wayne, PA: Turner
White, 2005.
Souayah N, Khella S. Neurology examination and board review. New York: McGraw-Hill, 2005;221-230.
Question 432: Clinical Adult - Neuro-ophthalmology/Neuro-otology
Discussion:
The history and examination is most consistent with benign positional vertigo. The other disorders listed would
be associated with additional neurological deficits and/or the vertigo would be less likely to be associated with
positional changes.
References:
Ropper AH, Brown RH. Adams and Victor's principles of neurology. 8th ed. New York: McGraw-Hill, 2005;
261-264.
Blumenfeld H. Neuroanatomy through clinical cases. 1st ed. Sunderland: Sinauer Associates, Inc., 2002.
Question 438: Clinical Adult - Motor Neuron/Nerve
Discussion:
This patient most likely has motor neuron disease, characterized by a mixture of upper and lower motor neuron
signs in all four limbs. Cervical spinal stenosis could also produce painless four-limb weakness with upper motor
neuron signs in the legs. Fasciculations and wasting in the lower extremities are not symptoms of cervical
stenosis as they imply a disorder of the lower motor neuron at the lumbosacral level. Although patients with
cervical stenosis may have spine and radicular pain, this is not always the case, and the absence of pain does not
exclude cervical stenosis as a diagnosis.
References:
34
2007 RITE Discussion & Reference Manual
Noseworthy J. Neurological therapeutics principles and practice. 2nd ed. London: Informa Healthcare, 2006.
Question 441: Clinical Adult - Movement Disorders
Discussion:
A levodopa or apomorphine challenge should be considered when the diagnosis of Parkinson’s disease is in
doubt. The other tests listed (olfaction, repeat MRI, PET, and autonomic testing) are not clinically useful in
distinguishing Parkinson’s disease from other parkinsonian syndromes based upon insufficient evidence from
clinical studies.
References:
Practice parameter: diagnosis and prognosis of new onset Parkinson disease (an evidence-based review). Report
of the Quality Standards Subcommittee of the American Academy of Neurology. Neurology 2006;66:968-975.
Question 442: Clinical Adult - Cerebrovascular Disease
Discussion:
This patient has suffered a transient ischemic event most likely related to either intracranial or extracranial
atheromatous disease. There is no evidence to suggest a cardiac embolic source. The most appropriate therapy at
this time is an antiplatelet agent. There is no advantage of clopidogrel over aspirin. Aspirin is less expensive.
There is no role in this case for either anticoagulation or tissue plasminogen activase (tPA).
References:
Noseworthy J. Neurological therapeutics principles and practice. 2nd ed. London: Informa Healthcare, 2006.
Clinical Pediatrics
Question 8: Clinical Pediatrics - Headache
Discussion:
The most important determinant of the differential between pseudotumor cerebri and chronic daily headache is
the presence of papilledema, which mandates additional testing, including measurement of CSF pressure.
References:
Silberstein SD. Wolff's headache. In: Silberstein SD, Lipton RB, Dalessio DJ, editors. Wolff's headache and
other head pain. 7th ed. New York: Oxford University Press, 2001.
Question 13: Clinical Pediatrics - Heritable Metabolic or Degenerative
Discussion:
Maple syrup urine disease (MSUD) is due to a congenital inability to decarboxylate leucine, isoleucine, and
valine. There is an associated severe metabolic acidosis and hypoglycemia. Infants with MSUD develop poor
feeding and lethargy progressing to coma during the first week or two of life; examination reveals fluctuating
hypertonicity and opisthotonus. Blood pH and CO2 are typically normal in the aminoacidopathies (such as
phenylketonuria and homocystinuria), galactosemia, and urea cycle defects. Plasma ammonia is elevated in the
urea cycle disorders.
References:
Swaiman KF, Ashwal S, Ferriero DM. Pediatric neurology: principles and practice. 4th ed. Philadephia: Mosby,
2006.
Question 22: Clinical Pediatrics - Learning/Language Disorders
Discussion:
35
2007 RITE Discussion & Reference Manual
Discussion:
This child has tuberous sclerosis complex, an autosomal dominant disorder associated with mutations in the
tuberin and merlin genes. Intellectual outcome is best correlated with seizure control.
References:
Gomez MR, Sampson J, Whittemore V, editors. Tuberous sclerosis complex. 3rd ed. New York: Oxford
University Press, 1999.
Curatolo P, Verdecchia M, Bombardieri R. Vigabatrin for tuberous sclerosis complex. Brain Dev
2001;23(7):649-653.
Question 43: Clinical Pediatrics - Headache
Discussion:
Migraines are more likely to be frontal than unilateral in children. Photophobia and other criteria can be inferred
by the parents' observations, such as pulling curtains. Ibuprofen has been demonstrated efficacious in children;
to date oral triptans have not.
References:
AAN Practice parameter: Pharmacological treatment of migraine headaches in children and adolescents.
Neurology 2004;63;2215-2224.
Question 44: Clinical Pediatrics - Learning/Language Disorders
Discussion:
Asperger syndrome is classified with the autistic spectrum disorders (pervasive developmental disorders). It
shares the features of autistic disorder except that language is spared. Children with Asperger syndrome are often
referred to as "little professors" owing to their extensive knowledge of very restricted fields of interest.
References:
American Psychiatric Association. Diagnostic and statistical manual of mental disorders (DSM IV-TR). 4th ed.
Washington DC: American Psychiatric Association, 2000.
Question 59: Clinical Pediatrics - Disturbances of Consciousness
Discussion:
The Presidents Commission recommeded 100% oxygen ventilation for 10 minutes, followed by passive 100%
oxygen for a period long enough to achive a PCO2 of 60. The other tests have high incidence of false-negative
results.
References:
Swaiman KF, Ashwal S, Ferriero DM. Pediatric neurology: principles and practice. 4th ed. Philadephia: Mosby,
2006.
Question 66: Clinical Pediatrics - Motor Neuron/Neuromuscular
Discussion:
Examination of the affected infant is extremely nonspecific. Infants with congenital myotonic dystrophy present
with a variety of findings in the newborn period, including hypotonia, contractures, poor suck, respiratory
distress, cardiac arrhythmias, but not characteristic facies or myotonia seen later. Examination of the mother is
much more likely to reveal the typical facial features and mild weakness of eye closure and neck flexors of
myotonic dystrophy. DNA testing is definitive, but clinical diagnosis is usually possible with examination of
mother, not infant.
References:
36
2007 RITE Discussion & Reference Manual
Swaiman KF, Ashwal S, Ferriero DM. Pediatric neurology: principles and practice. 4th ed. Philadephia: Mosby,
2006.
Question 81: Clinical Pediatrics - Neurosurgery
Discussion:
At birth the median head circumference of a normal term infant is 35 cm. A head circumference of 45 cm is
greater than the 98th percentile. In the absence of other cerebral malformations, congenital aqueductal stenosis is
the most common cause of noncommunicating hydrocephalus. A familial X-linked form accounts for 2% of
these. Increased intracranial pressure in the context of hypoxic-ischemic encephalopathy does not cause
increased head circumference at birth. Klippel-Feil syndrome (fusion of the cervical vertebra) may be associated
with Chiari malformation but is a much less common cause of hydrocephalus. Choroid plexus papillomas are
typically located in one lateral ventricle and become symptomatic after the perinatal period, usually by
obstructing ventricular outflow. Lissencephaly is typically associated with microcephaly.
References:
Bradley WG, Daroff RB, Fenichel GM, et al, editors. Neurology in clinical practice. 3rd ed. Boston:
Butterworth-Heinemann, 2000.
Question 90: Clinical Pediatrics - Behavioral/Psychiatric
Discussion:
Tourette syndrome is a disorder including motor tics over a year and vocal tics and is often associated with
attention deficit hyperactivity disorder and/or obsessive-compulsive disorder. Coprolalia is rare and typically
occurs in adolescence or later.
References:
American Psychiatric Association. Diagnostic and statistical manual of mental disorders (DSM IV-TR). 4th ed.
Washington DC: American Psychiatric Association, 2000.
Swaiman KF, Ashwal S, Ferriero DM. Pediatric neurology: principles and practice. 4th ed. Philadephia: Mosby,
2006.
Question 120: Clinical Pediatrics - Tumors
Discussion:
Optic gliomas are the most common CNS tumor for children with neurofibromatosis type 1, but other cranial
nerves can be affected by neurofibromas or schwannomas. Astrocytomas, meningiomas, medulloblastomas,
ependymomas, and hamartomas also occur with increased frequency in patients with neurofibromatosis type 1.
References:
Berg BO, editor. Principles of child neurology. New York: McGraw-Hill, 1996.
Question 126: Clinical Pediatrics - Motor Neuron/Neuromuscular
Discussion:
The patient has the typical history and findings of subacute or acute onset of descending paralysis involving
cranial nerves, neck, and shoulder girdle characteristic of infant botulism. Sluggish or fatiguable pupils are a
characteristic finding of botulism. Nonketotic hyperglycinemia would not usually have a descending paralysis,
nor would pupils be sluggish. Spinal muscular atrophy would likely have proximal weakness but rarely has
ptosis and never has pupillary involvement. Myotonic discharges are not usually seen in infants, even with
myotonic dystrophy. The clinical picture is not one of myotonic dystrophy. The child described has findings
suggesting peripheral weakness, not central.
References:
37
2007 RITE Discussion & Reference Manual
Thompson JA, Filloux FM, Van Orman CB, et al. Infant botulism in the age of botulism immune globulin.
Neurology 2005; 64:2029-2032.
Question 127: Clinical Pediatrics - Motor Neuron/Neuromuscular
Discussion:
The patient has the typical history and findings of subacute or acute onset of descending paralysis involving
cranial nerves, neck, and shoulder girdle characteristic of infant botulism. Sluggish or fatiguable pupils are a
characteristic finding of botulism. Human botulism immunoglobulin has been shown to significantly shorten the
course of the condition. Broad-spectrum antibiotics may acutely worsen weakness.
References:
Thompson JA, Filloux FM, Van Orman CB, et al. Infant botulism in the age of botulism immune globulin.
Neurology 2005; 64:2029-2032.
Question 138: Clinical Pediatrics - Developmental Disorders
Discussion:
Lack of appropriate representative play in a young child presenting with developmental delay strongly suggests a
diagnosis of autism.
References:
Swaiman KF, Ashwal S, Ferriero DM. Pediatric neurology: principles and practice. 4th ed. Philadephia: Mosby,
2006.
Question 142: Clinical Pediatrics - Behavioral/Psychiatric
Discussion:
Masturbation is common in infancy and early childhood and often not recognized. It does not require
intervention or futher testing. Videotapes of episodes by parents are often most helpful in establishing the
diagnosis.
References:
Yang ML, Fullwood E, Goldstein J, Mink J. Masturbation in infancy and early childhood presenting as a
movement disorder: 12 cases and a review of the literature. Pediatrics 2005;116:1427-1432.
Question 175: Clinical Pediatrics - Epilepsy
Discussion:
Benign focal epilepsy of childhood with centrotemporal sharp waves presents from 2 to 13 years of age. About
20% of patients only have one seizure and in two thirds of the cases seizures are infrequent. Children with this
disorder are neurologically normal and the majority of patients outgrow the disorder. Carbemazepine is the
treatment of choice if treatment is necessary.
References:
Wyllie, E. The treatment of epilepsy, principles and practice. 3rd ed. Philadelphia: Lippincott, Williams &
Wilkins, 2001.
Question 177: Clinical Pediatrics - Heritable Metabolic or Degenerative
Discussion:
Metachromatic leukodystrophy has striking peripheral neuropathy. Rett's syndrome has microcephaly.
Alexander, Canavan, and Tay-Sachs diseases have megaloencephaly and hyperreflexia.
References:
38
2007 RITE Discussion & Reference Manual
Swaiman KF, Ashwal S, Ferriero DM. Pediatric neurology: principles and practice. 4th ed. Philadephia: Mosby,
2006.
Question 183: Clinical Pediatrics - Critical Care/Trauma
Discussion:
Appropriate therapies for refractory status epilepticus include phenobarbital, pentobarbital, or benzodiazepine
drips. The correct dose for phenobarbital loading is 20mg/kg. If a second dose of fosphenytoin is given after a
20mg/kg load, it is generally limited to an additional 5 mg/kg. Midazolam and pentobarbital doses are incorrect
in the options. They should be: midazolam 0.2 mg/kg slow IV bolus followed by 0.75 micrograms/kg/min to 10
micrograms/kg/min IV drip; pentobarbital 5 mg/kg to 15 mg/kg IV bolus over 1 hour followed by 0.5 mg/kg/h
to 3.0 mg/kg/h IV drip. Ventilatory assistance is usually necessary, as are vasopressors. Continuous EEG
monitoring is ideal.
References:
Wyllie, E. The treatment of epilepsy, principles and practice. 3rd ed. Philadelphia: Lippincott, Williams &
Wilkins, 2001.
Question 184: Clinical Pediatrics - Movement Disorders
Discussion:
The basal globus pallidus lesions produce an "eye of the tiger" on MRI, seen with pantothenate kinase
deficiency (PKAN). The globus pallidus lesions are from iron deposition. Huntington's disease produces caudate
atrophy, Niemann-Pick disease may have dystonia and rigidity but does not produce isolated globus pallidus
lesions. Idiopathic torsion dystonia and Sandifer's syndrome have normal MRI. PKAN is no longer called
Hallervorden-Spatz disease because of Hallervorden's role in Nazi genocide.
References:
Swaiman KF, Ashwal S, Ferriero DM. Pediatric neurology: principles and practice. 4th ed. Philadephia: Mosby,
2006.
Question 192: Clinical Pediatrics - Headache
Discussion:
The majority of children with recurrent headaches with complete clearing between episodes do not need
neuroimaging. The following would mandate neuroimaging in a child presenting with headaches: auras lasting
more than an hour, persistent neurological findings, abnormal neurological examination between headaches,
predominantly occipital headaches, loss of vision at headache peak, or recent significant change in headache
pattern.
References:
Silberstein SD. Wolff's headache. In: Silberstein SD, Lipton RB, Dalessio DJ, editors. Wolff's headache and
other head pain. 7th ed. New York: Oxford University Press, 2001.
Question 201: Clinical Pediatrics - Vascular Disorders
Discussion:
Moyamoya syndrome can be primary genetic or secondary to various conditions, including radiation therapy
involving the circle of Willis, sickle cell disease, neurofibromatosis, and Down syndrome. Primary moyamoya
syndrome is more frequent in persons of Asian descent.
References:
Swaiman KF, Ashwal S, Ferriero DM. Pediatric neurology: principles and practice. 4th ed. Philadephia: Mosby,
2006.
39
2007 RITE Discussion & Reference Manual
Questions 214 - 218: Clinical Pediatrics - Infectious Disease
Discussion:
The most common cause of early neonatal sepsis and meningitis is group B beta-hemolytic streptococcus.
Tuberculous meningitis presents insidiously, then in later stages causes severe basilar meningitis, strokes due to
involvement of basilar vessels, and hydrocephalus. It is poorly reversible at this stage. Young adults living in
dormitories or barracks are at risk for meningococcal disease. A newborn with microcephaly, organ
involvement, and subependymal calcifications most likely has congenital Cytomegalovirus, although other
congenital infections may occasionally mimic this. A child from a hilly region in the Upper Midwest presenting
during the late summer with fever, headache, stiff neck, and focal seizures has La Crosse encephalitis.
References:
Swaiman KF, Ashwal S, Ferriero DM. Pediatric neurology: principles and practice. 4th ed. Philadephia: Mosby,
2006.
Question 233: Clinical Pediatrics - Developmental Disorders
Discussion:
Aicardi syndrome is an X-linked dominant syndrome, presumed lethal in males, with agenesis of corpus
callosum, retinal lacunae, development disabilities, and frequently associated with infantile spasms.
References:
Swaiman KF, Ashwal S, Ferriero DM. Pediatric neurology: principles and practice. 4th ed. Philadephia: Mosby,
2006.
Question 240: Clinical Pediatrics - Headache
Discussion:
The lesion is typical of the acute inflammatory phase of neurocysticercosis. In the United States, most children
with cysticercosis live in households with recent immigrants, but many are lifelong US residents. Most
neurocysticercosis lesions will regress without treatment. Anticonvulsants are needed for several months, but
only about 25% ultimately have chronic epilepsy requiring ongoing treatment. CT is generally adequate for
diagnosis. Nearly all single lesion neurocysticercosis resolves spontaneously in 3 to 6 months.
References:
Ong S, Talan DA, Moran GJ, et al. Neurocysticercosis in radiologically imaged seizure patients in U.S.
emergency departments. Emerg Infect Dis 2002;8:608-613.
Question 323: Clinical Pediatrics - Epilepsy
Discussion:
The seizure pattern described is one of benign rolandic epilepsy, for which the EEG signature is
central-temporal spikes, particularly in sleep, often bilateral independent.
References:
Swaiman KF, Ashwal S, Ferriero DM. Pediatric neurology: principles and practice. 4th ed. Philadephia: Mosby,
2006.
Question 345: Clinical Pediatrics - Neonatal
Discussion:
MRI and, when available, diffusion-weighted imaging and magnetic resonance spectroscopy gives the most
information on outcome of a newborn with persistent low Apgar scores and neurological complications.
40
2007 RITE Discussion & Reference Manual
References:
Ment LR, Bada HS, Barnes P, et al. Practice parameter: Neuroimaging of the neonate. Neurology
2002;58:1726-1738.
Question 346: Clinical Pediatrics - Neonatal
Discussion:
The drop in hematocrit may indicate an intracranial bleed. Compared to MRI, a CT scan without contrast is
safest and will detect most bleeds. EEG is not indicated, and contrast will add little and may pose a risk because
of age and systemic illness.
References:
Ment LR, Bada HS, Barnes P, et al. Practice parameter: Neuroimaging of the neonate. Neurology
2002;58:1726-1738.
Question 350: Clinical Pediatrics - Movement Disorders
Discussion:
Clusters of head drops with flexor spasms of the trunk occuring around meal time in an otherwise healthy
7-month-old with a normal EEG suggest benign myoclonus of infancy. The normal EEG effectively rules out
infantile spasms. Benign occipital epilepsy of childhood, Lafora body disease, and Tourette syndrome occur in
older children.
References:
Bradley WG, Daroff RB, Fenichel GM, et al, editors. Neurology in clinical practice. 3rd ed. Boston:
Butterworth-Heinemann, 2000.
Question 366: Clinical Pediatrics - Inflammatory Disease
Discussion:
Fever is present in 90% of children with systemic lupus erythematosus, and about the same number have joint
involvement. It is more common in African Americans than whites or Hispanics. Seizures and cranial
neuropathies are common presenting features. Acute encephalopathy is less common and is life-threatening.
Sickle cell disease usually presents at a younger age. Lyme disease rarely causes seizures or confusion and has a
different rash. MELAS and moyamoya syndrome ususally present with isolated infartions or transient ischemic
attacks and do not have systemic symptoms.
References:
Bradley WG, Daroff RB, Fenichel GM, et al, editors. Neurology in clinical practice. 3rd ed. Boston:
Butterworth-Heinemann, 2000.
Question 382: Clinical Pediatrics - Spine
Discussion:
Hallmarks of Friedreich's ataxia are a combination of weakness, ataxia, absent tendon jerks, Babinski sign, and
abnormal position sense. Romberg sign is the best indication of loss of position sense, not cerebellar
functioning. Hammer toes, pes cavus deformities, and scoliosis are frequent findings.
References:
Swaiman KF, Ashwal S, Ferriero DM. Pediatric neurology: principles and practice. 4th ed. Philadephia: Mosby,
2006.
Question 383: Clinical Pediatrics - Neonatal
41
2007 RITE Discussion & Reference Manual
Discussion:
Neonatal seizures usually have a serious underlying reason. Treatable causes need to be recognized since delay
in treatment can aggrevate serious neurological comprimise. Group B Streptococcus is the most common cause
of bacterial meningitis/septicemia in the neonate and has about a 25% mortality rate; of the survivors, about
20% will have severe deficits. If seizures in the neonate are secondary to subarachnoid hemorrhage, there
usually is a good outcome. Benign epileptic syndromes and electrolyte disturbances (hypocalcemia or
hypomagnesemia) in the newborn can generally present by the end of the first week, but infants are not febrile.
References:
Volpe JJ. Neurology of the newborn. 4th ed. Philadelphia: WB Saunders, 2001.
Question 386: Clinical Pediatrics - Vascular Disorders
Discussion:
Cerebrovascular disease occurs in 25% of sickle cell disease. Eighty percent of events occur at or before age 15,
and most are caused by a progressive cerebral vasculopathy that can be partially arrested by chronic transfusion
therapy to keep hemoglobin S below 30%. Most events are thrombotic.
References:
Swaiman KF, Ashwal S, Ferriero DM. Pediatric neurology: principles and practice. 4th ed. Philadephia: Mosby,
2006.
Question 396: Clinical Pediatrics - Disturbances of Consciousness
Discussion:
Lead poisoning typically presents in young children living in substandard housing with peeling paint. However,
children living in environments where paint is being removed or scraped may be at risk, even in upper-income
environments. The other toxins, which are sometimes ingested by toddlers, do not present with increased
intracranial pressure.
References:
Nordin J, Rolnick S, Ehlinger E, et al. Lead levels in high-risk and low-risk young children in the
Minneapolis-St Paul metropolitan area. Pediatrics 1998;101:72-76.
Question 409: Clinical Pediatrics - Developmental Disorders
Discussion:
Cerebral palsy is nonprogressive disease, and the study with the highest yield is MRI. The proper diagnosis of
cerebral palsy excludes progressive or degenerative disorders. If the MRI is normal or nondiagnostic, further
tests could be ordered. An EEG should be obtained only with a positive history of seizurelike events.
References:
Ashwal S, Russman B, Blasco P, et al. Practice Parameter: Diagnostic assessment of the child with cerebral
palsy: Report of the Quality Standards Subcommittee of the American Academy of Neurology. Neurology
2004;62;851-863.
Question 414: Clinical Pediatrics - Headache
Discussion:
Caffeine withdrawal is a common cause of acute severe headache in migraineurs. Vitamins D or A can cause
pseudotumor cerebri, but onset is not abrupt. B vitamins are not associated with exacerbation of migraine.
References:
Silberstein SD. Wolff's headache. In: Silberstein SD, Lipton RB, Dalessio DJ, editors. Wolff's headache and
42
2007 RITE Discussion & Reference Manual
other head pain. 7th ed. New York: Oxford University Press, 2001.
Ethics, Law and Humantities Committee, American Academy of Neurology. Humanistic dimension of
professionalism in the practice of neurology. Neurology 2001;56:1261-1263.
Question 415: Clinical Pediatrics - Headache
Discussion:
This is a classic presentation of benign paroxysmal vertigo, a syndrome that occurs in children in the first 5
years of life and is a frequent precursor of migraine. Investigations (MRI, EEG) are normal. The episodes
usually resolve by the end of the first decade.
References:
Bradley WG, Daroff RB, Fenichel GM, et al, editors. Neurology in clinical practice. 3rd ed. Boston:
Butterworth-Heinemann, 2000.
Question 417: Clinical Pediatrics - Epilepsy
Discussion:
The 12-year-old boy has a characteristic history for juvenile myoclonic epilepsy with early-morning myoclonic
and generalized tonic-clonic seizures, beginning in the second decade of life. Absence seizures may also occur.
The EEG reveals generalized, fast (4 Hz to 6 Hz) spike-and-wave complexes. Valproic acid is the treatment of
choice and recurrence of seizures is likely to occur if treatment is stopped.
References:
Menkes JH, Sarnat HB. Textbook of child neurology. 6th ed. Philadelphia: Lippincott, Williams & Wilkins,
2000.
Question 418: Clinical Pediatrics - Epilepsy
Discussion:
This 12-year-old boy has a characteristic history for juvenile myoclonic epilepsy with early-morning myoclonic
and generalized tonic clonic seizures, beginning in the second decade of life. Absence seizures may also occur.
The EEG reveals generalized, fast (4 Hz to 6 Hz) spike-and-wave complexes. Valproic acid is the treatment of
choice and recurrence of seizures is likely to occur if treatment is stopped.
References:
Menkes JH, Sarnat HB. Textbook of child neurology. 6th ed. Philadelphia: Lippincott, Williams & Wilkins,
2000.
Question 429: Clinical Pediatrics - Disturbances of Consciousness
Discussion:
Breath-holding spells occur in infants, usually triggered by sudden pain or frustration. Consciousness is lost
prior to variable brief clonic jerking. These are common and occur in up to 5% of infants.
References:
Stephenson JPB. Fits and Faints. Philadelphia: JB Lippincott Co., 1990.
Question 437: Clinical Pediatrics - Epilepsy
Discussion:
This is a typical presentation for infantile spasms. Adrenocorticotropic hormone (ACTH) is an accepted
treatment. Colic or reflux are often mistaken for infantile spasms, although flexor spasms are not at all typical
for reflux. Phenobarbital has not been shown effective in treatment and may worsen the disorder.
43
2007 RITE Discussion & Reference Manual
References:
Wyllie, E. The treatment of epilepsy, principles and practice. 3rd ed. Philadelphia: Lippincott, Williams &
Wilkins, 2001.
Practice parameter: Medical treatment of infantile spasms: Report of the American Academy of Neurology and
the Child Neurology Society. Neurology 2004;62:1668-1681.
Question 439: Clinical Pediatrics - Motor Neuron/Neuromuscular
Discussion:
The congenital myasthenic syndromes are not related to an immune process but are caused by genetic defects
affecting the neuromuscular junction. These include defects in acetylcholine synthesis and packaging (familial
infantile myasthenia gravis), end-plate deficiency of acetylcholinesterase, acetylcholine receptor deficiency, and
the slow channel syndrome.
References:
Dubowitz V. Muscle disorders in childhood. Philadelphia: WB Saunders, 1995.
Question 443: Clinical Pediatrics - Disturbances of Consciousness
Discussion:
The child's presentation (opsoclonus, myoclonus, ataxia syndrome, alternatively called "OMS", "dancing eyes
syndrome" or Kinsbourne's Syndrome) is most often a paraneoplastic manifestation of occult neuroblastoma in
this age group. The most common initial misdiagnosis is acute cerebellar ataxia of childhood, a benign
condition. The most helpful test in determining etiology of suspected OMS is CT of the chest, abdomen, and
pelvis using oral contrast and fine cut images.
References:
Mitchell WG, Davalos-Gonzales Y, Brumm V, et al. Opsoclonus-ataxia caused by childhood neuroblastoma:
Developmental and neurological sequelae. Pediatrics, 2002;109:86-98.
Contemporary Issues
Question 6: Contemporary Issues - Ethics
Discussion:
Physicians and other health care professionals must report suspected cases of child abuse. It is not the
physician's obligation to prove the abuse, and testing to rule out other etiologies should not postpone reporting.
Glutaric aciduria, Menkes disease, and hemophilia may present with intracranial or subdural bleeding but are
less likely in this circumstance. Retinal hemorrhages are unusual in these conditions, however. Physicians are
protected from liability for reporting in good faith, even if the allegation is ultimately not supported.
References:
Swaiman KF, Ashwal S, Ferriero DM. Pediatric neurology: principles and practice. 4th ed. Philadephia: Mosby,
2006.
Question 37: Contemporary Issues - Ethics
Discussion:
Requests to lie to patients about their diagnoses arise from time to time. Some are based on the patient and
family's cultural traditions, and some are based on fear of consequences. In this case, the son's request is based
on fear of depression and is not a sufficient justification to lie to the patient. Telling the patient her son asked that
her diagnosis should be withheld inappropriately undermines trust between the physician and the son, and
between the son and his mother. To honor the son's request while later telling the patient her diagnosis by
44
2007 RITE Discussion & Reference Manual
between the son and his mother. To honor the son's request while later telling the patient her diagnosis by
telephone consitutes lying to the son and is inappropriate. The AAN Code of Professional Conduct recommends
telling the truth to patients who have decision-making capacity. AAN guidelines on humanistic dimension of
professionalism in the practice of neurology state that neurologists have a role in helping patients and families
overcome their fears of neurological disorders. Truth-telling allows patients with probable Alzheimer's disease
to seek appropriate treatment and to make arrangements for their treatment preferences (eg, end-of-life care
planning) and to select their surrogate decision maker. Not all patients want to know everything about their
diagnosis and treatment, and some may wish to have their family handle this information and decisions for
them. Asking the patient in advance how much information she wishes to receive respects her autonomy by
allowing her to make a choice.
References:
Donatelli LA, Geocadin RG, Williams MA. Ethical issues in critical care and cardiac arrest: clinical research,
brain death, and organ donation. Semin Neurol 2006;26:452–460.
Ethics, Law and Humantities Committee, American Academy of Neurology. Humanistic dimension of
professionalism in the practice of neurology. Neurology 2001;56:1261-1263.
Question 48: Contemporary Issues - HIPAA
Discussion:
HIPAA regulations regarding patients' protected health information (PHI) are important to safeguard patients'
privacy and confidentiality. HIPAA regulations are not intended to interfere with the legitimate exchange of PHI
between health professionals caring for patients, as delay of information sharing could result in patient harm.
The United States Department of Health and Human Services Office of Civil Rights states that "(physicians)
may disclose protected health information for the treatment activities of any health care provider...." For
example, a primary care provider may send a copy of an individual’s medical record to a specialist who needs
the information to treat the individual, or vice versa. In addition, the circumstance need not be a medical
emergency, nor is there a requirement that the request be made in writing as long as copies of the records are
being sent for the purpose of patient care.
References:
United States Department of Health and Human Services Office for Civil Rights--HIPAA. Standards for Privacy
of Individually Identifiable Health Information [45 CFR Parts 160 and 164].
http://www.hhs.gov/ocr/hipaa/guidelines/guidanceallsections.pdf.
Question 79: Contemporary Issues - Driving
Discussion:
Driving is mildly impaired in drivers with probable Alzheimer's disease (AD) at a severity of Clinical Dementia
Rating (CDR) 0.5. Patients and their families should be told that patients with possible AD with a severity of
CDR 0.5 pose a significant traffic safety problem when compared to other elderly drivers. Referral of the patient
for a driving performance evaluation by a qualified examiner should be considered. Drivers with AD at a
severity of CDR 1 pose a significant traffic safety problem both from crashes and from driving performance
measurements. Patients and their families should be told that patients with AD with a severity of CDR 1 or
greater have a substantially increased accident rate and driving performance errors, and, therefore,
discontinuation of driving should be strongly considered.
References:
Dubinsky RM, Stein AC, Lyons K. Practice parameter: risk of driving and Alzheimer's disease (an
evidence-based review): report of the Quality Standards Subcommittee of the American Academy of Neurology.
Neurology 2000;54(12):2205-2211.
Question 83: Contemporary Issues - Ethics
Discussion:
45
2007 RITE Discussion & Reference Manual
In the United States, the laws that govern withdrawal of artificial hydration and nutrition (AHN) are established
by the individual states, territories, and District of Columbia. There is significant variation in the laws regarding
the withdrawal of AHN in the absence of an advance directive, with state laws categorized as permissive and
restrictive. In more than two thirds of the states, it is settled that surrogate decision makers are allowed to decide
to withdraw AHN without review and approval by a judge if the patient is in PVS and there is no AD. On the
other hand, in the presence of an advance directive (AD), state laws are consistent in permitting withdrawal of
AHN if it is directed in the AD. There are no laws that specify the nature of diagnostic testing that must be
performed to confirm the diagnosis of PVS.
References:
North Dakota Century Code. 23-06.5 Health Care Directives. http://www.legis.nd.gov/cencode/t23c065.pdf.
Larriviere D, Bonnie RJ. Terminating artificial nutrition and hydration in persistent vegetative state patients:
current and proposed state laws. Neurology 2006;66;1624-1628.
Question 152: Contemporary Issues - Ethics
Discussion:
Huntington's disease is an autosomal dominant trait with complete penetrance, and offspring from affected
patients have a 50% chance of developing the disease. Most patients become symptomatic in the third or fourth
decade. Presymptomatic testing for Huntington's disease should be performed only voluntarily at the request of
an at-risk patient. Genetic testing should be accompanied by pretest and posttest counseling. Guidelines are
available and emphasize voluntariness, confidentiality, safety, absence of coercion, and availability of
counseling.
References:
Case adapted from American Academy of Neurology Ethics, Law and Humanities Committee. Ethical
Dimensions of Neurologic Practice: A case-based curriculum for neurology residents. March 2000.
Went L. Ethical issues policy statement on Huntington's disease molecular genetics predictive test. International
Huntington Association. World Federation of Neurology. J Med Genet 1990:27;34-38.
Question 171: Contemporary Issues - End-of-Life/Palliative Care
Discussion:
Organ donation most commonly occurs in the context of severe brain injury and brain death. Public awareness
and support of organ donation and transplantation has increased in the last decade. As a result, families often
recognize that the patient's circumstances may be such that organ donation is a possibility. The family's request is
not illegal, nor does it indicate they are inappropriate decision makers for the patient because the patient has
previously indicated the desire to be an organ donor. While it is usually considered improper for physicians to
raise the topic of organ donation with a family before brain death has been determined, it is proper to respond to
family requests about organ donation whenever they are made. Organ donation can occur even for "coroner's
cases" as the coroner can be in the operating room at the time the organs are removed for transplantation.
Because patients must be evaluated for medical suitability for organ donation, and because conversations with
families about organ donation require considerable skill, physicians should work closely with representatives of
organ procurement organizations when organ donation is considered a possibility.
References:
Williams MA, Lipsett PA, Rushton CH, et al. The physician's role in discussing organ donation with families.
Crit Care Med 2003;31(5):1568-1573.
Question 188: Contemporary Issues - Core Competencies
Discussion:
Medical errors occur frequently and are currently thought to result from systems errors. The AAN Patient Safety
Committee web page (http://www.aan.com/professionals/patient/patient_saf.cfm) offers patient safety tips as
well as a link to the 2006 Patient Safety Colloquium syllabus, which addresses many of the issues in this test
46
2007 RITE Discussion & Reference Manual
well as a link to the 2006 Patient Safety Colloquium syllabus, which addresses many of the issues in this test
question (http://www.aan.com/professionals/patient/pdfs/colloquium_syllabi_06.pdf). The AAN, the American
Medical Association, and other societies urge that errors should be disclosed to patients under most
circumstances and should include an apology and a promise to investigate the causes of the error. Although in
this case the patient was not harmed, the additional laboratory tests and extra hospital day are serious
consequences of the error that justify the disclosure. The purpose of disclosure is not to place blame on
individuals, but rather to inform the patient of the events. Errors should be disclosed by persons with the
greatest experience, and those with responsiility for the patient's care--in this case, the attending physician. It is
inappropriate to ask a resident to make an error disclosure alone, although it would be appropriate for the
resident to be present with the attending physician when the error is disclosed. It is inappropriate to expect any
health care professional will never make an error; instead, it is important to acknowledge that we all committ
errors and have an obligation to investigate errors to prevent future recurrence. Failure to disclose the error for
fear of malpractice lawsuit is inappropriate, as it constitutes concealment.
References:
Institute for Safe Medical Practices. List of error-prone abbreviations, symbols and dose designations.
http://www.ismp.org/tools/errorproneabbreviations.pdf.
Patient Safety Colloquium. http://www.aan.com/professionals/patient/pdfs/colloquium_syllabi_06.pdf.
Question 191: Contemporary Issues - End-of-Life/Palliative Care
Discussion:
Although advances in understanding the pathophysiology of ALS have stimulated the development of new drug
therapies, the mainstay of treatment for ALS patients remains symptomatic management. The AAN Practice
Parameter on the care of the patient with ALS recommends frequent discussions of the goals of care because
decision making may be subject to change as the disease becomes more severe. Between 40% and 73% of
patients experience pain in later stages of ALS, and AAN guidelines recommend use of opioids when
non-narcotic treatment fails. While the use of carefully titrated benzodiazepines to alleviate anxiety and
respiratory distress is indicated, the use of neuromuscular blockade is not supported by AAN guidelines. Few
patients on long-term invasive ventilation regret their choice, but because patient satisfaction is higher with
noninvasive than invasive ventilation, noninvasive ventilation should be considered first.
References:
Quality Standards Subcommittee of the American Academy of Neurology. Practice parameter: the care of the
patient with amyotrophic lateral sclerosis. Neurology 1999; 52:1311-1323.
http://www.neurology.org/cgi/reprint/52/7/1311.pdf
Question 332: Contemporary Issues - Driving
Discussion:
Determining the ability to drive following stroke is a difficult task. Numerous studies have been performed
suggesting the most reliable determinant of driving ability is the roadside test. The simulated driving test and the
roadside test are more predictive than neuropsychological measures and neurologist opinion.
References:
Akinwuntan AE, Feys H, De Weerdt W, et al. Prediction of driving after stroke: a prospective study.
Neurorehabil Neural Repair 2006;20:417-423.
Akinwuntan AE, De Weerdt W, Feys H, et al. The validity of a road test after stroke. Arch Phys Med Rehabil
2005;86:421-426.
Question 359: Contemporary Issues - Core Competencies
Discussion:
The American Medical Association has recently published guidelines regarding the ethical use of
industry-sponsored funds for social gatherings and other offerings provided by pharmaceutical companies to
47
2007 RITE Discussion & Reference Manual
industry-sponsored funds for social gatherings and other offerings provided by pharmaceutical companies to
physicians in the interest of endorsing products. Pharmaceutical companies are restricted from providing strictly
meals or entertainment to physicians or their families. It is appropriate to accept educational grants from
pharmaceutical companies if the source is disclosed, the educational content and speakers are selected by the
program director, and the presentation is free from bias.
References:
Accreditation Council for Continuing Medical Education. The Standards for Commercial Support.
http://www.accme.org/
Question 372: Contemporary Issues - Business
Discussion:
When choosing an ICD-9-CM diagnosis code for a diagnostic test, the code most consistent with the test result
or confirmed diagnosis should be used. A principle rule for choosing an ICD-9-CM code is to code to the highest
level of specificity. Based on the history and clinical findings, the most specific choice is carpal tunnel
syndrome. Codes that describe symptoms and signs as opposed to diagnoses are acceptable for reporting
purposes when a related definitive diagnosis has not been established or confirmed. V-codes are considered by
Medicare as "Supplementary Classification of Factors Influencing Health Status and Contact With Health
Services” and are not used for diagnoses.
References:
CMS. ICD-9-CM Official Guidelines for Coding and Reporting. Available online at:
http://www.cdc.gov/nchs/datawh/ftpserv/ftpicd9/icdguide05.pdf.
Question 408: Contemporary Issues - Core Competencies
Discussion:
In the NICU, implied consent to medical treatment is suitable in emergency circumstances, as patients would
otherwise be harmed by the delay needed to obtain informed consent from a surrogate. However, the concept of
implied consent for emergency treatment cannot be expanded to apply to other procedures or interventions
performed on the same patient in the absence of an emergency. The tracheostomy and percutaneous endoscopic
gastrostomy (PEG), while necessary, are elective and not emergency procedures, and informed consent from a
surrogate decision maker is necessary. The hospital attorney does not have the legal authority to approve
treatment decisions, and, as a representative of the hospital's interests, cannot appropriately advocate for the
patient's interests. Life-sustaining therapies cannot be withdrawn on the basis of the ICU teams' decision alone.
Therefore a surrogate decision maker must be found. The ethics consultation team can serve to represent the
patient's interests and decide whether the treatment recommended by the NICU team is ethically permissible. A
court of law would take the ethics consultation team's recommendations seriously when approving a guardian
for the patient. The NICU team may make the same recommendations to the ethics consultation team or
guardian that would be made to a family in a similar situation, including either withdrawal of life-sustaining
therapies or insertion of a tracheostomy and PEG.
References:
Donatelli LA, Geocadin RG, Williams MA. Ethical issues in critical care and cardiac arrest: clinical research,
brain death, and organ donation. Semin Neurol 2006;26:452–460.
Question 431: Contemporary Issues - End-of-Life/Palliative Care
Discussion:
In emergency circumstances where irreversible harm or death would occur by withholding requested treatment,
it is best to initiate the treatment and then seek consultation from an ethics committee. An ethics consultation can
rarely be done immediately as there would be insufficient time to gather the information necessary to offer a
recommendation. Refusals to treat on the basis of medical futility require a due process of evaluation of the
patient's interests, the family's wishes, and the medical facts of the case, including prognosis. Medical futility
cannot be determined unilaterally without ethics consultation. Anencephaly is not equivalent to brain death, as
48
2007 RITE Discussion & Reference Manual
there is function of the brain stem and there are respiratory efforts.
References:
Anomynous. In the matter of baby "K" No. 93-1899, No. 93-1923, No. 93-1924. United States Court of Appeals
16F.3d590; 1994 US app. (LEXIS 2215).
Question 434: Contemporary Issues - HIPAA
Discussion:
The HIPAA Privacy Rule at 45 CFR 164.510(b) specifically permits physicians to share information that is
directly relevant to the involvement of a spouse, family members, friends, or other persons identified by a
patient in the patient’s care or payment for health care. If the patient is present or is otherwise available prior to
the disclosure and has the capacity to make health care decisions, physicians may discuss this information with
the family and these other persons if the patient agrees or, when given the opportunity, does not object.
Physicians may also share relevant information with the family and these other persons if they can reasonably
infer, based on professional judgment, that the patient does not object. In this case, the neurologist does not have
sufficient information to infer whether the patient would allow this information to be shared with her husband,
and it is better to ask the patient's permission first. Information can be shared in medical emergencies; however,
weight gain is not an emergency. HIPAA regulations apply equally to specialists and primary care physicians,
and HIPAA regulations apply equally to verbal and written communication.
References:
United States Department of Health and Human Services. Health Information Privacy and Civil Rights
Questions & Answers; Disclosure to Family and Friends.
http://healthprivacy.answers.hhs.gov/cgi-bin/hipaa.cfg/php/enduser/ std_alp.php.
United States Department of Health and Human Services Office for Civil Rights--HIPAA. Standards for Privacy
of Individually Identifiable Health Information [45 CFR Parts 160 and 164].
http://www.hhs.gov/ocr/hipaa/guidelines/guidanceallsections.pdf.
Neuroimaging
Question 51: Neuroimaging - Stroke
Discussion:
Parenchymal hemorrhages can be detected on MRI imaging documented within 2 hours of onset with gradient
T2 (also called T2*) and EPI sequences due to magnetic susceptibility effects of deoxyhemoglobin, which is the
earliest step in the breakdown of oxyhemoglobin.
References:
Patel M, Edelman R, Warach S. Detection of Hyperacute Primary Intraparenchymal Hemorrhage by Magnetic
Resonance Imaging. Stroke 1996;27:2321-2324.
Question 230: Neuroimaging - CSF Circulation Disorders
Discussion:
The CT shown is consistent with normal pressure hydrocephalus. The clinical presentation associated with
normal pressure hydrocephalus consists of gait apraxia, executive function impairment, and urinary
incontinence. There is no evidence of any lesions to suggest the clinical presentations of aphasia, hemisensory
neglect, Parkinson's disease, or seizures.
References:
Osborne AG. Diagnostic Neuroradiology. St Louis: Mosby, 1994;154-168.
Question 231: Neuroimaging - Critical Care/Trauma
49
2007 RITE Discussion & Reference Manual
Discussion:
The scans show multiple small ring-enhancing lesions consistent with cerebral abscesses located at the
supratentorial gray-white junction. In addition, a posterior occipital enhancing lesion is noted. The appearance is
most consistent with septic emboli. By the imaging appearance alone, cerebral metastases are not ruled out. In
the current case, the patient had endocarditis secondary to intravenous drug abuse. Uncomplicated pyogenic
meningitis typically shows intense meningeal enhancement of the cerebral convexity, tentorium, and falx.
Multiple sclerosis may show lesions with a similar appearence, but they would not involve the cortex, as seen on
the sagittal image, and the enhancement ring would tend to open to the side of the gray matter, not the white
matter as seen in the axial image of this case.
References:
Greenberg JO, editor. Neuroimaging: a companion to Adams and Victor's principles of neurology. 2nd ed. New
York: McGraw-Hill, 1999.
Question 232: Neuroimaging - Infection
Discussion:
CT shows multiple parenchymal calcifications with mild atrophy; this is characteristic of a host of infections,
including toxoplasmosis, rubella, Cytomegalovirus, and herpes. Calcification seen in Sturge-Weber syndrome is
typically cortical and gyriform. Although lipomas may calcify, they are extraaxial, usually single, and are
hypodense on CT. The scan does not show malformations of the sulci or ventricles to suggest schizencephaly or
colpocephaly.
References:
Greenberg JO. Neuroimaging: a companion to Adams and Victor's principles of neurology. New York:
McGraw-Hill, 1999.
Question 235: Neuroimaging - Spine
Discussion:
MR is the imaging modality of choice for evaluation of spinal infections. The typical MRI appearance is loss of
distinction between the endplates, disks, and adjacent vertebral bodies on T1-weighted views and increased
T2-weighted signal. These areas show enhancement. The findings with high sensitivity for spinal infections
include evidence of paraspinal, or epidural, inflammatory tissues, contrast enhancement of the disc, and
hyperintensity on fluid-gradient T2-weighted views. Spinal infections rarely involve only one vertebral body
and adjacent disc or only the epidural space.
References:
Ledermann H, Schweitzer M, Morrison W, Carrino J. MR imaging findings in spinal infections: rules or myths?
Radiology 2003;228:506-514.
Question 237: Neuroimaging - Spine
Discussion:
A hyperintense signal is seen on both T1-weighted and T2-weighted images, suggesting that the lesion is a
lipoma. Neurosurgical and radiation therapy consultations are not necessary for the management of this lesion.
A lumbar puncture and blood cultures are not recommended. Lipomas are benign.
References:
Edelman R, Hesselink J, editors. Clinical magnetic resonance imaging. Philadelphia: WB Saunders, 1990.
Question 238: Neuroimaging - Brain Tumors
Discussion:
50
2007 RITE Discussion & Reference Manual
MR is effective in demonstrating a displaced optic chiasm because the optic chiasm is usually slightly more
intense than pituitary tumors.
References:
Daniels DL, Haughton VM, Czervionke LF. MR of the skull base. In: Bradley WG, Stark D. Magnetic
resonance imaging. St. Louis: Mosby, 1988.
Greenberg JO. Neuroimaging: a companion to Adams and Victor's principles of neurology. New York:
McGraw-Hill, 1999.
Question 241: Neuroimaging - Autoimmune Disorders (Non-MS)
Discussion:
Leptomeningeal enhancement, parenchymal enhancement, and, especially, pituitary stalk/hypothalamic
involvement in a patient with cranial nerve symptoms is characteristic of neurosarcoidosis. About 5% to 10% of
sarcoidosis patients develop CNS involvement. Lack of hemiatrophy and prominent (asymmetrical) choroid
plexus enhancement make the diagnosis of Sturge-Weber syndrome less likely. No gyral enhancement is
present. Low CSF pressure would not be limited to one hemisphere and would not have masslike and
leptomeningeal enhancement. The enhancing lesion is intraaxial and therefore is not a meningioma.
References:
Osborn AG. Diagnostic neuroradiology. St. Louis: CV Mosby, 1994.
Question 242: Neuroimaging - Dementia
Discussion:
The most striking change between the two studies is the pronounced decrease in glucose metabolism in the
frontal and anterior temporal regions. This pattern is characteristic of frontotemporal dementia. Alzheimer’s
disease tends to decrease metabolism in the parietotemporal association cortex, relatively spared in the scan
shown. Parkinson’s disease does not cause a decrease in metabolism. As in this case, in Huntington’s chorea
there is decreased metabolism in the head of the caudate nuclei, but the frontal and temporal lobes are not as
affected. Finally, diffuse Lewy body disease causes decreased metabolism in the occipital regions, relatively
spared in this scan.
References:
Masdeu JC, Zubieta JL, Arbizu J. Neuroimaging as a marker of the onset and progression of Alzheimer's
disease. J Neurol Sci 2005;236:55-64.
Kantarci K, Jack Jr C. Neuroimaging in Alzheimer disease: an evidence-based review. Neuroimaging Clin N
Am 2003;13:197-209.
Coleman RE. Positron emission tomography diagnosis of Alzheimer's disease. Neuroimaging Clin N Am
2005;15:837-846.
Question 243: Neuroimaging - Brain Tumors
Discussion:
The most likely diagnosis is glioblastoma multiforme. On the FLAIR image, the lesion presents with mass
effect, too pronounced for an arteriovenous malformation, choroid plexus calcification, or an infarction. The
abnormal high-intensity region extends across the corpus callosum to the right hemisphere, a feature common in
malignant gliomas. The abnormal contrast enhancement suggests that the tumor has abnormal vessels with
marked permeability, another finding in glioblastoma. However, macroscopically dilated vessels, characteristic
of an arteriovenous malformation are not present. A choroid plexus papilloma is very unusual in this age group.
A toxoplasma abscess tends to show enhancement at the periphery and with a more regular appearance than in
the present case.
References:
51
2007 RITE Discussion & Reference Manual
Greenberg JO, editor. Neuroimaging: a companion to Adams and Victor's principles of neurology. 2nd ed. New
York: McGraw-Hill, 1999.
Question 244: Neuroimaging - Infection
Discussion:
Multiple basal ganglia high-signal foci are seen bilaterally. They are more numerous than normal
Virchow-Robin spaces. Toxoplasmosis resides also in basal ganglia, but then there is usually also mass effect.
Cryptococcus enters the intracranial space through the perivascular spaces, and the organisms are surrounded by
gelatinous material that is seen as high signal foci. This correlates with "soap-bubble" abscesses seen grossly.
The correct answer is cryptococcus infection.
References:
Osborn AG. Diagnostic neuroradiology. St. Louis: CV Mosby, 1994.
Question 245: Neuroimaging - Developmental/Neurogenetic Disorders
Discussion:
Sagittal T1-weighted image shows a Chiari II malformation with deformity of the tectum of the mesencephalon,
caudalization of the cerebellar vermis into the cervical spinal canal, and a deformity of the medial aspect of the
cerebral hemisphere, with an absent posterior corpus callosum. The cerebral aqueduct is not visualized, and the
ventricles do not appear enlarged, suggesting there is aqueductal stenosis and that a functioning shunt should be
present as was the case. There is a high degree of association between the cerebral changes of Chiari II
malformation and the presence of a lumbar myelomeningocele, thus the lumbar spine should be and was
dysraphic. Patients with Chiari II malformations do not have to be and are frequently not mentally retarded.
There is no evidence of contusion, subdural hematoma, or other forms of trauma on the section shown.
References:
McLone DG, Naidich TP. Developmental morphology of the subarachnoid space, brain vasculature, and
contiguous structures, and the cause of the Chiari II malformation. AJNR Am J Neuroradiol 1992;13(2):463-482.
Question 246: Neuroimaging - Epilepsy
Discussion:
The image shows heterotopic gray matter in the left frontal region. Deformity of the left frontal horn is seen with
ex-vacuo hydrocephalus. No mass effect is seen to suggest tumor. Rather, the lesion is destructive. Heterotopic
gray matter refers to cortical neurons in abnormal location. Heterotopia is caused by migrational arrest of the
affected neuroblasts. The cause of the incomplete migration is unknown. Three kinds of heterotopic gray matter
have been described: (1) nodular (periventricular), (2) laminar (in deep white matter), like in this patient, and (3)
band heterotopia (double cortex).
References:
Guerrini R, Filippi T. Neuronal migration disorders, genetics, and epileptogenesis. J Child Neurol
2005;20:287-299.
Question 247: Neuroimaging - Stroke
Discussion:
The large vascular malformation is an aneurysm of the vein of Galen. The sagittal time-of-flight image shows
particularly well the blood backing up into the posterior portion of the superior sagittal sinus, which is markedly
dilated. The fast flow in the malformation creates a flow void, visible as a dark structure in both the T2- and
T1-weighted images. The morphology of this malformation is characteristic and differentiates it from the other
answer options. Cavernous and venous angiomas are much smaller. A dural fistula has an important arterial
component, absent in this case. A hemangioma has the appearance of a tumor, without the large vessels visible
in this patient.
52
2007 RITE Discussion & Reference Manual
References:
Greenberg JO, editor. Neuroimaging: a companion to Adams and Victor's principles of neurology. 2nd ed. New
York: McGraw-Hill, 1999.
Question 250: Neuroimaging - Spine
Discussion:
Diastematomyelia is a rare form of spinal dysraphism. On MRI the cord appears split with a sagittal cleft of
varying extent. Fiber, cartilaginous, or osseous septums produce complete or incomplete divisions of the spinal
cord into hemicords. Patients may have neurological defects in the lower limbs such as gait disorders, sphincter
disturbance, or muscle atrophy.
References:
Sami H, Ross E, Walter M, Goli S. Split Spinal Cord (Diastematomyelia). Neurology 2003; 60:491.
Question 253: Neuroimaging - Stroke
Discussion:
The distribution of the changes on postendarterectomy MRI are consistent with an acute infarct in the
distribution of the anterior cerebral artery.
References:
Greenberg JO, editor. Neuroimaging: a companion to Adams and Victor's principles of neurology. 2nd ed. New
York: McGraw-Hill, 1999.
Question 254: Neuroimaging - Stroke
Discussion:
The MRI shows a subacute left middle cereberal infarct with a subacute hematoma. The hematoma has increased
T1 signal and decreased T2 signal. There is mild ipsilateral dilation of the left ventricle indicating an anteceding
chronic infarct. There is no mass effect or midline shift to indicate an acute left infarct. Chronic hemorrhage
would cause hypointense signal on both pulse sequences
References:
Barnett HJM. Stroke: pathophysiology, diagnosis, and management. New York: Churchill Livingstone, 1992.
Question 255: Neuroimaging - Epilepsy
Discussion:
The axial T2-weighted image shows a symmetrical brain with normal size ventricles. The frontal lobe cortex is
flat, and the white matter, which is hypointense, does not show the normal interdigitation into the gray matter in
this region. Thus, the cortex is thick, and the patient has pachygyria. Such an abnormality develops during the
period of neuronal migration, occurring during the second trimester.
References:
Zimmerman RA, Bilaniuk LT. Pediatric central nervous system. In: Stark DD, Bradley WG, editors. Magnetic
resonance imaging. St. Louis: CV Mosby, 1988.
Question 256: Neuroimaging - Developmental/Neurogenetic Disorders
Discussion:
MR images demonstrate dilatation of the cord around a central cystic cavity, consistent with
syringohydromyelia. Note also on the sagittal images that the inferior portion of the cerebellum has herniated
53
2007 RITE Discussion & Reference Manual
down into the upper cervical canal, diagnostic of a Chiari malformation. Hydromyelia is commonly seen in
association with both Chiari type I and Chiari type II malformation.
References:
Greenberg JO. Neuroimaging: a companion to Adams and Victor's principles of neurology. New York:
McGraw-Hill, 1999.
Question 258: Neuroimaging - Multiple Sclerosis
Discussion:
The MRI reveals far too much white matter disease for Alzheimer's disease. Metachromatic leukodystrophy
typically produces by this age more diffuse and confluent areas of demyelination than those shown. Progressive
multifocal leukoencephalopathy is less common, and large regions (rather than small plaques) of white matter
signal change are expected. Given the diffuse white matter lesions that are characteristic of multiple sclerosis
plaques, her age, and gender, multiple sclerosis is the most likely diagnosis. Vasculitis typically involves the
gray/white matter junction.
References:
Benedict RH, Weinstock-Guttman B, Fishman I, et al. Prediction of neuropsychological impairment in multiple
sclerosis: comparison of conventional magnetic resonance imaging measures of atrophy and lesion burden. Arch
Neurol 2004;61:226-230.
Question 259: Neuroimaging - Movement Disorders
Discussion:
The patient has cerebellar and pontine atrophy. This finding and the abnormal signal in the pons, in the form of a
cross, are found in olivopontocerebellar atrophy but not the other diagnostic possibilities. In the Chiari
malformation the posterior fossa is crowded, the opposite of what happens here. With an arachnoid cyst, the
subarachnoid space could be dilated but not the fourth ventricle. Finally, the inferior vermis is not atretic as it is
in the Dandy-Walker syndrome.
References:
Srivastava T, Singh S, Goyal V, et al. "Hot cross bun" sign in two patients with multiple system
atrophy-cerebellar. Neurology 2005;64:128.
Question 260: Neuroimaging - Critical Care/Trauma
Discussion:
Anoxia causes increased T2 signal in the globus pallidus, deep portions of the sulci, and superior cerebellum,
which are all anatomical regions that are susceptible to hypoxia. The necrotic globus pallidus enhances after
gadolinium administration.
References:
Greenberg JO. Neuroimaging: a companion to Adams and Victor's principles of neurology. New York:
McGraw-Hill, 1999.
Question 261: Neuroimaging - Spine
Discussion:
Although the MRI tends to rule out an astrocytoma and ependymoma because there is only mild mass effect, it
could still be an inflammatory lesion, giving rise to the increased signal on the T2 images. A heavily
T2-weighted sequence (T2-CISS) shows a cluster of vessels on the dorsal aspect of the cord compatible with
venous dilation due to arteriovenous shunting by a dural fistula. The selective angiogram confirms this
impression because the venous system is filled so rapidly by a brief arterial injection that both arterial and
venous vessels are seen on the same angio frame. This disorder corresponds to the myelopathy of
54
2007 RITE Discussion & Reference Manual
Foix-Alajouanine.
References:
Greenberg JO, editor. Neuroimaging: a companion to Adams and Victor's principles of neurology. 2nd ed. New
York: McGraw-Hill, 1999.
Koch C. Spinal dural arteriovenous fistula. Curr Opin Neurol 2006;19:69-75.
Question 262: Neuroimaging - Critical Care/Trauma
Discussion:
The patient was a diabetic with severe vascular disease. The morphology of the hemispheric lesions resembles
vascular disease more than either multiple sclerosis or a lymphoma. Of the three vessels mentioned in the
options, the right carotid does not show the flow void characteristic of circulating blood.
References:
Greenberg JO, editor. Neuroimaging: a companion to Adams and Victor's principles of neurology. 2nd ed. New
York: McGraw-Hill, 1999.
Question 265: Neuroimaging - Stroke
Discussion:
The FLAIR MRI image shows an area of high signal intensity in the distribution of the recurrent artery of
Heubner. It is consistent with an infarct in this tissue. The recurrent artery of Heubner arises from the A1
segment of the anterior cerebral artery and supplies the anteroinferior portion of the caudate nucleus, the
putamen, and the anterior limb of the internal capsule.
References:
Netter FH. Nervous system anatomy and physiology. New Jersey: CIBA Medical Education Division, 1986.
Question 268: Neuroimaging - Dementia
Discussion:
The patient had a subdural hematoma with mixed densities, some corresponding to a more recent bleeding. The
shape of the lesion and the low density of the deeper portion are uncharacteristic of an epidural hematoma, with
a lens shape and the high density of fresh blood. The lesion is outside the brain, compressing the cortex of the
right hemisphere. For this reason, infarction is not an option. A meningioma "en plaque" would be more
homogeneous and have higher density.
References:
Greenberg JO, editor. Neuroimaging: a companion to Adams and Victor's principles of neurology. 2nd ed. New
York: McGraw-Hill, 1999.
Question 270: Neuroimaging - Brain Tumors
Discussion:
This is a typical location for a colloid cyst. This usually causes low signal in the T2-weighted image; however,
the MRI signal can be variable.
References:
Dalessio DJ. Diagnosing the severe headache. Neurology 1994;44(Suppl 3):S6-S12.
Question 272: Neuroimaging - Brain Tumors
Discussion:
55
2007 RITE Discussion & Reference Manual
Pleomorphic xanthoastrocytoma typically produce a large cyst with superficially positioned mural nodule. Most
lesions arise within the temporal or parietal lobes. This lesion is surrounded with minimal hemosiderin rim but
no edema, and this excludes abscess and hematomas. The signal characteristics of cavernous hemangioma are
heterogeneous (popcorn). The mass effect is too little for a hematoma of this size. Porencephalic cysts do not
exert mass effect.
References:
Osborne A. Diagnostic imaging: brain. Philadelphia: Elsevier, 2004.
Question 274: Neuroimaging - Spine
Discussion:
Transverse myelitis affects predominantly the thoracic spinal cord and on imaging reflects as an increased signal
in the spinal cord on T2 images extending several segments. The spinal tap would show lympocytic
predominance and increased protien. There is no evidence of disc herniation, tumor, or spinal artery stroke on
the image. Gullain-Barre syndrome would not cause any increased signal in the spinal cord.
References:
Edelman R, Hesselink J, editors. Clinical magnetic resonance imaging. Philadelphia: WB Saunders, 1990.
Question 276: Neuroimaging - Spine
Discussion:
Axial image demonstrates a small, constricted spinal canal. This is mainly due to degenerative facet joint
disease. No disc herniation or synovial cyst is present. The nerve roots are coiled above the narrow area since
they cannot slide freely though the narrow part. Distal nerve roots are stretched. The correct answer is spinal
stenosis.
References:
Ross J, Zawadzki MB, Chen M, et al. Diagnostic imaging: spine. Salt Lake City: Saunders-Elsevier, 2004.
Question 277: Neuroimaging - Dementia
Discussion:
The MRI study shows a recent subcortical hemorrhage in the subcortical white matter of the right paracentral
region. This hemorrhage is responsible for the recent hemiparesis. The subcortical location of the hemorrhage is
typical of amyloid (or congophilic) angiopathy and very atypical for a hypertensive hemorrhage. Hemorrhages
do not tend to happen with either CADASIL or subcortical arteriosclerotic encephalopathy. Finally, the pattern
of multiple lesions in the white matter is not present in frontotemporal dementia.
References:
Chao CP, Kotsenas AL, Broderick DF. Cerebral amyloid angiopathy: CT and MR imaging findings.
Radiographics 2006;26:1517-1531.
Greenberg JO, editor. Neuroimaging: a companion to Adams and Victor's principles of neurology. 2nd ed. New
York: McGraw-Hill, 1999.
Question 279: Neuroimaging - Spine
Discussion:
An intradural, extramedullary mass in the thoracic spine is causing significant cord compression. This
represents either neurofibroma or meningioma and requires surgical removal. Neither tumor is radiosensitive.
Imaging with CT or myelography is not likely to add any further useful information.
References:
56
2007 RITE Discussion & Reference Manual
Osborn A. Diagnostic neuroradiology. St. Louis: Mosby, 1994.
Question 280: Neuroimaging - Stroke
Discussion:
The internal carotid artery segment shown is immediately proximal to the takeoff of the middle cerebral artery
(which goes to the right) and the anterior cerebral artery (which goes to the left).
References:
Osborn A. Introduction to cerebral angiography. Hagerstown, MD: Harper & Row, 1980.
Question 281: Neuroimaging - Critical Care/Trauma
Discussion:
The scan demonstrates findings highly suggestive of acute or subacute herpes simplex virus (HSV) encephalitis
involving the temporal lobe. The lesion crosses the vascular boundaries of the middle and posterior cerebral
arteries, making an ischemic insult unlikely. The contrast-enhancing pattern would be unusual for glioblastoma
or lymphoma. A contusion could occur in this location but would contain evidence of hemorrhage and MR
evidence of parenchymal blood degradation products, which is not seen in this case. HSV encephalitis primarily
involves limbic structures. MR typically shows temporal and inferior frontal lobe swelling, with low signal on
T1-weighted images and high signal on T2-weighted images. Postcontrast studies in the acute and subacute
setting often show cortical and leptomeningeal enhancement. The lesion may not be hemorrhagic in the early
stages.
References:
de Almeida Magalhaes AC, Bacheschi LA. Viral and nonviral infections of the nervous system. In: Greenberg
JO, editor. Neuroimaging: a companion to Adams and Victor's principles of neurology. 2nd ed. New York:
McGraw Hill, 1999.
Question 283: Neuroimaging - Critical Care/Trauma
Discussion:
The CT scan shows a collection of air in the right frontal lobe, with the bottom portion having an air fluid level.
The finding is not consistent with an artifact nor with a vascular event. Infection could be present; however, the
way in which the air would gain access to the intracranial compartment would have to involve a break in the
skull, most often in the air containing sinuses of the cranial base. In this case, the air collection entered through
a fracture of the frontal sinus. In order for the air to occupy as much space within the brain parenchyma as it
does, there would have to be a loss of substance of the brain focally at the site. This is consistent with the patient
having had a loss of brain substance (volume) due to a prior focal hemorrhagic contusion.
References:
Woodruff WW. Fundamentals of neuroimaging. Philadelphia: WB Saunders, 1993.
Question 284: Neuroimaging - CSF Circulation Disorders
Discussion:
The correct diagnosis is communicating hydrocephalus. Dilation of the Sylvian fissure and other sulci may
suggest atrophy. However, this feature is not uncommon in shunt-responsive hydrocephalus, as documented in
the references. On the coronal images note that the sulci at the high parietal convexity are compressed, as
compared to the markedly dilated posterior extent of the Sylvian fissure.
References:
Kitagaki H, Mori E, Ishii K, et al. CSF spaces in idiopathic normal pressure hydrocephalus: morphology and
volumetry. AJNR Am J Neuroradiol 1998;19:1277-1284.
57
2007 RITE Discussion & Reference Manual
Question 285: Neuroimaging - Brain Tumors
Discussion:
The postinjection T1-weighted MR image shows enhancement of an extra-axial small mass that displaces the
brain parenchyma away from the inner table of the skull. The inner table of the skull is focally expanded at the
center of the mass. The findings are characteristic of a meningioma. A "dural tail sign" is present on the
postcontrast image. An epidural hematoma would have been of high signal intensity on the preinjection study.
Without expansion of the bone, a depressed fracture would have a plane of cleavage within the calvarium and
may also have had an underlying cortical contusion.
References:
Berry I, Brant-Zawadski M, Osaki L, et al. Gd-DTPA in clinical MR of the brain: extra-axial lesions and normal
structures. AJNR Am J Neuroradiol 1986;7:789.
Question 286: Neuroimaging - Multiple Sclerosis
Discussion:
Enhancement is circumscribed to the margins of the lesion and affects only the white matter. Several patches of
enhancement are in subcortical white matter. The overall appearance of the enhancement resembles an open
ring, a finding characteristic of the large lesion (Schilder's) variety of multiple sclerosis. An infarction would
give rise to cortical enhancement. The other lesions tend to cause closed rings, and characteristically the gray
matter tends to enhance more than the white matter if the lesion is as close to the cortex as this one is. In this
patient the lesion shown improved on steriods. Six months later he developed a similar lesion in the frontal lobe.
References:
Masdeu JC, Quinto C, Olivera C, et al. Open-ring imaging sign: highly specific for atypical brain demyelination.
Neurology 2000;54:1427-1433.
Question 287: Neuroimaging - Stroke
Discussion:
A developmental venous anomaly (DVA) or venous angioma is occasionally identified in the work-up of
patients with neurological symptoms. Headache is the most common symptom. Seizures are also commonly
associated. There is no real evidence, however, that either headaches or seizures are caused by the DVA, and in
most instances the DVA is incidental. It is very important to understand that the DVA represents the venous
drainage of the brain tissue in which it is situated. Removal of the DVA may cause a venous infarction.
References:
Rigamonti D, Hsu FPK, Huhn S. Angiographically occult vascular malformations in neurovascular surgery. In:
Carter PL, Spetzler RF, editors. New York: McGraw-Hill, 1994.
Question 288: Neuroimaging - CSF Circulation Disorders
Discussion:
There is marked enlargement of the third and lateral ventricles, with a normal-sized fourth ventricle. Note also
the peculiar configuration of the quadrigeminal plate, which appears displaced and compressed superiorly, but
normally inferiorly, reflecting dilation of the proximal portion of the aqueduct only. The axial T2 image
through the midbrain also fails to demonstrate the normal flow void seen at the level of the aqueduct that
originated from high velocity of CSF traveling through the patent aqueduct. These findings are consistent with a
diagnosis of obstructive hydrocephalus, with obstruction at the level of the aqueduct (excluding the choices of
atrophy or communicating hydrocephalus). In this case, there is no evidence of a mass in the quadrigeminal
plate, and obstruction is due to stenosis.
References:
58
2007 RITE Discussion & Reference Manual
Bradley WB, Quencer RM. Hydrocephalus, atrophy, and intracranial CSF flow. In: Stark DD, Bradley WG.
Magnetic resonance imaging, Vol. 1. 2nd ed. St Louis: CV Mosby, 1992.
Question 289: Neuroimaging - Critical Care/Trauma
Discussion:
The sagittal two-dimensional phase contrast venogram with a velocity encoding of 15 cm/s is tailored to
demonstrate slow flow such as that in the dural venous sinuses. The scan demonstrates practically no flow in the
superior sagittal sinus (SSS). One prominent cortical vein shows flow. Just adjacent to this cortical vein, minor
flow is seen in the SSS. This finding is consistent with near occlusion of the SSS. Higher velocity encodings are
needed to demonstrate arterial flow.
References:
Medlock MD, Olivero WC, Hanigan WC, et al. Children with cerebral venous thrombosis diagnosed with
magnetic resonance imaging and magnetic resonance angiography. Neurosurgery 1992;31:870-876.
Question 290: Neuroimaging - Spine
Discussion:
In the images, the intervertebral disc near a postsurgical foreign body is obliterated, and there is enhancement of
the sheath of the psoas muscle. These findings are most suggestive of infection. Several MRI patterns have been
described as indicating spinal infections, including decreased disk height, disk hypointensity on T1-weighted
images, and hyperintensity on T2-weighted images, disk enhancement, erosion of vertebral endplates, the loss
of distinction between endplate and disk, along with the adjacent vertebral bodies on T1-weighted signal is most
frequently seen. These areas show enhancement. Spinal infections are most frequently associated with
paraspinal or epidural inflammatory tissues.
References:
Ledermann H, Schweitzer M, Morrison W, Carrino J. MR imaging findings in spinal infections: rules or myths?
Radiology 2003;228:506-514.
Question 292: Neuroimaging - Infection
Discussion:
The ring-enhancing lesion in the lower portion of the precentral region has the characteristics of a brain abscess.
Corresponding to the ring of enhancement in the gadolinium study is a dark rim in the FLAIR image. This rim
has a rich vascular network and abundant macrophages. The core of the lesion, containing an excess of water as
compared to brain, is hyperintense on T2 (FLAIR) and hypointense on T1. Around the ring there is brain
edema, also hyperintense on T2 and hypointense on T1. None of the other options have similar characteristics.
Astrocytomas of the types indicated seldom enhance. Areas of enhancement in an infarct tend to involve the
cortical ribbon whereas in the lesion shown the ring is at the corticomedullary junction, mostly in the white
matter. The lesion shown is in the brain parenchyma, not outside, as would happen with a meningioma.
References:
Kastrup O, Wanke I, Maschke M. Neuroimaging of infections. NeuroRx 2005;2:324-332.
Greenberg JO, editor. Neuroimaging: a companion to Adams and Victor's principles of neurology. 2nd ed. New
York: McGraw-Hill, 1999.
Question 293: Neuroimaging - Dementia
Discussion:
The enhanced CT image demonstrates multiple irregular ring-enhancing lesions adjacent to and compressing the
lateral ventricles. There is extensive white matter vasogenic edema in both hemispheres. This appearance is most
consistent with a glioblastoma that has spread through the corpus callosum to involve both hemispheres. In
immunocompromised patients, lymphoma can have a similar appearance, but in immunocompetent patients,
59
2007 RITE Discussion & Reference Manual
immunocompromised patients, lymphoma can have a similar appearance, but in immunocompetent patients,
lymphoma is most commonly seen as a homogenously enhancing mass. Herpes encephalitis in adults nearly
always affects the temporal lobes. Neurocysticercosis can occur in the brain parenchyma or as cysts within the
ventricles. First seen as cysts in the brain, cysticercosis can incite an inflammatory reaction when the larva dies,
but the lesions are usually small, well circumscribed, and located at the corticomedullary junction.
References:
Osborn AG. Diagnostic neuroradiology. St. Louis: CV Mosby, 1994.
Question 294: Neuroimaging - Developmental/Neurogenetic Disorders
Discussion:
The lesion at the tip of the left temporal lobe spares the cortical ribbon and is clearly extraaxial, ruling out old
head trauma or herpes simplex encephalitis. The signal characteristics are typical of CSF, ruling out a
metastasis. The lesion on the right perimesencephalic cistern cannot be incisural sclerosis, characterized by
gliosis not by a CSF-like lesion like the one shown. Both lesions are arachnoid cysts.
References:
Greenberg JO. Neuroimaging: a companion to Adams and Victor's principles of neurology. New York:
McGraw-Hill, 1999.
Question 296: Neuroimaging - Brain Tumors
Discussion:
Axial CTs show a large mass lesion filling the right orbit, compressing and deforming the posterior margin of
the globe. The inferior cut through the maxillary sinus shows that the lateral wall of the maxilla is destroyed and
is associated with the soft tissue mass. Metastatic disease commonly involves bone and adjacent soft tissue, as in
this case. Facial trauma does not produce a disappearance of bone but fractures it. Fibrous dysplasia expands
bone but does not destroy it. Hemangioma may involve bone and soft tissue, but is not a destructive process. In
the orbit, capillary hemangiomas are commonly found in the cutaneous tissues and periorbita but do not extend
retroglobar and do not destroy bone. Cavernous hemangiomas are discrete masses that may be found intraorbital
or extraorbital but do not usually cross these margins.
References:
Zimmerman RA, Bilaniuk LT. Computed tomography of the orbit. In: Lee S, Rao K, editors. Cranial
computerized tomography. New York: McGraw-Hill, 1983.
Question 299: Neuroimaging - Spine
Discussion:
The scans show active spondylitis at L4-L5 and remote spondylitis at L2-L3. Spondylitis may include
osteomyelitis, discitis, and epidural abscess formation, each of which are seen in the current case at L4-L5. The
enhancement and T2 hyperintensity of the L4-L5 disc is consistent with active discitis. In contrast, metastatic
disease usually spares and does not cross the disc space. Remote spondylitis often results in the changes seen in
the current case at L2-L3; the disc space is markedly narrowed, and the adjacent vertebral bodies show
degenerative changes. Incidental note is made of scoliosis, which is maximal at L2.
References:
Osborn AG. Diagnostic neuroradiology. St. Louis: CV Mosby, 1994.
Question 300: Neuroimaging - Dementia
Discussion:
The lesion in the left temporal lobe is isodense on T1 and enhances with gadolinium. It is outside the temporal
lobe. It causes compression of the tip and medial aspect of the temporal lobe, with partial herniation of the
60
2007 RITE Discussion & Reference Manual
parahippocampal gyrus. These features are characteristic of a meningioma. Both Alzheimer’s disease and
frontotemporal dementia would present with atrophy, not mass effect. Both an astrocytoma and a glioblastoma
are intraparenchymal tumors.
References:
Greenberg JO. Neuroimaging: a companion to Adams and Victor's principles of neurology. New York:
McGraw-Hill, 1999.
Question 301: Neuroimaging - Critical Care/Trauma
Discussion:
Dissection and total occlusion of the left internal carotid are incorrect because there is no evidence for
dissection. Emergency surgery is not required and it is clear that there is a large ulcerated plaque, not a
dissection. The ulcerated plaque could have resulted from the accident, but it is far more likely that it happened
spontaneously. If plaque material from within a crater this large had broken off and embolized during the
accident, the patient would have had serious neurological morbidity. Therefore, the correct answer is that the
accident did not cause the findings.
References:
Watridge CB, Muhlbauer MS, Lowery RD. Traumatic carotid artery dissection: diagnosis and treatment.
Neurosurg 1989;71(6):854-857.
Question 302: Neuroimaging - Critical Care/Trauma
Discussion:
The most likely diagnosis is subarachnoid hemorrhage. The high density of the recent bleeding outlines the
subarachnoid space around the brain. Its shape rules out the other types of hemorrhage mentioned as options.
There is hydrocephalus, but it is not idiopathic; rather, it is caused by the subarachnoid hemorrhage. The air with
pneumocephalus would have a much lower density than any of the structures seen on the image.
References:
Greenberg JO, editor. Neuroimaging: a companion to Adams and Victor's principles of neurology. 2nd ed. New
York: McGraw-Hill, 1999.
Question 303: Neuroimaging - Infection
Discussion:
Both the clinical picture and the imaging suggest a subacute process, with quick worsening. Of the given
options, listeriosis and multiple sclerosis usually have a subacute course. Of the two, the imaging is more typical
of listeriosis, an infection produced by Listeria monocytogenes that has a preference for the brainstem. In the
second study, the larger abscess in the tegmentum of the pons, visible in the gadolinium-enhanced study, is
accompanied by a subependymal cluster of smaller abscesses. In addition, there is enhancement of the
ependymal lining, suggesting the infectious nature of the lesion.
References:
Falini A, Kesavadas C, Pontesilli S, et al. Differential diagnosis of posterior fossa multiple sclerosis
lesions--neuroradiological aspects. Neurol Sci 2001;22 (Suppl 2):S79-S83.
Antal EA, Loberg EM, Dietrichs E, Maehlen J. Neuropathological findings in 9 cases of listeria monocytogenes
brain stem encephalitis. Brain Pathol 2005;15:187-191.
Question 305: Neuroimaging - Epilepsy
Discussion:
The lesion in the left hippocampus has the imaging characteristics of a cavernous angioma, with a rim of
61
2007 RITE Discussion & Reference Manual
low-intensity hemosiderin surrounding a cluster of high-intensity, fluid-containing vessels. The absence of
draining veins rules out both an arteriovenous malformation and a venous angioma. Astrocytoma and
oligodendroglioma do not have the imaging characteristics present in this MRI.
References:
Greenberg JO. Neuroimaging: a companion to Adams and Victor's principles of neurology. New York:
McGraw-Hill, 1999.
Question 307: Neuroimaging - Brain Tumors
Discussion:
The lesion of the left optic nerve is an astrocytoma. Neurofibromatosis type 1 (NF1) is associated with the
development of both benign and malignant tumors. The left optic nerve is thickened, suggesting the presence of
a tumor, but it does not enhance, suggesting that the lesion is not a meningioma, neurofibroma, or schwannoma.
Astrocytomas (glioma) are seen in 15% to 20% of these individuals. The most common locations include the
optic nerve, optic chiasm, hypothalamus, and, less often, the posterior fossa.
References:
Mohr JP, Gautier JC, editors. Guide to clinical neurology. New York: Churchill Livingstone, 1995.
Question 308: Neuroimaging - Brain Tumors
Discussion:
This extra-axial lesion diffusely and homogenously enhances with contrast. It sits on the dura. The morphology
and location make less likely the diagnoses of acoustic neurinoma and trigeminal schwannoma. Astrocytoma
and hemangioma are intra-axial tumors. This is an en-plaque variant meningioma.
References:
Mohr JP, Gautier JC, editors. Guide to clinical neurology. New York: Churchill Livingstone, 1995.
Question 309: Neuroimaging - Metabolic
Discussion:
The high subcortical signal in T2, affecting multiple gyri, suggests increased vascular permeability, as it happens
in all the conditions listed as possible answers. However, only eclampsia fails to have other findings and causes
white matter changes with the topography and shape present in the images. Multiple sclerosis tends to cause
sharper lesions, with a periventricular distribution. In abscesses, there are round, cystic collections. There is no
evidence of nodular deposits of choriocarcinoma. Emboli in these locations would be accompanied by cortical
necrosis, not evident in this image.
References:
Orrison WW. Neuroimaging. Philadelphia: WB Saunders, 2000.
Question 314: Neuroimaging - Stroke
Discussion:
The CT and MRI scan findings are highly consistent with Sturge-Weber syndrome (SWS). Cerebral lesions in
SWS are most frequently found in the parieto-occipital area. On CT scans, parenchymal calcification is found
adjacent to the leptomeningeal lesions in a pericapillary distribution in the fourth cortical layer, possibly
secondary to chronic tissue hypoxia. Noncontrast MR images in SWS typically show unilateral cerebral atrophy
with enlargement of the ipsilateral subarachnoid spaces and ventricular cavities. Enlarged tubular regions of
flow void and gyriform hypointensities (decreased signal on T1-weighted and T2-weighted images) reflect
venous collateralization and cortical "serpentine" calcification, respectively. Contrast-enhanced MRI is best for
demonstrating the salient CNS vascular abnormalities in SWS. The leptomeninges show intense enhancement
overlying the involved cortical region secondary to leptomeningeal (pial) angiomatosis or a compromised
62
2007 RITE Discussion & Reference Manual
blood-brain barrier secondary to chronic cortical ischemia. This meningeal enhancement often extends well
beyond the region of parenchymal atrophy.
References:
Provenzale JM, Tavares JM. Clinical cases in neuroradiology. Philadelphia: Lea & Febiger, 1994.
Question 316: Neuroimaging - Critical Care/Trauma
Discussion:
Pituitary apoplexy is a syndrome of infarction or hemorrhage of the pituitary gland. It is associated with
headache, ophthalmoparesis, visual defects, and encephalopathy. It may occur acutely or subacutely and can be
associated with coma or death. It occurs more often in pituitary adenomas.
References:
Rogg JM, Tung GA, Anderson G, Cortez S. Pituitary apoplexy: early detection with diffusion-weighted MR
imaging. AJNR Am J Neuroradiol 2002;23:1240-1245.
Question 347: Neuroimaging - Technical
Discussion:
The internal cerebral vein, which is one of the major components of the deep draining system of the brain, is
located in the roof of the third ventricle between the leaflets of the veli interpositi.
References:
Parent A. Carpenter's human neuroanatomy. 9th ed. Baltimore: Williams & Wilkins, 1996.
Question 356: Neuroimaging - Technical
Discussion:
Of the medical devices and conditions listed, only cardiac pacemakers constitute an absolute contraindication for
MRI. Many heart valves are MR compatible, particularly those produced in the last decade. Middle ear
prosthesis are not considered hazardous to patients, but the device itself can be damaged. An MRI can be
performed in a patient with an aneurysm clip depending on the material used for the clip. Pregnancy is a relative
but not absolute contradiction to MRI. It should be used with caution in the first trimester.
References:
Bushong SC. Magnetic resonance imaging: physical and biological principles. St. Louis: CV Mosby, 1996.
Question 425: Neuroimaging - Stroke
Discussion:
Luxury perfusion refers to increased perfusion in the periphery of an ischemic area. It is best documented with
PET or SPECT. Some authors have used the term to refer to gray matter enhancement, which is best seen on
MRI 3 days to several weeks after a cerebral infarct and results from contrast leaking into the perivascular space
in an area of ischemic insult.
References:
Bakshi R, Ketonen L. Brain MRI in clinical neurology. In: Joynt RJ, Griggs RC, editors. Baker’s clinical
neurology. Philadelphia: Lippincott, Williams & Wilkins, 2001.
Pathology
Question 17: Pathology - Demyelinating Disease
63
2007 RITE Discussion & Reference Manual
Discussion:
Neuromyelitis optica antibody is a newly developed serum antibody test that identifies up to 70% of cases of
neuromyelitis optica (Devic's disease). The test is negative in standard cases of multiple sclerosis and in other
demyelinating and disorders disorders.
References:
Lennon VA, Wingerchuk DM, Kryzer TJ, et al. A serum autoantibody marker of neuromyelitis optica:
distinction from multiple sclerosis. Lancet 2004;364:2106-2112.
Question 42: Pathology - Demyelinating Disease
Discussion:
Huffing glue causes diffuse leukoencephalopathy, not neuronal necrosis in either cerebral or cerebellar cortices.
References:
Graham DI, Lantos PL. Greenfield's neuropathology. 7th ed. London: Arnold, 2002.
Question 73: Pathology - Cerebrovascular Disease
Discussion:
Dolichoectasia (fusiform aneurysm) most commonly affects the supraclinoid segment of the internal carotid
artery and the basilar artery, and is seen in patients with advanced cerebral atherosclerosis. Berry aneurysms are
associated with both coarctation of the aorta and arteriovenous malformations (3% to 9% of patients with
intracranial AVMs have berry aneurysms), but fusiform aneurysms are not. Fungi cause arteritis and aneurysmal
change but not the elongated vessel distention of dolichoectasia. HIV occasionally is associated with but perhaps
not the direct cause of smaller vessel vasculitis.
References:
Graham DI, Lantos PL. Greenfield's neuropathology. 7th ed. London: Arnold, 2002.
Question 80: Pathology - Epilepsy
Discussion:
Although the etiology of hippocampal (Ammon's horn sclerosis) is controversial, with some authors contending
the lesions are the cause, and others the result, of the seizures, the lesion is nonetheless the most common
pathology found in patients with tissue resected for temporal lobe epilepsy. Seizure control after temporal
lobectomy is best with hippocampal sclerosis but is also very good with neoplasms and vascular malformations,
although these are less common causes of temporal lobe epilepsy. End folium sclerosis with neuronal loss
confined to the CA4 sector of the hippocampus is rare, as is Rasmussen's encephalitis.
References:
Burger P, Scheithauer B, Vogel FS. Surgical pathology of the nervous system and its coverings. 4th ed.
Philadelphia: Churchill Livingstone, 2002.
Question 93: Pathology - Critical Care/Trauma
Discussion:
Hemorrhage and necrosis of the central core of the spinal cord usually results from traumatic subluxation injury
of the cervical spine. Rarely, hematomyelia can also result from intraspinal vascular malformations.
References:
Graham DI, Lantos PL. Greenfield's neuropathology. 7th ed. London: Arnold, 2002.
Question 104: Pathology - Epilepsy
64
2007 RITE Discussion & Reference Manual
Discussion:
Of the tumor types listed, the most likely to show a superficial cystic appearance and to occur in the temporal
lobe of a young adult is the pleomorphic xanthoastrocytoma (PXA), a WHO grade II tumor. Eosinophilic
granular bodies are histological features most commonly found in low grade gliomas, such as pleomorphic
xanthoastrocytoma. They are small aggregates of eosinophilic, hyaline material situated in the processes of
astrocytes.
References:
Burger P, Scheithauer B, Vogel FS. Surgical pathology of the nervous system and its coverings. 4th ed.
Philadelphia: Churchill Livingstone, 2002.
Question 133: Pathology - Tumors
Discussion:
Angiotropic (intravscular) lymphoma is a disease of multiple organs that involves the central nervous system in
greater than 30% of cases. It is usually a B cell lymphoma by immunophenotyping, and causes multiple small
brain infarcts, often of deep white matter. A patient with CNS involvement by angiotropic lymphoma may show
lymphoma cells only within the blood vessels and may not have either intraparenchymal lymphomatous masses
or significant meningeal involvement. The old name for this tumor prior to the advent of specific
immunohistochemical markers for lymphoma was "malignant angioendotheliomatosis", reflceting the erroneous
belief that the tumor cells were endothelial in origin.
References:
Graham DI, Lantos PL. Greenfield's neuropathology. 7th ed. London: Arnold, 2002.
Question 135: Pathology - Toxic/Metabolic Disease
Discussion:
Peripheral neuropathies can be caused by inorganic arsenic, lead, mercury, and thallium. Aluminum intoxication
can produce an encephalopathy but does not produce a neuropathy. Cyanide, manganese, and bismuth do not
cause peripheral neuropathy.
References:
Graham DI, Lantos PL. Greenfield's neuropathology. 7th ed. London: Arnold, 2002.
Question 139: Pathology - Hypothalmus/Pituitary
Discussion:
Bromocriptine (Parlodel) and cabergoline are dopamine agonists that function by the same mechanism as
prolactin inhibitory factor (PIF, which is dopamine) to inhibit prolactin synthesis and release by
prolactin-secreting pituitary adenomas (prolactinomas). Bromocriptine or cabergoline treatment causes a
decrease in the tumor cell cytoplasmic volume without actually killing the adenoma cells. Therefore, the
prolactinoma may reexpand if bromocriptine is withdrawn. Bromocriptine has no significant role in
neuro-oncology other than in suppression of prolactin-secreting tumors.
References:
Burger P, Scheithauer B, Vogel FS. Surgical pathology of the nervous system and its coverings. 4th ed.
Philadelphia: Churchill Livingstone, 2002.
Question 156: Pathology - Neurodegenerative Disease
Discussion:
Familial frontotemporal dementia that arises from a mutation on chromosome 17 results in a mutation of the
gene encoding for tau protein.
65
2007 RITE Discussion & Reference Manual
References:
Ellison D, Love S, Chimelli L, et al. Neuropathology: a reference text of CNS pathology. 2nd ed. Edinburgh:
Mosby, 2004.
Question 164: Pathology - Critical Care/Trauma
Discussion:
Critical care myopathy is an underrecognized disorder characterized by selective loss of myosin from myofibers.
Also known as myosin-losing myopathy, this disorder is most commonly seen in critically ill patients treated
with corticosteroids and neuromuscular blockers.
References:
Ruff RL. Why do ICU patients become paralyzed? Ann Neurol 1998;43:154-155.
Question 180: Pathology - Toxic/Metabolic Disease
Discussion:
Hemorrhagic necrosis of the putamen is characteristic of methanol poisoning. Methanol may be ingested as an
alcohol substitute. There is selective vulnerability of the putamen, among the choices listed.
References:
Schochet SS, Gray F. Acquired metabolic disorders. In: Gray F, De Girolami U, Poirier J. Escourolle and
Poirier manual of basic neuropathology. 4th ed. Boston: Butterworth-Heinemann 2004; 198.
Question 187: Pathology - Neurodegenerative Disease
Discussion:
Corticobasal degeneration classically presents with progressive asymmetric apraxias, rigidity, and/or aphasia due
to neuronal loss and atrophy of the peri-Sylvian region. Ballooned neurons and astrocytic plaques with
tau-immunoreactivity are a diagnostic feature of corticobasal degeneration.
References:
Graham DI, Lantos PL. Greenfield's neuropathology. 7th ed. London: Arnold, 2002.
Questions 224 - 228: Pathology - Neuromuscular Disease
Discussion:
Perifascicular atrophy is virtually pathognomonic of dermatomyositis and is particular characteristic of juvenile
dermatomyositis. The image is of an onion bulb or hypertrophic neuropathy and best matches a clinical scenario
suggestive of an indolent demyelinating neuropathy such as Charcot-Marie-Tooth disease 1a. The figure depicts
a ragged-red fiber as are seen in some, but not all mitochondrial encephalomyopathies. The ocular system,
including the eye and ocular muscles are frequently involved in mitochondrial disease. This shows an H&E
stained muscle fiber with a rimmed vacuole, characteristic of the most common inflammatory in older adults,
inclusion body myositis. Neurogenic atrophy (group atrophy) of skeletal muscle is characteristic of spinal
muscular atrophy type I (Werdnig-Hoffman disease).
References:
Lidov H, De Girolami U, Gherardi R. Skeletal muscle diseases. In: Gray F, De Girolami U, Poirier J. Escourolle
and Poirier manual of basic neuropathology. 4th ed. Boston: Butterworth-Heinemann, 2004;
287,302-304,309-312,308-310.
Vallat J-M, Anthony D, De Girolami U. Peripheral nerve diseases. In: Gray F, De Girolami U, Poirier J.
Escourolle and Poirier manual of basic neuropathology. 4th ed. Boston: Butterworth-Heinemann 2004; 337-338.
66
2007 RITE Discussion & Reference Manual
Question 229: Pathology - Developmental
Discussion:
This brain shows multiple areas of cavitation (liquefaction necrosis) involving the deeper cortical layers and
underlying white matter, a condition known as multicystic encephalopathy or multicystic encephalomalacia.
The lesions are most pronounced in the distribution of the anterior and middle cerebral arteries. They are the
result of circulatory disturbances during the latter half of pregnancy or the neonatal period.
References:
Nelson J, Mena H, Parisi JE, Schochet SS, editors. Principles and practice of neuropathology. 2nd ed. New
York: Oxford University Press, 2003.
Question 236: Pathology - Tumors
Discussion:
The biopsy of a right parietal lobe mass showed a tumor composed of cells with monotonous round nuclei
surrounded by prominent perinuclear halos ("fried egg" apppearance). These features are characteristic of
oligodendroglioma. Of the other choices listed, ependymomas and astroblastomas characteristically show
perivascular pseudorosettes, with stout nontapering cytoplasmic processes in the latter. Fibrillary astrocytomas
and gemistocytic astrocytomas show eosinophilic cytoplasm rather than the clear perinuclear halos of
oligodendrogliomas, with multiple elongated cytoplasmic processes in fibrillary astrocytomas and large rounded
globular cytoplasm in gemistocytic astrocytomas.
References:
Burger P, Scheithauer B, Vogel FS. Surgical pathology of the nervous system and its coverings. 4th ed.
Philadelphia: Churchill Livingstone, 2002.
Question 239: Pathology - Tumors
Discussion:
The microscopic appearance is that of a capillary-rich neoplasm with abundant foamy cells containing lipid. This
is a hemangioblastoma. The cell origin of the hemangioblastoma is still unknown, despite many investigative
efforts. The tumor arises independently or in conjunction with von Hippel-Lindau disease. The most common
location is in the cerebellum, but the spinal cord can also be a site of origin. Pilocytic astrocytomas, also
commonly found in the cerebellum, are composed of astrocytes with elongate, bipolar, eosinophilic cell
processes; these tumors often show Rosenthal fibers. Medulloblastomas are malignant small blue cell tumors of
neuronal origin that manifest little visible cytoplasm. Ependymomas are glial tumors which show formation of
ependymal canals, or more commonly, perivascular pseudorosettes.
References:
Burger P, Scheithauer B, Vogel FS. Surgical pathology of the nervous system and its coverings. 4th ed.
Philadelphia: Churchill Livingstone, 2002.
Question 248: Pathology - Cerebrovascular Disease
Discussion:
Autosomal dominant cerebral cavernous angioma (malformation) (CCM1) syndrome occurs in Hispanic
individuals and is characterized by intraparenchymal cavernous malformations that can produce seizures,
impairment of function, or hemorrhage. The photograph illustrates a multi-channeled vascular malformation
composed of closely juxtaposed hyalinized, non-arterial vessels.
References:
Ellison D, Love S, Chimelli L, et al. Neuropathology: a reference text of CNS pathology. 2nd ed. Edinburgh:
Mosby, 2004.
67
2007 RITE Discussion & Reference Manual
Question 249: Pathology - Neuromuscular Disease
Discussion:
The muscle biopsy specimen shows a necrotizing vasculitis with fibrinoid necrosis of the vessel walls, not
amyloid angiopathy. Vasculitis may be seen in amphetamine-induced vasculitis, polyarteritis nodosa, rheumatoid
vasculitis, and Wegener's granulomatosis, but not in polymyalgia rheumatica which is associated with temporal
arteritis and usually type II fiber atrophy in the muscle biopsy, not vasculitis. Takayasu's arteritis affects large
blood vessels such as aortic arch, not small intramuscular vessels. ALS causes no inflammation of intramuscular
blood vessels, but does show neurogenic atrophy.
References:
Nelson J, Mena H, Parisi JE, Schochet SS, editors. Principles and practice of neuropathology. 2nd ed. New
York: Oxford University Press, 2003.
Question 251: Pathology - Cerebrovascular Disease
Discussion:
The lesion shown in this image is a remote infarct. The preservation of the outer layer of the cortex would argue
against a remote contusion. An old hematoma cavity would be smoother, and would not necessairly be confined
to the vascular distribution territory of a branch of the middle cerebral artery as this lesion is. Both an abscess
and metastatic carcinoma would be accompanied by brain swelling rather than shrinkage; note the lack of mass
effect on the ventricular system.
References:
Graham DI, Lantos PL. Greenfield's neuropathology. 7th ed. London: Arnold, 2002.
Question 252: Pathology - Cerebrovascular Disease
Discussion:
The gross findings are those of a remote hemorrhagic stroke. Microscopically, the blood vessels of the meninges
and cortex showed amorphous eosinophilic material indicative of cerebral amyloid angiopathy. Neither primary
or secondary systemic amyloidoses cause amyloid deposits within cerebral blood vessels. Cerebral autosomal
dominant arteriopathy with subcortical infarcts and leukoencephalopathy (CADASIL) deposits would not stain
with amyloid stains such as thioflavin S or Congo red and would not be as homongeneous or affect the
superficial cortex and meninges. Chronic hypertension causes thickened walls of small blood vessels due to
hyalinization and lipohyalinosis; no amyloid is deposited.
References:
Graham DI, Lantos PL. Greenfield's neuropathology. 7th ed. London: Arnold, 2002.
Question 263: Pathology - Developmental
Discussion:
The defect is anencephaly, the most common of the neural tube closure defects. Folate supplementation has been
known to lower the recurrence risk of these defects however a recent report also documents the presence of
anti-folate receptor antibodies in some pregnancies complicated by neural tube closure defects.
References:
Rothenberg SP, da Casta M, Sequeira J, et al. Autoantibodies against folate receptors in women with a
pregnancy complicated by a neural-tube defect. N Engl J Med 2004;350:134-142.
Encha-Razavi F, Folkerth R, Harding B. Congenital malformations and perinatal diseases. In: Gray F, De
Girolami U, Poirier J. Escourolle and Poirier manual of basic neuropathology. 4th ed. Boston:
Butterworth-Heinemann, 2004;250.
68
2007 RITE Discussion & Reference Manual
Question 264: Pathology - Critical Care/Trauma
Discussion:
The graphic shows an acute subdural hematoma. Subdural hematomas result from disruption of bridging veins
and are seen more commonly in older persons with cortical atrophy with resultant tension on these veins. The
inciting trauma often is a fall, but may not be recalled.
References:
Graham DI, Lantos PL. Greenfield's neuropathology. 7th ed. London: Arnold, 2002.
Question 269: Pathology - Demyelinating Disease
Discussion:
The photograph shows syringomyelia. The lesion consists of a large cystic space, which would not be seen with
subacute combined degeneration, AIDS-associated myelopathy, or mutliple sclerosis. Syringomyelia is a cystic
lesion in the spinal cord that interrupts the crossing pain fibers and damages anterior horn cells. Muscle atrophy,
often results but vibratory, position, and touch senses are usually spared.
References:
Graham DI, Lantos PL. Greenfield's neuropathology. 7th ed. London: Arnold, 2002.
Question 271: Pathology - Infectious Disease
Discussion:
The photograph shows abundant exudate, especially along vessels, in the subarachnoid space on the dorsal
surface of the brain. This is most consistent with purulent leptomeningitis. Hemophilus influenzae rarely causes
meningitis in the adult, and pneumococcal meningitis is more common than meningococcal meningitis in the
elderly patient. With this history of alcoholism and asplenism, the most likely diagnosis is pneumococcal
meningitis.
References:
Nelson J, Mena H, Parisi JE, Schochet SS, editors. Principles and practice of neuropathology. 2nd ed. New
York: Oxford University Press, 2003.
Question 273: Pathology - Basic Reactions
Discussion:
The photograph shows Wallerian degeneration, with shrinkage of one medullary pyramid and atrophy of the
ipsilateral cerebral peduncle. The Wallerian degeneration is secondary to destruction of the corticospinal tract
above this level, such as an infarction in the posterior limb of the internal capsule.
References:
Graham DI, Lantos PL. Greenfield's neuropathology. 7th ed. London: Arnold, 2002.
Question 275: Pathology - Neurodegenerative Disease
Discussion:
The gross photograph of the brain shows lobar atrophy. Lobar atrophy of the frontal and temporal lobes is
typical of Pick's disease. Microscopically, intracytoplasmic argyrophilic neuronal inclusions and gliosis are
observed in Pick body Pick's disease.
References:
Graham DI, Lantos PL. Greenfield's neuropathology. 7th ed. London: Arnold, 2002.
69
2007 RITE Discussion & Reference Manual
Question 278: Pathology - Demyelinating Disease
Discussion:
The spinal cord shows tract degeneration in the dorsal spinocerebellar columns and posterior columns, with
myelin loss highlighted by the Luxol fast blue-periodic acid Schiff stain for myelin. Tract degeneration is seen in
Friedreich's ataxia, but not multiple sclerosis or acute disseminated encephalomyelitis. Tabes dorsalis affects the
posterior columns of the spinal cord, but not the dorsal spinocerebellar tracts.
References:
Graham DI, Lantos PL. Greenfield's neuropathology. 7th ed. London: Arnold, 2002.
Question 282: Pathology - Prion Disease
Discussion:
The image shows spongiform change of the cerebral cotex consistent with a prion disease. Neuronal loss and
gliosis complete the classic histologic triad. Prion diseases are caused by misfolding of a normal cellular protein
PrP. Such misfolding may be induced by mutation or by exposure of normal cellular prion protein to pathogenic
prions. Misfolding alters the secondary structure such that the protein becomes highly resistant to chemical or
thermal methods of strerilization.
References:
Graham DI, Lantos PL. Greenfield's neuropathology. 7th ed. London: Arnold, 2002.
Question 291: Pathology - Neurodegenerative Disease
Discussion:
The brain shows severe atherosclerosis of intracranial vessels as well as bilateral remote anterior cerebral artery
infarctions. The ventricles are slightly enlarged due to hydrocephalus ex vacuo. This patient actually had a
single, unpaired anterior cerebral artery, accounting for the bilateral nature of the infarctions. Bilateral anterior
cerebral artery territory infarcts often result in profound apathy, lack of motivational intent, akinetic mutism,
weakness of the lower extremities and urinary incontinence.
References:
Ellison D, Love S, Chimelli L, et al. Neuropathology: a reference text of CNS pathology. 2nd ed. Edinburgh:
Mosby, 2004.
Question 297: Pathology - Cerebrovascular Disease
Discussion:
The photograph shows a massive basal ganglionic hemorrhage with rupture into the ventricular system. The
location of this hematoma is common for hypertensive intracerebral hemorrhage. Hypertension is a major risk
factor for this type of hemorrhage, and hypertension is associated with pathologic and electrocardiographic
evidence of left ventricular hypertrophy. Prostate cancer only rarely metastasizes to brain parenchyma and is
usually not hemorrhagic. Visceral cysts are not relevant to intracerebral hemorrhage, but are frequently seen in
von Hippel-Lindau disease with associated CNS hemangioblastoma.
References:
Graham DI, Lantos PL. Greenfield's neuropathology. 7th ed. London: Arnold, 2002.
Question 298: Pathology - Infectious Disease
Discussion:
Fungal infections with Coccidioides occur in regions of semiarid climate. In the United States, the Southwest
and California are the most common locations for these infections. Mature fungi in tissue are endospores, such
as the one pictured, which are round and have a refractile wall.
70
2007 RITE Discussion & Reference Manual
References:
Nelson J, Mena H, Parisi JE, Schochet SS, editors. Principles and practice of neuropathology. 2nd ed. New
York: Oxford University Press, 2003.
Question 310: Pathology - Developmental
Discussion:
This photograph of a coronal section of immature brain shows small foci of dystrophic calcification in white
matter corresponding to periventricular leukomalacia (PVL). This is the principal ischemic lesion of
prematurity. Microscopy would show coagulative necrosis, dystrophic calcifications, microglial activation, and
axonal injury.
References:
Encha-Razavi F, Folkerth R, Harding B. Congenital Malformations and Perinatal Diseases. In: Gray F, De
Girolami U, Poirier J. Escourolle and Poirier Manual of Basic Neuropathology. 4th ed. Boston:
Butterworth-Heinemann, 2004; 267.
Question 315: Pathology - Cerebrovascular Disease
Discussion:
The lesion is seen in the cortex of the gyri to the right of midline in this coronal section and is a typical example
of laminar necrosis due to hypoxic/ischemic injury.
References:
Schochet SS, Gray F. Acquired metabolic disorders. In: Gray F, De Girolami U, Poirier J. Escourolle and
Poirier manual of basic neuropathology. 4th ed. Boston: Butterworth-Heinemann 2004; 198.
Question 318: Pathology - Demyelinating Disease
Discussion:
Marburg's disease, also known as acute multiple sclerosis, can cause severe cerebral damage and render a
patient comatose within a few weeks. The patient does not go into remission and may die within weeks or
months after onset of the demyelinating disorder.
References:
Fernandez H, Eisenschenk S, Yachnis AT, Okun MS. Ultimate review for the neurology boards. 1st ed. New
York: Demos Medical Publishing, 2006.
Question 329: Pathology - Neurodegenerative Disease
Discussion:
The nucleus of Onufrowicz of the sacral cord is spared in motor neuron disease. The remaining motor nuclei are
involved to varying degrees in amyotrophic lateral sclerosis.
References:
Graham DI, Lantos PL. Greenfield's neuropathology. 7th ed. London: Arnold, 2002.
Question 338: Pathology - Infectious Disease
Discussion:
In tabes dorsalis, the degeneration is confined to the dorsal columns, whereas in AIDS myelopathy, amyotrophic
lateral sclerosis, Friedreich's ataxia, and pernicious anemia degeneration occurs in other tracts.
References:
71
2007 RITE Discussion & Reference Manual
References:
Graham DI, Lantos PL. Greenfield's neuropathology. 7th ed. London: Arnold, 2002.
Question 342: Pathology - Neurodegenerative Disease
Discussion:
Choline acetyltransferase is the synthetic enzyme for acetylcholine and is a marker of the axonal termini of
cholinergic neurons. The neurons of the basal nucleus are the major cholinergic projections to the cerebral
cortex, and with depopulation of this projection nucleus there is a corresponding reduction in cortical choline
acetyltransferase.
References:
Graham DI, Lantos PL. Greenfield's neuropathology. 7th ed. London: Arnold, 2002.
Question 351: Pathology - Neurodegenerative Disease
Discussion:
Paraneoplastic cerebellar degneration is most commonly associated with ovarian carcinoma and some breast
tumors. It can also be seen with Hodgkin's lymphoma and small cell carcinoma of the lung.
References:
Ellison D, Love S, Chimelli L, et al. Neuropathology: a reference text of CNS pathology. 2nd ed. Edinburgh:
Mosby, 2004.
Question 357: Pathology - Toxic/Metabolic Disease
Discussion:
Ethylene glycol toxicity has birefringent calcium oxalate deposits in and around blood vessels. Oxalic acid
crystals may be detected in the urine.
References:
Graham DI, Lantos PL. Greenfield's neuropathology. 7th ed. London: Arnold, 2002.
Question 365: Pathology - Demyelinating Disease
Discussion:
Cerebral autosomal dominant arteriopathy with subcortical infarcts and leukoencephalopathy (CADASIL)
results from missense mutations of the Notch 3 gene on chromosome 19. The small deep white matter vessels
show replacement of the media by eosinophilic, periodic acid-Schiff positive, Congo red negative, granular
material. Ultrastructurally, there is compact electron-dense material known as granular osmophilic material
surrounding myocytes in the arterial media. Although the predominant effects of CADASIL are on the central
nervous system, it is a systemic vasculopathy that can be appreciated on skin or nerve biopsy. Vasculitis is not
seen in CADASIL.
References:
Ellison D, Love S, Chimelli L, et al. Neuropathology: a reference text of CNS pathology. 2nd ed. Edinburgh:
Mosby, 2004.
Question 375: Pathology - Tumors
Discussion:
Patients with tuberous sclerosis have a variety of systemic and CNS tumors. In the brain, astrocytic hamartomas
called tubers and candle gutterings are present. Subependymal giant cell astrocytomas are true neoplasms arising
in the vicinity of the foramen of Monro. These tumors may produce CSF obstruction, and they may rarely
72
2007 RITE Discussion & Reference Manual
undergo malignant degeneration.
References:
Kleihues P, Cavenee WK, editors. Pathology and genetics of tumors of the nervous system. 2nd ed. New York:
Oxford University Press, 2000.
Burger P, Scheithauer B, Vogel FS. Surgical pathology of the nervous system and its coverings. 4th ed.
Philadelphia: Churchill Livingstone, 2002.
Question 385: Pathology - Critical Care/Trauma
Discussion:
The injury described above fits best with a contrecoup type injury. The individual decelerates when he falls and
strikes the back of his head. Because the head was in motion and he struck his occiput, the contrecoup sites will
be the frontal lobes and temporal tips. The irregular bony contours within the cranial vault in the anterior and
middle fossae exacerbate the injury.
References:
Nelson J, Mena H, Parisi JE, Schochet SS, editors. Principles and practice of neuropathology. 2nd ed. New
York: Oxford University Press, 2003.
Question 395: Pathology - Tumors
Discussion:
Rhabdoid meningioma is a variant of meningioma that often has aggressive clinical behavior with numerous
recurrences and the potential for metastasis. It is a World Health Organization (WHO) grade III tumor. Many
meningiomas have distinctive histological appearances, such as secretory, fibrous, and psammomatous subtypes,
but have no adverse prognosis associated specifically with their morphology and are usually WHO grade I
tumors.
References:
Burger P, Scheithauer B, Vogel FS. Surgical pathology of the nervous system and its coverings. 4th ed.
Philadelphia: Churchill Livingstone, 2002.
Question 426: Pathology - Basic Reactions
Discussion:
The type of edema seen with acute, obstructive, high-pressure hydrocephalus is interstitial edema. Hydrostatic
edema is seen when high arterial pressures overcome cerebrovascular resistance and drive water across the
capillary walls into the extracellular space. Vasogenic edema is the most frequent type of edema and is seen
around tumors or traumatic lesions. Hypo-ostomic edema occurs with marked reduction in serum osmolality,
and cytotoxic edema is most commonly caused by hypoxia.
References:
Graham DI, Lantos PL. Greenfield's neuropathology. 7th ed. London: Arnold, 2002.
Question 445: Pathology - Hypothalmus/Pituitary
Discussion:
Neurosarcoidosis is uncommon, even in patients with systemic sarcoidosis. Of the anatomic sites mentioned, the
hypothalamus is most commonly affected. It may be assoicated with a peripheral neuropathy or myopathy and
on biopsy is a histological diagnosis of exclusion. Specifically, fungi and tuberculosis must be eliminated as
diagnoses in cases with granulomatous inflammation.
References:
73
2007 RITE Discussion & Reference Manual
Ellison D, Love S, Chimelli L, et al. Neuropathology: a reference text of CNS pathology. 2nd ed. Edinburgh:
Mosby, 2004.
Pharmacology/Chemistry
Question 5: Pharmacology/Chemistry - Movement Disorders
Discussion:
Suboptimal doses of carbidopa are a frequent cause of nausea/vomiting at the initiation of levodopa therapy. It
takes 100 mg to 150 mg of carbidopa per day to saturate the peripheral aromatic aminoacid decarboxylase
enzyme to prevent peripheral side effects of levodopa. Promethaziine and metaclopramide block dopamine
receptors and may worsen the parkinsonism.
References:
Roger E, Kurlan JB. Treatment of movement disorders. Philadelphia: Lippincott, 1995.
Question 18: Pharmacology/Chemistry - Neuromuscular Disorders
Discussion:
Vitamin B12 (cobalamin) plays an important role in DNA synthesis and neurologic function. Deficiency can lead
to a wide spectrum of hematologic, neurologic, and psychiatric manifestations. It is a common cause of
macrocytic (megaloblastic) anemia and, in advanced cases, pancytopenia. Neurologic sequelae from vitamin B12
deficiency include paresthesias, peripheral neuropathy, and demyelination of the corticospinal tract and dorsal
columns (subacute combined systems disease). The diagnosis of vitamin B12 deficiency has traditionally been
based on low serum vitamin B12 levels, usually less than 200 pg per mL (150 pmol per L), along with clinical
evidence of disease. However, studies indicate that older patients tend to present with neuropsychiatric disease in
the absence of hematologic findings.Furthermore, measurements of metabolites such as methylmalonic acid and
homocysteine have been shown to be more sensitive in the diagnosis of vitamin B12 deficiency than
measurement of serum B12 levels alone
References:
Oh RC, Brown DL. Vitamin B12 deficiency. Am Fam Physician 2003; 67(5):979-986.
Question 20: Pharmacology/Chemistry - Headache
Discussion:
Giant cell emporal arteritis is a vasculitis can lead to devastating ophthalmic and systemic complications. Jaw
claudication, superficial temporal artery tenderness, fever, and visual loss are seen. Treatment primarily to
prevent visual loss requires extended management with corticosteroids - initially at a high dose (e.g. Prednisone
80-100 mg/d).
References:
Bhatti MT, Tabandeh H. Giant cell arteritis: diagnosis and management. Curr Opin Ophthalmol
2001;12(6):393-399.
Question 21: Pharmacology/Chemistry - Headache
Discussion:
Sumatriptan and other triptans are felt to provide acute relief for migraine by acting as 5HT1 receptors,
specifically 1b and 1d, resulting in decreased activity in the trigeminovascular system. Drugs acting at
adrenergic, specifically beta and alpha, receptors provide prophylactic effects.
References:
Tepper SJ, Rapoport AM, Sheftell FD. Mechanisms of action of the 5-HT1B/1D receptor agonists. Arch Neurol
74
2007 RITE Discussion & Reference Manual
2002;59(7):1084-1088.
Question 25: Pharmacology/Chemistry - Other Pain Syndromes
Discussion:
Transient receptor potential (TRP) ion channels are molecular gateways in sensory systems, an interface between
the environment and the nervous system. Several TRPs transduce thermal, chemical, and mechanical stimuli
into inward currents, an essential first step for eliciting thermal and pain sensations. Precise regulation of the
expression, localization, and function of the TRP channels is crucial for their sensory role in nociceptor
terminals, particularly after inflammation when they contribute to pain hypersensitivity by undergoing changes
in translation and trafficking as well as diverse posttranslational modifications.
References:
Story GM, Gereau RW 4th. Numbing the senses: role of TRPA1 in mechanical and cold sensation. Neuron
2006;50(2):177-180.
Wang H, Woolf CJ. Pain TRPs. Neuron 2005;46(1):9-12.
Question 26: Pharmacology/Chemistry - Headache
Discussion:
Cluster headache will often respond acutely to oxygen inhalation at a flow rate of 8 L/min to 10 L/min via face
mask. It may be triggered or exacerbated by vasodilating substances such as nitroglycerin, histamine, and
ethanol. Drugs such as triptans, steroids, and dihydroergotamine may also be helpful in acute headache
management. Verapamil, lithium, and methysergide can be effective prophylactic agents for cluster headaches.
Oral analgesics such as codeine are generally ineffective in treating cluster headache, as are anticonvulsants and
diuretics.
References:
Campbell JK, Caselli RJ. Headache and other craniofacial pain. In: Bradley WG, Daroff RB, Fenichel GM, et al,
editors. Neurology in clinical practice. 2nd ed. Boston: Butterworth-Heinemann, 1996.
Question 32: Pharmacology/Chemistry - Cerebrovascular Disease
Discussion:
Cessation of electrical activity due to hyperpolarization that occurs acutely after an ischemic insult is the result
of activation of potassium (K+) membrane channels.
References:
Dugan LL, Kim-Han JS. Hypoxia-ischemic brain injusry and oxidative stress. In: Siegel GJ, Albers RW, Brady
ST, Price DL, editors. Basic Neurochemistry: molecular, cellular, and medical aspects. Boston: Elsevier,
2006;559-573.
Question 50: Pharmacology/Chemistry - Epilepsy
Discussion:
Praziquantel is effective in treating cerebral cysticercosis. Active uninflamed cysts are responsive. Praziquantel
will produce inflammation and edema as it kills the larva, which may temporarily result in symptoms such as
headache.
References:
Garcia HH, Evans CA, Nash TE, et al. Current consensus guidelines for treatment of neurocysticercosis. Clin
Microbiol Rev 2002;15(4):747-756.
Question 54: Pharmacology/Chemistry - Movement Disorders
75
2007 RITE Discussion & Reference Manual
Discussion:
Dyskinesias are more commonly associated with levodopa than direct dopamine agonists (DDAs).
Hallucinations more commnon with the DDAs. Nausea, orthostatic hypotension roughly equal. Peripheral
edema is not uncommon with DDAs, but rarely if ever seen with levodopa.
References:
Tintner R, Jankovic J. Dopamine agonists in Parkinson's disease. Expert Opin Investig Drugs
2003;12(11):1803-1820.
Question 61: Pharmacology/Chemistry - Movement Disorders
Discussion:
Midodrine is a prodrug that is transformed in the liver to a potent agonist of alpha receptors in arteries and veins.
Given its predictable absorption and peak effect, it is the sympathomimetic of choice for treatment of
neurogenic orthostatic hypotension. One common side effect is scalp pruritus, a manifestation of alpha
adrenergic mediated piloerection. Its most potentially serious side effect is supine hypertension. Midodrine
should not be administered late in the evening and patients taking the drug should avoid lying flat.
References:
Jankovik J, Gilden JL, Hiner BC, et al. Neurogenic orthostatic hypotension: a double-blind, placebo-controlled
study with midodrine. Am J Med 1993;95:38-48.
Bradley WG, Daroff RB, Fenichel GM, et al, editors. Neurology in clinical practice. 3rd ed. New York:
Butterworth-Heinemann, 1999.
Question 63: Pharmacology/Chemistry - Movement Disorders
Discussion:
Restless legs syndrome (RLS) is clinically defined as an urge to move the legs with or without paresthesia,
worsening of symptoms with rest and transient improvement with activity, and worsening of symptoms in the
evening and night. It is often genetic but may also occur in the setting of iron deficiency, uremia, pregnancy,
neuropathy, and possibly other conditions. Among the current treatment options offered for the treatment of
RLS, dopaminergic agents have provided the best evidence for efficacy in symptom relief.
References:
Ondo WG. Restless legs syndrome. Neurol Clin 2005;23(4):1165-1185, viii.
Chahine LM, Chemali ZN. Restless legs syndrome: a review. CNS Spectr 2006;11(7):511-520.
Question 74: Pharmacology/Chemistry - Movement Disorders
Discussion:
Iron deficiency can cause (secondary) restless legs syndrome (RLS). In addition, evidence suggests that there is
an abnormality in brain iron that somehow translates to a hypodopaminergic state in idiopathic RLS.
References:
Sun ER, Chen CA, Ho G, et al. Iron and the restless legs syndrome. Sleep 1998;21(4):371-377.
Question 87: Pharmacology/Chemistry - Other Pain Syndromes
Discussion:
Type 1 (Andrade) familial amyloidosis, inherited as an autosomal dominant, is associated with amyloid
deposition in peripheral nerves. It is characterized by progressive loss of pain and temperature sensation,
lancinating pain, and severe generalized autonomic failure. Mutations have been demonstrated in the gene that
codes for transthyretin, the protein that transports thyroxine and retinol binding protein.
76
2007 RITE Discussion & Reference Manual
codes for transthyretin, the protein that transports thyroxine and retinol binding protein.
References:
Bosch EP, Smith BE. Disorders of peripheral nerves. In: Bradley WG, Daroff RB, Fenichel GM, et al, editors.
Neurology in clinical practice. 3rd ed. Boston: Butterworth-Heinemann, 2000.
Question 88: Pharmacology/Chemistry - Neurogenetics
Discussion:
Dopa-responsive dystonia (DRD), in classic cases, manifests in early childhood with walking problems due to
dystonia of the lower limbs. The dystonia is frequently accompanied by parkinsonian features such as reduced
facial expression or slowing of fine finger movements. Biochemically, the disorder is typically characterized by
low levels of the neurotransmitter metabolite homovanillic acid and reduced levels of neopterin and
tetrahydrobiopterin (BH4) in the cerebrospinal fluid. This is due to heterozygote mutations of the GTP
cyclohydrolase I gene, which is the rate-limiting enzyme in the synthesis of BH4. BH4 is an essential cofactor
for tyrosine hydroxylase (TH), the rate-limiting enzyme in the synthesis of dopamine. Reduced levels of BH4
lead to the dopamine-deficit syndrome DRD because of reduced TH activity. Other genes implicated in the
pathogenesis of this disorder are the TH gene itself and the parkin gene.
References:
Bandmann O, Wood NW. Dopa-responsive dystonia -- the story so far. Neuropediatrics 2002;33(1):1-5.
Question 92: Pharmacology/Chemistry - Other Pain Syndromes
Discussion:
Postherpetic neuralgia is the persistence of the pain of herpes zoster virus more than 3 months after resolution
of the rash and is relatively common, affecting 10% to 15% of those with herpes zoster. Zoster-associated pain
is used to describe the continuum of pain from acute herpes zoster to the development of postherpetic neuralgia.
The time interval used in the clinical case definition of postherpetic neuralgia varies in the literature from 1 to 6
months after resolution of the rash. The incidence of postherpetic neuralgia increases with age. Administration
of antiviral agents within 72 hours of the onset of herpes zoster can reduce the intensity and duration of acute
illness and can prevent postherpetic neuralgia. Efforts at prevention of herpes zoster and postherpetic neuralgia
are important in that 40% to 50% of those with postherpetic neuralgia do not respond to any treatment.
However, tricyclic antidepressants (amitriptyline, nortriptyline, desipramine, and maprotiline), gabapentin,
pregabalin, opioids, and topical lidocaine patches are effective in reducing pain and improving quality of life and
should be used in the treatment of postherpetic neuralgia. Aspirin in cream is possibly effective in the relief of
pain in patients with postherpetic neuralgia, but the magnitude of benefit is low, as is seen with capsaicin. The
effectiveness of carbamazepine, nicardipine, biperiden, chlorprothixene, ketamine, He:Ne laser irradiation,
intralesional triamcinolone, cryocautery, topical piroxicam, extract of Ganoderma lucidum, dorsal root entry
zone lesions, and stellate ganglion block are unproven in the treatment of postherpetic neuralgia.
References:
Dubinsky R, Kabbani H, El-Chami Z, et al. Practice Parameter: Treatment of postherpetic neuralgia: An
evidence-based report of the Quality Standards Subcommittee of the American Academy of Neurology.
Neurology 2004;63:959-965.
Question 98: Pharmacology/Chemistry - Epilepsy
Discussion:
Oxcarbazepine is a derivative of carbamazepine and shares many similarities with carbamazepine, including its
mechanism of action, ability to induce hepatic metabolism of oral contraceptives and other drugs, risk of
hyponatremia, and indication for treatment of partial epilepsy. Unlike carbamazepine, oxcarbazepine is reduced
to 10-monohydroxy-carbamazepine and does not undergo oxidation to an epoxide. This may explain its fewer
side effects as compared to carbamazepine.
References:
77
2007 RITE Discussion & Reference Manual
Schmidt D, Elger CE. What is the evidence that oxcarbazepine and carbamazepine are distinctly different
antiepileptic drugs? Epilepsy Behav 2004;5(5):627-635.
Question 115: Pharmacology/Chemistry - Cerebrovascular Disease
Discussion:
The structural basis of the blood-brain barrier (BBB) is the presence of tight junctions (zonula occludens)
between capillary endothelial cells. Tight junctions consist of macromolecular complexes, including the
transmembrane proteins occludin and claudin, and the associated zonula occludens (ZO) proteins and cingulin.
References:
Huber JD, Egleton R, Davis TP. Molecular physiology of tight junctions in the blood-brain barrier. Trends
Neurosci 2001;24:719-726.
Question 117: Pharmacology/Chemistry - Neuromuscular Disorders
Discussion:
The findings are most consistent with propofol infusion syndrome. Propofol impairs mitochondrial fatty acid
oxidation and oxidative phosphorylation. Propofol raises levels of malonylcarnitine that inhibits CPT1, blocking
long chain fatty acid transport into mitochondria. Complex II function is secondarily inhibited. Children have
limited carbohydrate stores, and require larger doses of propofol for sedation, thus placing them at higher risk of
this syndrome than adults. Inadequate caloric intake before or during the infusion further increases the risk, by
leading to inability to meet metabolic demands and suppress fat metabolism. Hemofiltration has been shown to
correct the metabolic abnormalities in propofol infusion syndrome. Carbohydrate intakes of 6 mg/kg/min to 8
mg/kg/min during propofol infusion might prevent it.
References:
Wolf A, Weir P, Segar P, et al. Impaired fatty acid oxidation in propofol infusion syndrome. Lancet
2001;357:606-607.
Question 121: Pharmacology/Chemistry - Other Pain Syndromes
Discussion:
Opioid withdrawal resembles severe influenza with the additional features of mydriasis, lacrimation, rhinorrhea,
piloerection, yawning, sneezing, anorexia nausea, vomiting, and diarrhea. Delirium tremens and seizures do not
occur. Classic withdrawal techniques utilize long-acting oral agents such as methadone. Studies of aggressive
(rapid) withdrawal regimens used over 5 days have found that the least severe symptoms and shortest duration
of withdrawal was achieved with a combination of naltrexone, clonidine, and buprenorphine.
References:
Kosten TR, O'Connor PG. Management of drug and alcohol withdrawal. N Engl J Med 2003;348:1786-1795.
Question 131: Pharmacology/Chemistry - Neurogenetics
Discussion:
Autosomal dominant hypokalemic periodic paralysis typically presents in the first 3 decades with episodes of
weakness occurring following rest after exercise, particularly if the subject is exposed to high-carbohydrate
meals, emotional stress, or cold. The respiratory and cardiac muscles are almost always spared, and attacks last
for hours. Symptoms often resolve in later life, although patients may be left with residual weakness. Most
patients have mutations in the skeletal muscle calcium channel gene, although mutations may be found in
sodium and potassium channels in the remainder. Secondary hypokalemic paralysis occurs in older patients with
chronic medical problems predisposing them to hypokalemia. Preventive treatment with acetazolamide is usually
effective in abolishing or attenuating episodes, and oral potassium supplements may be helpful during acute
attacks. Some patients require the use of potassium-sparing diuretics such as spironolactone or triamterene.
Nephrolithiasis is a complication of long-term therapy with acetazolamide and should be screened for with
78
2007 RITE Discussion & Reference Manual
Nephrolithiasis is a complication of long-term therapy with acetazolamide and should be screened for with
annual ultrasound examinations.
References:
Jones HR, De Vivo DC, Daras BT, editors. Neuromuscular disorders of infancy, childhood and adolescence.
Philadelphia: Butterworth-Heinemann, 2003.
Question 136: Pharmacology/Chemistry - Dementia
Discussion:
Frontotemporal dementia (FTD) is the most common syndrome in which the focus of neurodegeneration is the
frontal lobes. FTD is frequently familial. It is also often due to a susceptibility locus on chromosome 17q21-22.
Some 17q21-22-linked families have mutations in the tau gene and most have microscopically visible aggregates
of hyperphosphorylated tau. Demonstrating that mutations in tau can produce neurodegeneration will necessitate
a reassessment of the role of tau in the pathogenesis of the many diseases in which tau biology is disrupted.
References:
Wilhelmsen KC. Frontotemporal dementia genetics. J Geriatr Psychiatry Neurol 1998; 11(2):55-60.
Question 145: Pharmacology/Chemistry - Headache
Discussion:
Chronic paroxysmal hemicrania is considered a syndrome with two pivotal characteristics: (1) unilateral
headache of moderate intensity, with episodes lasting under 45 minutes on average, occurring multiple times
throughout the day, and at a 3:1 ratio female:male; and (2) absolute response to indomethacin. Patients may also
complain of a continuous low-grade headache in between the more severe attacks (hemicrania continua).
References:
Pareja JA, Antonaci F, Vincent M. The hemicrania continua diagnosis. Cephalalgia 2001;21(10):940-946.
Question 151: Pharmacology/Chemistry - Movement Disorders
Discussion:
Anticholinergic agents such as trihexyphenidyl have been used in the treatment of Parkinson's disease (PD) since
the 19th century. Anticholinergic drugs are typically used in younger PD patients (<60 years) in whom resting
tremor is the dominant clinical feature and cognitive function is preserved. They are not effective for other
parkinsonian features such as rigidity, akinesia, gait dysfunction. Anticholinergic therapy in older patients is less
indicated due to the side effect profile (ie, dry mouth, constipation, urinary retention, confusion, tachycardia,
and close angle glaucoma) and the limited efficacy associated with these drugs. Levodopa is probably more
effective for the control of tremor and other parkinsonian features than any other drug. COMT inhibitors and
MAO-B inhibitors such as tolcapone and selegiline, and amantadine are indicated for motor fluctuations
associated with PD and to provide neuroprotection. Propranolol is most effective for essential tremor but not the
resting tremor of parkinsonism.
References:
Romrell J, Fernandez HH, Okun MS. Rationale for current therapies in Parkinson's disease. Expert Opin
Pharmacother 2003;4(10):1747-1761.
Katzenschlager R, Sampaio C, Costa J, Lees A. Anticholinergics for symptomatic management of Parkinson's
disease. Cochrane Database Syst Rev 2003;(2):CD003735.
Practice parameter: diagnosis and prognosis of new onset Parkinson disease (an evidence-based review). Report
of the Quality Standards Subcommittee of the American Academy of Neurology. Neurology 2006;66:968-975.
Question 174: Pharmacology/Chemistry - Aging, Degenerative Diseases
79
2007 RITE Discussion & Reference Manual
Discussion:
Midodrine is a prodrug that is converted in the liver to an alpha agonist. Its predictable absorption and
pharmacokinetics makes it the drug of choice for treatment of orthostatic hypotension unresponsive to
fludrocortisone.
References:
Riley DE. Orthostatic hypotension in multiple system atrophy. Curr Treat Options Neurol 2000;2(3):225-230.
Question 181: Pharmacology/Chemistry - Sleep Disorders
Discussion:
Beagles with inherited narcolepsy-cataplexy were intially shown to have defective orexin receptors. In humans,
where narcolepsy is associated with human leucocyte antigen (HLA) abnormalities, recent studies have shown
that narcolepsy with cataplexy is usually caused (>90%) by the lack of two related brain chemicals called
hypocretin-1 and hypocretin-2. The cause of narcolepsy without cataplexy is still under investigation. Over 90%
of patients with narcolepsy-cataplexy carry HLA-DQB1*0602. This marker is more specific and sensitive than
the old marker HLA-DR2, and so it is speculated that patients with narcolepsy-cataplexy probably have an
autoimmune disorder.
References:
Zeitzer JM, Nishino S, Mignot E. The neurobiology of hypocretins (orexins), narcolepsy and related therapeutic
interventions. Trends Pharmacol Sci 2006;27(7):368-374.
Question 202: Pharmacology/Chemistry - Other Pain Syndromes
Discussion:
A-delta nociceptive fibers use glutamate as their fast transmitter. C-fiber slower transmitted nociception involves
a number of neurotransmitters and neuromodulators, the most important of which is substance P.
References:
Costigan M, Scholz J, Samad T, Woolf CJ. Pain. In: Siegel GJ, Albers RW, Brady ST, Price DL, editors. Basic
Neurochemistry: molecular, cellular, and medical aspects. Boston: Elsevier, 2006;927-937.
Questions 209 - 213: Pharmacology/Chemistry - Neuromuscular Disorders
Discussion:
1. Unrecognized copper deficiency appears to be a common cause of idiopathic myelopathy in adults. The
clinical picture bears striking similarities to the syndrome of subacute combined degeneration associated with
vitamin B12 deficiency. Early recognition and copper supplementation may prevent neurological deterioration.
2. Juvenile myoclonic epilepsy is a common type of epilepsy with onset occurring during adolescence. This is a
fairly common type of epilepsy, which can develop between 8 and 26 years of age but usually starts between
ages 12 and 16. Historically, the large majority of patients become seizure-free when treated with valproate. 3.
Neuromyotonia is a rare condition of spontaneous and continuous muscle fiber activity of peripheral nerve
origin. It represents the more severe phenotype of peripheral nerve hyperexcitability and, when acquired, is often
associated with antibodies to voltage-gated potassium channels. There are no specific published
electromyographic or clinical diagnostic criteria for this disorder. Carbamazapine, topiramate, and gabapentin
have all shown efficicacy. 4. Hyperkalemic periodic paralysis involves attacks of muscle weakness or paralysis,
alternating with periods of normal muscle function. Attacks usually begin in early childhood. They tend to occur
while resting after exercise or exertion. Attacks may also be triggerd by cold expsoure or eating small amounts
of potassium. It is not associated with low potassium in the bloodstream (serum potassium). Glucose or other
carbohydrates (sugars) given during an attack may reduce the severity. Intravenous calcium or diuretics such as
furosemide may need to be given to stop sudden attacks. Intravenous glucose and insulin cause potassium to
move into the cell and may reduce weakness without a loss of total body potassium. A high-carbohydrate diet
may be recommended. Acetazolamide, a medication that prevents attacks of familial periodic paralysis, is also
effective in preventing attacks of hyperkalemic periodic paralysis. Thiazide diuretics such as chlorothiazide are
80
2007 RITE Discussion & Reference Manual
also effective and have fewer side effects than acetazolamide. 5. Tourette syndrome (TS) is a neurological
disorder characterized by repetitive, stereotyped, involuntary movements and vocalizations called tics. Tics are
classified as either simple or complex. Neuroleptics are the most consistently useful medications for tic
suppression; a number are available but some are more effective than others (for example, haloperidol and
pimozide). Alfa-adrenergic agents such as guanifencine and clonidine are first-choice treatments for TS; typical
antipsychotics are more effective but are troublesome because of their long-term side effect profiles.
References:
Verrotti A, Manco R, di Marco G, et al. The treatment of juvenile myoclonic epilepsy. Expert Rev Neurother
2006;6(6):847-854.
Jurkat-Rott K, Lerche H, Lehmann-Horn F. Skeletal muscle channelopathies. J Neurol 2002;249(11):1493-1502.
Maddison P. Neuromyotonia. Clin Neurophysiol 2006;117(10):2118-2127.
Shavitt RG, Hounie AG, Rosario Campos MC, Miguel EC. Tourette's syndrome. Psychiatr Clin North Am
2006;29(2):471-486.
Kumar N, Gross JB Jr, Ahlskog JE. Copper deficiency myelopathy produces a clinical picture like subacute
combined degeneration. Neurology 2004;63(1):33-39.
Question 319: Pharmacology/Chemistry - Movement Disorders
Discussion:
Cabergoline, pergolide, bromocriptine, and dihydroergocriptine are all ergot-derived dopamine agonists that
have been associated with serosal fibrosis syndromes. Ropinirole is a nonergoline and does not have this adverse
effect.
References:
Tintner R, Manian P, Gauthier P, Jankovic J. Pleuropulmonary fibrosis after long-term treatment with the
dopamine agonist pergolide for Parkinson disease. Arch Neurol 2005;62(8):1290-1295.
Question 336: Pharmacology/Chemistry - Epilepsy
Discussion:
Vigabatrin enhances GABAergic transmission by inhibiting GABA transaminase, thus increasing GABA
concentration at the synapse. The other agents listed inactivate sodium channels.
References:
Mcnamara JO. Drugs effective in the therapy of the epilepsies. In: Hardman JG, Limbierd LE, Goodman AG,
editors. Goodman and Gilman's the pharmacological basis of therapeutics. 10th ed. New York: McGraw-Hill,
2001;521-547.
Question 340: Pharmacology/Chemistry - Neurogenetics
Discussion:
Familial periodic paralyses are typical channelopathies (ie, caused by functional disturbances of ion channel
proteins). The episodes of flaccid muscle weakness observed in these disorders are due to underexcitability of
sarcolemma leading to a silent electromyogram and the lack of action potentials even upon electrical
stimulation. Interictally, ion channel malfunction is well compensated so that special exogenous or endogenous
triggers are required to produce symptoms in the patients. An especially obvious trigger is the level of serum
potassium (K+), the ion responsible for resting membrane potential and degree of excitability. The clinical
symptoms can be caused by mutations in genes coding for ion channels that mediate different functions for
maintaining the resting potential or propagating the action potential, the basis of excitability. The phenotype is
determined by the type of functional defect brought about by the mutations rather than the channel effected
because the contrary phenotypes hyperkalemic periodic paralysis (HyperPP) and hypokalemic periodic paralysis
(HypoPP) may be caused by point mutations in the same gene. Still, the common mechanism for inexcitability
81
2007 RITE Discussion & Reference Manual
in all known episodic-weakness phenotypes is a long-lasting depolarization that inactivates sodium ion (Na+)
channels, initiating the action potential.
References:
Jurkat-Rott K, Lerche H, Lehmann-Horn F. Skeletal muscle channelopathies. J Neurol 2002;249(11):1493-1502.
Question 348: Pharmacology/Chemistry - Movement Disorders
Discussion:
Restless legs syndrome (RLS) is clinically defined as an urge to move the legs with or without paresthesia,
worsening of symptoms with rest and transient improvement with activity, and worsening of symptoms in the
evening and night. It is often genetic but may also occur in the setting of iron deficiency, uremia, pregnancy,
neuropathy, and possibly other conditions. Effective treatments include dopaminergics and narcotics. Its
underlying pathogenesis is presently unknown. Women are more affected than men and early-onset disease is
associated with familial cases. The excellent response to dopaminegic drugs points to a central role of dopamine
in the pathophysiology of RLS. Iron may also represent a primary factor in the development of RLS, as
suggested by recent pathological and brain imaging studies. However, the way dopamine and iron, and probably
other compounds, interact to generate the circadian pattern in the occurrence of RLS and periodic limb
movement symptoms remains unknown. Altered circadian rhythmicity in dopamine metabolism and enhanced
circadian variations in dopaminergic functions have been reported in the disorder. Dysfunction or atrophy of
cells from the diencephalic-spinal dopamine system has been suggested to explain the efficacy of dopaminergic
drugs in relieving RLS symptoms and the circadian rhythmicity of RLS. Studies support the hypothesis that
these dopaminergic neurons and spinal pathways may be more involved in the pathophysiology of RLS than the
nigrostriatal system. Among the current treatment options offered for the treatment of RLS, dopaminergic agents
have provided the best evidence for efficacy in symptom relief.
References:
Ondo WG. Restless legs syndrome. Neurol Clin 2005;23(4):1165-1185, viii.
Clemens S, Rye D, Hochman S. Restless legs syndrome: revisiting the dopamine hypothesis from the spinal
cord perspective. Neurology 2006 Jul 11;67(1):125-130.
Barriere G, Cazalets JR, Bioulac B, et al. The restless legs syndrome. Prog Neurobiol 2005;77(3):139-165.
Question 352: Pharmacology/Chemistry - Epilepsy
Discussion:
Drugs that inhibit carbamazepine metabolism results in elevated levels and toxicity. These include phenytoin,
cimetidine, diltiazem, erythromycin, verapamil, fluoxetine, and isoniazid. Alternately, carbamazapine can
accelerate hepatic breakdown of a number of drugs, including its own metabolism. The most common
interaction is with oral contraceptives, sodium valproate, ethosuximide, corticosteroids, anticoagulants,
antipsychotics, cyclosporine, and methylphenidate. Meperidine and methylphenyldate can lower the seizure
threshold in epilepsy patients resulting in worsening seizures independent of antiepileptic therapy.
References:
Ruffmann C, Bogliun G, Beghi E. Epileptogenic drugs: a systematic review. Expert Rev Neurother
2006;6(4):575-589.
Pauwels O. Factors contributing to carbamazepine-macrolide interactions. Pharmacol Res 2002;45(4):291-298.
Question 353: Pharmacology/Chemistry - Demyelinating Disorders
Discussion:
Interferon-beta-1b (Betaseron, Betaferon) is a nonglycosylated recombinant human interferon-beta approved for
high-frequency subcutaneous (SC) administration in the treatment of multiple sclerosis (MS). Its mechanism of
action is unknown but may involve modulation of the autoimmune pathogenic processes of MS. In a
randomized, double-blind trial in patients with relapsing-remitting MS (RRMS), SC interferon-beta-1b 250
82
2007 RITE Discussion & Reference Manual
micrograms (8 million International Units [MIU]) every other day reduced the annual relapse rate and increased
the proportion of relapse-free patients compared with placebo. It also reduced relapse severity, hospitalizations,
and disease activity assessed by MRI and increased the time to first relapse.
References:
Jacobs LD, Beck RW, Simon JH, et al. Intramuscular interferon beta-1a therapy initiated during a first
demyelinating event in multiple sclerosis. CHAMPS Study Group. N Eng J Med 2000;343(13):898-904.
Question 367: Pharmacology/Chemistry - Dementia
Discussion:
Loss or dysfunction of the acetylcholine projection to the cerebral cortex has been shown experimentally in
humans and animals to result in cognitive disturbance. Loss of neurons from the nucleus Basalis of Meynert was
shown early on to characterize patients with Alzheimer's disease. Recent studies however have shown that the
depletion of cortical acetylcholine has been significantly more severe in patients with dementia with Lewy
bodies that even in Alzheimer's disease. Neither fund a temporal dimension or Huntington's disease has
significant loss of cortical cholinergic projections, at least in early to moderate stages. Vascular dementia shows
variable loss, depending on where the vascular lesions are; in addition, many of these patients have mixed
Alzheimer's/vascular pictures.
References:
Samuel W, Alford M, Hofstetter CR, Hansen L. Dementia with Lewy bodies versus pure Alzheimer disease:
differences in cognition, neuropathology, cholinergic dysfunction, and synapse density. J Neuropathol Exp
Neurol 1997;56(5):499-508.
Question 380: Pharmacology/Chemistry - Aging, Degenerative Diseases
Discussion:
Amyotrophic lateral sclerosis (ALS) is a chronic neurodegenerative disease of upper and lower motor neurons.
Clinically, patients present with skeletal muscle weakness, muscle atrophy, and hyperreflexia. Symptoms
progress to death in 50% of those affected within 3 to 5 years after symptom onset. Death is usually related to
respiratory failure. Ten percent of cases are inherited and 20% of these are linked to mutations of the superoxide
dismutase 1 gene.
References:
Miller RG, Rosenberg JA, Gelinas DF, et al. Practice Parameter: The care of the patient with amyotrophic lateral
sclerosis (an evidence-based revew): Report of the Quality Standards Subcommittee of the American Academy
of Neurology. Neurology 1999;52:1311.
Question 394: Pharmacology/Chemistry - Epilepsy
Discussion:
Patients with newly diagnosed epilepsy who require treatment can be initiated on standard antiepileptic drugs
(AEDs), such as carbamazepine, phenytoin, valproic acid, phenobarbital, or on the new AEDs, lamotrigine,
gabapentin, oxcarbazepine, or topiramate. The choice of the AED should depend on the characteristics of the
patient, including potential interaction with concomitant medications. Carbamazapine, topiramate, valproic acid,
gabapentin, and lamotrogine have reported efficacy as monotherapy or as adjunct therapy for newly diagnosed
epilepsy. In addition, studies have demonstrated that carbamazepine, valproate, and lamotrigine are also
effective in the treatment of acute mania and suggest efficacy as maintenance therapy in bipolar disorder. Recent
studies do not support the efficacy of topiramate as monotherapy or adjunct therapy of acute mania or mixed
episodes in adults with bipolar I disorder. Topiramate has also been associated with changes in serum lithium
levels and lithium toxicity and therefore would not be indicated for primary treatment of epilepsy in bipolar
patients well controlled on lithium.
References:
Gajwani P, Forsthoff A, Muzina D, et al. Antiepileptic drugs in mood-disordered patients. Epilepsia
83
2007 RITE Discussion & Reference Manual
Gajwani P, Forsthoff A, Muzina D, et al. Antiepileptic drugs in mood-disordered patients. Epilepsia
2005;46(Suppl 4):38-44.
Abraham G, Owen J. Topiramate can cause lithium toxicity. J Clin Psychopharmacol 2004;24(5):565-567.
Vasudev K, Macritchie K, Geddes J, et al. Topiramate for acute affective episodes in bipolar disorder. Cochrane
Database Syst Rev 2006 Jan 25;(1):CD003384.
Question 399: Pharmacology/Chemistry - Neurogenetics
Discussion:
Dystrophin is a membrane-bound protein distributed along the intracellular surface of the sarcolemma and is a
member of the superfamily of cytoskeletal proteins; dystrophin interacts with actin and may contribute to
structural and functional stability of the plasma membrane.
References:
Jones HR, De Vivo DC, Darras BT, editors. Neuromuscular disorders of infancy, childhood and adolescence.
Philadelphia: Butterworth-Heinemann, 2003.
Question 400: Pharmacology/Chemistry - Epilepsy
Discussion:
Valproate can produce hair loss that rarely requires discontinuation of the drug.
References:
Holland KD. Efficacy, pharmacology, and adverse effects of antiepileptic drugs. Neurol Clin 2001;19:313-345.
Question 407: Pharmacology/Chemistry - Headache
Discussion:
For migraine prevention, individual medications have been put into treatment groups based on their established
clinical efficacy, significant adverse events, safety profile, and clinical experience of the US Headache
Consortium participants: Group 1 medications (amitryptiline, divalproex sodium, propranolol/timolol) have
proven high efficacy and mild to moderate adverse events. Group 2 medications (verapamil, nonsteroidal
anti-inflammatory drugs, gabapentin, vitamin B2) with lower efficacy (ie, limited number of studies, studies
reporting conflicting results, efficacy suggesting only “modest” improvement) and mild to moderate adverse
events. Group 3 medication (topiramate, diltiazam, antidepressants) are used based on opinion, not randomized
controlled trials, and may have low to moderate or frequent or severe adverse events. Group 4 medication
(methysergide) has shown proven efficacy but frequent or severe adverse events (or safety concerns) or complex
management issues. Group 5 (tegretol, clonidine, clonazepam) medications are proven to have limited or no
efficacy.
References:
Silberstein S. Practice parameter: Evidence-based guidelines for migraine headache (an evidence-based review).
Report of the Quality Standards Subcommittee of the American Academy of Neurology. Neurology 2000; 55:
754-763.
Question 422: Pharmacology/Chemistry - Epilepsy
Discussion:
Mutations of the FIL-1 gene in chromosome Xq28, encoding for filamin-1, produce periventricular heterotopy.
This reflects a complete failure of neuronal migration that may cause seizures. It is dominant in females and
lethal in males. Filamin-1 interacts with actin and is critically involved in the early phases of migration.
References:
Gleeson JG, Walsh CA. Neuronal migration disorders: from genetic disease to developmental mechanisms.
84
2007 RITE Discussion & Reference Manual
Trends Neurosci 2000; 22:402-410.
Question 435: Pharmacology/Chemistry - Epilepsy
Discussion:
Pseudoseizures represent the opposite end of the spectrum from seizures that mimic psychiatric disorders: they
are paroxysmal changes in behavior that resemble epileptic seizures but are without organic cause and expected
EEG changes. Accurately distinguishing pseudoseizures from epilepsy and other illnesses is difficult because of
the breadth and overlap of symptoms seen in each condition and because of the frequent co-occurrence of
pseudoseizures and epilepsy. For the assessment of true seizure versus pseudoseizure, EEG is indicated in the
peri-ictal evaluation. Emergent or urgent neuroimaging should be considered if there is a new seizure pattern or
new seizure type or prolonged postictal confusion or worsening mental status. Elevated serum prolactin assay,
when measured in the appropriate clinical setting at 10 to 20 minutes after a suspected event, is a useful adjunct
for the differentiation of generalized tonic–clonic or complex partial seizure from psychogenic nonepileptic
seizure among adults and older children. However, the use of serum prolactin assay has not been established in
the evaluation of status epilepticus, repetitive seizures, and neonatal seizures.
References:
Chen D, So Y, Fisher R. The use of serum prolactin level in diagnosisng epileptic seizures. Report of the
Therapeutics and Technology Assessment Subcommittee of the American Academy of Neurology. Neurology
2005;65:668-675.
Physiology
Question 24: Physiology - EMG
Discussion:
Common peroneal nerve divides into superficial and deep branches. Injury to the deep branch weakens the toe
and foot dorsiflexors, with sensory changes over the web of the skin between the first and second toe.Lesion of
the superficial branch affects the everters, with sensory deficit over most of the dorsum of the foot. Injury to the
femoral nerve weakens the quadriceps muscle. Injury to the obturator nerve weakens the adductors, internal and
external rotators of the thigh. Injury to the tibial nerve weakens the foot inverters, dorsiflexors and intrinsic foot
muscles.
References:
Kimura J. Electrodiagnosis in diseases of nerve and muscle: principles and practice. 2nd ed. Philadelphia: Davis,
1989;507-510.
Question 28: Physiology - EEG
Discussion:
Although periodic waveforms can be seen in other conditions, the presence of generalized periodic 1 Hz sharp
waves with progressive dementia is strongly suggestive of Creutzfeldt-Jakob disease.
References:
Niedermeyer E, Lopes da Silva F, editors. Electroencephalography: basic principles, clinical applications, and
related fields. 4th ed. Philadelphia: Lippincott, Williams & Wilkins, 1998.
Question 29: Physiology - EEG
Discussion:
Drowsiness in a normal adult will slow background rhythm on the EEG to theta activity and sometimes bursts of
generalized moderate-to-high amplitude 5 Hz to 7Hz theta can be seen. Frontocentral beta activity will increase
in prominence during drowsiness (16 Hz to 20 Hz) and occasionally bursts of faster activity can be seen.
85
2007 RITE Discussion & Reference Manual
References:
Daly DD, Pedley TA. Current practice of clinical electroencephalography. 2nd ed. New York: Raven Press,
1990.
Question 30: Physiology - Autonomic Studies
Discussion:
The Valsalva maneuver evaluates the baroreflex arc and its sympathetic and parasympathetic responses. The
maneuver consists of four phases. In phase 4, there is overshoot of the blood pressure due to the persistent
increase in peripheral resistance, normalization of venous return, and stroke volume. The mean blood pressure
can increase by more than 10 mm Hg. If the blood pressure does not increase, this indicates dysfunction of the
sympathetic response.
References:
Hilz MJ, Dutsch M. Quantitative studies of autonomic function. Muscle Nerve 2006;33:6-20.
Question 34: Physiology - EEG
Discussion:
The presence of periodic lateralized sharp wave discharges in a patient with a recent febrile illness and onset of
seizures would strongly suggest herpes simplex encephalitis.
References:
Niedermeyer E, Lopes da Silva F, editors. Electroencephalography: basic principles, clinical applications, and
related fields. 4th ed. Philadelphia: Lippincott, Williams & Wilkins, 1998.
Question 36: Physiology - EMG
Discussion:
Early signs of Guillain-Barré syndrome include diminished or lost muscle stretch flexes. Electrophysiological
studies reveal slowing of peripheral conduction velocity as well as increased central conduction time, including
prolonged F-waves and distal motor latencies. Interestingly, the disease tends to spare the sural nerve sensory
action potential, often regarded as one of the first affected in other neuropathies. Reduction of amplitude of
compound muscle action potentials with distal stimulation implies a poor prognosis.
References:
Kimura J. Electrodiagnosis in diseases of nerve and muscle: principles and practice. 2nd ed. Philadelphia: Davis,
1989;507-510.
Murray NMF, Wade DT. The sural sensory action potential in Gullain-Barré syndrome. Muscle Nerve
1980;3:444.
Question 46: Physiology - EEG
Discussion:
Long-term use of chlorpromazine has been reported to lead to generalized paroxysmal bursts in EEGs.
References:
Engel J, Pedley TA, editors. Epilepsy—a comprehensive textbook. Philadelphia: Lippincott-Raven, 1998.
Question 52: Physiology - EMG
Discussion:
Lesions of the upper trunk of the brachial plexus will cause weakness in proximal upper extremity with
86
2007 RITE Discussion & Reference Manual
abnormal SNAPs on NCS in the radial and median nerves. The deltoid muscle is innervated by the axillary
nerve which arises from the upper trunk/posterior cord. The biceps muscle is innervated by the
musculocutaneous nerve which arises from the upper trunk/lateral cord.
References:
Kimura J. Electrodiagnosis in disease of nerve and muscle. 3rd ed. New York: Oxford University Press, 2001.
Question 55: Physiology - EMG
Discussion:
Increased jitter is seen in a variety of neuromuscular disorders and is not specific to any particular disease. Jitter
is not increased in metabolic myopathies and steroid myopathies.
References:
Stalberg E, Trontelj JV. Single fiber electromyography. 2nd ed. New York: Raven Press, 1994.
Question 57: Physiology - EMG
Discussion:
Myotonic discharges may be present as spontaneous activity on EMG in myopathies. End plate spikes originate
from the motor end plate. Fasciculations, myokymia and neuromyotonia all originate from the motor axon.
References:
Preston DC, Shapiro BE. Electromyography and neuromuscular disorders: clinical-electrophysiologic
correlations. 2nd ed. Philadelphia: Elsevier, 2005.
Question 60: Physiology - EMG
Discussion:
Fascioscapulohumeral dystrophy patients have weakness of the facial muscles, shoulder and upper arm muscles
(sparing deltoids usually), and foot dorsiflexors, frequently with asymmetry. EMG is not specific but can often
show fibrillation potentials and short-duration, low amplitude, polyphasic motor unit action potentials.
References:
Kimura J. Electrodiagnosis in disease of nerve and muscle. 3rd ed. New York: Oxford University Press, 2001.
Question 76: Physiology - EMG
Discussion:
In a postsynaptic disorder (ie, myasthenia gravis), rapid repetitive stimulation causes no change in the
compound muscle action potential. There would typically be a >10% decrement seen on slow repetitive
stimulation. A significant increment on rapid repetitive stimulation is typically seen in presynaptic disorders
(Lambert-Eaton myasthenic syndrome, botulism).
References:
Katirji B, Kaminski HJ. Electrodiagnostic approach to the patient with suspected neuromuscular junction
disorder. Neurol Clin N Am 2002;20:557-586.
Question 78: Physiology - EEG
Discussion:
Generalized beta activity (activity over 14 Hz) can be seen as a consequence of drug use, particularly
benzodiazepines or barbiturates.
References:
87
2007 RITE Discussion & Reference Manual
References:
Niedermeyer E, Lopes da Silva F, editors. Electroencephalography: basic principles, clinical applications, and
related fields. 4th ed. Philadelphia: Lippincott, Williams & Wilkins, 1998.
Question 82: Physiology - EMG
Discussion:
In compression neuropathies, sensory fibers are usually affected first with conduction changes. The median
palmar sensory latency prolongation is the earliest change seen on EMG/nerve conduction studies in patients
with mild carpal tunnel syndrome.
References:
Kimura J. Electrodiagnosis in disease of nerve and muscle. 3rd ed. New York: Oxford University Press, 2001.
Question 85: Physiology - EEG
Discussion:
Favorable prognostic factors on EEG are variability, reactivity to external stimuli, varying sleep patterns,
increase in background frequencies. Poor prognostic factors are invariant pattern, no reactivity, monorthythmic
pattern, burst suppression, generalized periodic discharges, very low voltage tracing, and generalized
suppression. Over 96% of the patients with poor prognostic findings on the EEG following a cardiac arrest
either die within a few days after the cardiopulmonary arrest or if they survive, do so in a persistent vegetative
state.
References:
Daube JR, editor. Clinical neurophysiology. New York: Oxford University Press, 2002;98-101.
Daube JR, editor. Clinical neurophysiology. Continuum: Lifelong Learning in Neurology 1998:4(5);41.
Question 91: Physiology - Sleep
Discussion:
The goal of the multiple sleep latency test is to quantitate physiological sleepiness during waking hours and to
determine the occurrence of REM sleep near sleep onset. The goal of polysomnography is to quantitate the
amount of time spent in various stages of sleep during the night and to document clinically relevant events such
as cardiopulmonary abnormalities or sleep-related abnormal motor activity. Electro-oculogram, surface
electromyography of all four limbs, and EEG recording are all used while performing polysomnography and
multiple sleep latency test.
References:
Daube JR, editor. Clinical neurophysiology. New York: Oxford University Press, 2002;415-417.
Question 94: Physiology - EMG
Discussion:
Preganglionic separation of the cell bodies with lesions at the root level preserves the anatomic and physiologic
integrity of the peripheral axon. Despite sensory loss, nerve stimulation elicits a normal sensory action potential.
A preganglionic involvement spares the sensory nerve action potential, although degeneration of the motor
axons leads to muscle atrophy and reduction in amplitude of compound muscle potentials. The deep cervical
muscles receive innervation from the posterior, as opposed to anterior rami of the spinal nerves. Evidence of
denervation here indicates an intraforaminal lesion affecting the root or spinal nerve prior to the division into
the two rami.
References:
Kimura, J. Electrodiagnosis in diseases of nerve and muscle: principles and practice. 2nd ed. Philadelphia:
88
2007 RITE Discussion & Reference Manual
Davis, 1989;448-449.
Question 97: Physiology - EMG
Discussion:
Femoral nerve innervates the hip flexors and knee extensor muscles.
References:
Kimura J. Electrodiagnosis in disease of nerve and muscle. 3rd ed. New York: Oxford University Press, 2001.
Question 101: Physiology - EEG
Discussion:
In elderly patients in coma, EEG showing diffuse spike-and-wave discharges is most consistent with
nonconvulsive status epilepticus. Barbiturate overdose will show excessive beta activity on EEG. Alpha coma
will show diffuse nonreactive alpha rhythm. Metabolic encephalopathy usually shows diffuse slowing.
Pseudoseizures would have a normal EEG pattern.
References:
Niedermeyer E, Lopes da Silva F, editors. Electroencephalography: basic principles, clinical applications, and
related fields. 4th ed. Philadelphia: Lippincott, Williams & Wilkins, 1998.
Question 103: Physiology - EEG
Discussion:
Alzheimer’s disease is associated with a decrease or loss of alpha and beta activity at an earlier stage than other
disorders associated with dementia. Generalized periodic sharp waves and invariant alpha pattern are poor
prognostic indicators on EEG and are usually seen after cardiopulmonary arrest. Triphasic waves are seen in
50% of patients with hepatic coma. Excessive beta activity may been seen due to medications like
benzodiazepines or barbiturates.
References:
Daube JR, editor. Clinical neurophysiology. New York: Oxford University Press, 2002;159-160.
Question 106: Physiology - EMG
Discussion:
Isolated visible muscle twitching for several years with no other concurrent symptoms is most consistent with
fasciculation potentials as seen in benign fasciculation syndrome.
References:
Kimura J. Electrodiagnosis in disease of nerve and muscle. 3rd ed. New York: Oxford University Press, 2001.
Question 107: Physiology - EMG
Discussion:
The H-reflex is the electrophysiological equivalent of the monosynaptic tendon stretch reflex at the ankle.
References:
Kimura J. Electrodiagnosis in disease of nerve and muscle. 3rd ed. New York: Oxford University Press, 2001.
Question 109: Physiology - EMG
Discussion:
89
2007 RITE Discussion & Reference Manual
Polio is a motor neuron disease. Adults who had polio when they were younger may have large polyphasic
motor unit action potentials, due to reinnervation. They may also have some fibrillation and fasciculation
potentials.
References:
Kimura J. Electrodiagnosis in disease of nerve and muscle. 3rd ed. New York: Oxford University Press, 2001.
Question 119: Physiology - EMG
Discussion:
The patient's examination localizes to the right L5 root. Both the gluteus medius and peroneus longus muscles
receive their major innervation from the L5 nerve root.
References:
Brown WF, Bolton CF, editors. Clinical electromyography. 2nd ed. Boston: Butterworth-Heinemann, 1993.
Question 132: Physiology - EMG
Discussion:
Inclusion body myositis is an inflammatory myopathy with common clinical presentation of distal upper
extremity and proximal lower extremity weakness. It occurs more frequently in older patients. EMG shows
diffuse irritability and fibrillation potentials, as in amyotrophic lateral sclerosis, but the motor unit action
potentials are small and polyphasic.
References:
Day JW, Ranum LPW. Myotonic disorders. In: Katirji B, Kaminski HJ, Preston DC, et al, editors.
Neuromuscular disorders in clinical practice. Woburn, Mass: Butterworth-Heinemann, 2002;1078-1091.
Question 148: Physiology - EMG
Discussion:
The short head of the biceps femoris muscle is the only muscle proximal to the knee to receive innervation via
the peroneal division of the sciatic nerve.
References:
Kimura J. Electrodiagnosis in disease of nerve and muscle. 3rd ed. New York: Oxford University Press, 2001.
Question 150: Physiology - EEG
Discussion:
Light sensitivity manifested by photomyoclonus induced by photic stimulation can occur with abrupt alcohol
withdrawal.
References:
Niedermeyer E, Lopes da Silva F. Electroencephalography. 4th ed. Baltimore: Lippincott, Williams & Wilkins,
1998.
Question 159: Physiology - EEG
Discussion:
The pattern most commonly seen in children between the ages of 3 and 10 who have absence seizures is
generalized 3 Hz spike-and-wave discharges.
References:
90
2007 RITE Discussion & Reference Manual
Pedley TA, Mendiratta A, Walczak TS. Seizures and epilepsy. In: Ebersole JS, Pedley TA, editors. Current
practice of clinical electroencephalography. 3rd ed. Philadelphia: Lippincott, Williams & Wilkins,
2003;506-587.
Question 160: Physiology - EMG
Discussion:
First dorsal interosseus muscle is innervated by the ulnar nerve and C8 and T1 nerve roots.
References:
Kimura J. Electrodiagnosis in disease of nerve and muscle. 3rd ed. New York: Oxford University Press, 2001.
Question 162: Physiology - EMG
Discussion:
Hereditary neuropathy with liability to pressure palsies may present in childhood. Other family members may or
may not be symptomatic. Electrophysiologic studies reveal conduction blocks at sites of pressure and mild
diffuse slowing.
References:
Jones HR, Bolton CF, Harper CM. Pediatric clinical electromyography. Philadelphia: Lippincott, Wiliams &
Wilkins, 1996.
Question 165: Physiology - EEG
Discussion:
Lennox-Gastaut syndrome is characterized clinically by frequent generalized seizures of mixed type in
association with mental retardation and a slow spike-and-wave EEG pattern with increasing disorganization
during sleep.
References:
Niedermeyer E, Lopes da Silva F. Electroencephalography. 4th ed. Baltimore: Lippincott, Williams & Wilkins,
1998.
Question 168: Physiology - Basic Physiology
Discussion:
Depolarization of action potentials causes an initial negative departure from baseline due to rapid opening of
voltage-sensitive sodium channels. Closing of these channels due to an intrinsic inactivation time constant as
well as slow opening of voltage-sensitive potassium channels results in repolarization of the wave.
References:
Daube JR, editor. Clinical neurophysiology. Philadelphia: FA Davis, 1996;81-84.
Question 186: Physiology - EMG
Discussion:
The extensor indicis proprius (EIP) is innervated by the posterior interosseous nerve. This is derived from the
posterior cord, lower trunk of the brachial plexus. The ulnar and median innervated muscles of the hand are
supplied by the lower trunk, medial cord of the brachial plexus. Thus, if the EIP is involved, then the medial
cord could not be the primary site of the lesion.
References:
Preston DC, Shapiro BE. Electromyography and neuromuscular disorders: clinical-electrophysiologic
91
2007 RITE Discussion & Reference Manual
Preston DC, Shapiro BE. Electromyography and neuromuscular disorders: clinical-electrophysiologic
correlations. 2nd ed. Philadelphia: Elsevier, 2005; 59-64.
Question 193: Physiology - EEG
Discussion:
Alpha rhythm is normal background rhythm seen in posterior head regions in adults. It ranges from 8 Hz to 13
Hz.
References:
Daly DD, Pedley TA. Current practice of clinical electroencephalography. 2nd ed. New York: Raven Press,
1990.
Question 199: Physiology - EEG
Discussion:
Vertex or V-waves are high-voltage sharp-contoured waveforms that can occur with phase reversals on a bipolar
montage over the central areas.
References:
Klass DW, Westmoreland BF. Electroencephalography: general principles and adult electroencephalograms. In:
Daube JR, editor. Clinical Neurophysiology. Philadelphia: FA Davis, 1996.
Question 203: Physiology - EEG
Discussion:
Photic stimulation is performed to assess for EEG evidence of susceptibility to photosensitive seizures, which is
manifested as an epileptiform response, and for the occipital driving response.
References:
Fisch BJ, So EL. Activation methods. In: Ebersole JS, Pedley TA, editors. Current practice of clinical
electroencephalography. 3rd ed. Philadelphia: Lippincott, Williams & Wilkins, 2003;246-270.
Question 234: Physiology - EEG
Discussion:
Focal polymorphic delta is seen with hemispheric lesions such as infarct or tumor and is nonspecific as to
etiology. It generally is seen with processes involving white matter. Subdural hematoma will typically produce
attenuation of EEG amplitude in the region of the lesion.
References:
Klass DW, Westmoreland BF. Electroencephalography: general principles and adult electroencephalograms. In:
Daube JR, editor. Clinical neurophysiology. Philadelphia: FA Davis, 1996.
Question 257: Physiology - EEG
Discussion:
The EEG shows typical centrotemporal spikes, often associated with benign rolandic epilepsy of childhood,
which is manifested by clonic movements of the face and hand that often progress to a more generalized seizure.
References:
Niedermeyer E, Lopes da Silva F. Electroencephalography. 4th ed. Baltimore: Lippincott, Williams & Wilkins,
1998.
92
2007 RITE Discussion & Reference Manual
Question 266: Physiology - EEG
Discussion:
Anterior eyeblinks (first second of the page), facial muscle EMG artifact (throughout the page), and an occipital
dominant alpha rhythm (throughout the page) are characteristic of wakefulness.
References:
Blume WT, Kaibara M. Atlas of adult electroencephalography. New York: Raven Press, 1995.
Question 267: Physiology - EEG
Discussion:
An EEG showing triphasic waves is consistent with a metabolic encephalopathy, most commonly hepatic
encephalopathy.
References:
Niedermeyer E, Lopes da Silva F, editors. Electroencephalography: basic principles, clinical applications, and
related fields. 4th ed. Philadelphia: Lippincott, Williams & Wilkins, 1998.
Question 295: Physiology - EEG
Discussion:
The EEG was recorded in drowsiness and shows positive sharp waves in occipital leads. These are positive
occipital sharp transients and are normal phenomenon in sleep.
References:
Daly DD, Pedley TA. Current practice of clinical electroencephalography. 2nd ed. New York: Raven Press,
1990.
Question 304: Physiology - EEG
Discussion:
Subacute sclerosing panencephalitis is associated with periodic long-interval diffuse discharges in the EEG that
recur every 4 to 15 seconds.
References:
Klass DW, Westmoreland BF. Electroencephalography: principles and adult electroencephalograms. In: Daube
JR, editor. Clinical neurophysiology. Philadelphia: FA Davis, 1996.
Question 306: Physiology - EEG
Discussion:
This EEG shows generalized polyspikes and spike-wave discharges, the EEG correlate of primary generalized
epilepsy syndromes, which are variously characterized by generalized tonic-clonic seizures, generalized
myoclonic seizures, or absence seizures.
References:
Pedley TA, Mendiratta A, Walczak TS. Seizures and epilepsy. In: Ebersole JS, Pedley TA, editors. Current
practice of clinical electroencephalography. 3rd ed. Philadelphia: Lippincott, Williams & Wilkins,
2003;506-587.
Question 311: Physiology - Sleep
Discussion:
93
2007 RITE Discussion & Reference Manual
Stage REM is identified by relatively mixed frequency EEG and the presence of rapid eye movements. In some
subjects, a characteristic sawtooth pattern is intermittently observed in the EEG. EMG drops to the lowest level
of recording.
References:
Rechtschaffen A, Kales A. Manual of standardized terminology, techniques and scoring systems for sleep stages
of human subjects. Los Angeles: UCLA Brain Information Service/Brain Research Institute, 1968.
Question 312: Physiology - Sleep
Discussion:
The minimum duration required for a respiratory event to be called as obstructive apnea or hypopnea is 10
seconds in an adult.
References:
Pack AI. Advances in sleep-disordered breathing. Am J Respir Crit Care Med 2006;173:7-15.
Chesson A, Ferber R, Fry J, et al. Practice parameters for the indications for polysomnography and related
procedures. Sleep 1997;20:406.
Question 313: Physiology - Sleep
Discussion:
While managing a patient with obstructive sleep apnea, current symptoms, especially daytime sleepiness, AHI,
and co-morbid cardio/cerebro-vascular, metabolic, and pulmonary disorders should be taken into consideration.
As indicated by the AHI, he has moderate obstructive sleep apnea (OSA), co-morbid poorly controlled
hypertension and diabetes with significant daytime sleepiness. Therefore, his OSA should be aggressively
managed to decrease the daytime sleepiness, and possibly better control the hypertension and diabetes. Among
the options listed, CPAP if used appropriately, is 100 % effective and thus would be the best option. Surgery
has a success rate of ~ 50% with a high relapse rate. A dental appliance is good for only mild obstructive sleep
apnea. Weight loss should be encouraged in all sleep apnea patients with BMI > 25; however, it is not helpful by
itself. Hypnotic use by itself in moderate to severe obstructive sleep apnea might worsen sleep apnea and thus is
not the best option.
References:
Pack AI. Advances in sleep-disordered breathing. Am J Respir Crit Care Med 2006;173:7-15.
Chesson A, Ferber R, Fry J, et al. Practice parameters for the indications for polysomnography and related
procedures. Sleep 1997;20:406.
Question 321: Physiology - EEG
Discussion:
Frontal intermittent rhythmic delta activity can be seen with a variety of lesions including posterior fossa lesions,
encephalopathy, intracranial lesions, and increased intraventricular pressure. REM sleep is associated with a
low-voltage desynchronized EEG.
References:
Niedermeyer E, Lopes da Silva F. Electroencephalography. 3rd ed. Baltimore: Williams & Wilkins, 1993.
Question 322: Physiology - EEG
Discussion:
Electrocerebral inactivity is defined as no EEG activity over 2 microvolts.
References:
94
2007 RITE Discussion & Reference Manual
Daube JR, editor. Clinical neurophysiology. Philadelphia: FA Davis, 1996;81-84.
Question 325: Physiology - EEG
Discussion:
Normal variants during drowsiness and sleep include 14 Hz and 6 Hz positive waves, small sharp spikes, wicket
spikes, 6 Hz spike-and-wave, and rhythmic temporal theta. Subclinical rhythmic electrographic discharges of
adults (SREDA) is a normal variant during wakefulness.
References:
Daube JR, editor. Clinical neurophysiology. Philadelphia: FA Davis, 1996;81-84.
Question 326: Physiology - Basic Physiology
Discussion:
Tetanus toxin targets synaptobrevins in the presynaptic nerve terminal, blocking docking of the vesicle and
release of neurotransmitters. Alpha-bungarotoxin and curare bind to the nicotinic AChR in muscle. Brevetoxin
primarily causes prolonged sodium channel opening. Tetrodotoxin blocks sodium channels.
References:
Day JW, Ranum LPW. Myotonic disorders. In: Katirji B, Kaminski HJ, Preston DC, et al, editors.
Neuromuscular disorders in clinical practice. Woburn, Mass: Butterworth-Heinemann, 2002;1078-1091.
Kandel ER, Schwartz JH, Jessel TM. Principles of neural science. 4th ed. New York: McGraw-Hill, 2000.
Question 327: Physiology - EMG
Discussion:
Both Guillain-Barré syndrome (GBS) and chronic inflammatory demyelinating polyneuropathy (CIDP) are
acquired demyelinating autoimmune neuropathies. The CSF and electrodiagnostic findings are quite similar, and
cannot distinguish one from the other. The interval between onset of symptoms and disease plateau is the main
distinguishing feature, with the vast majority of GBS patients reaching a plateau within four weeks of disease
onset.
References:
Kimura J. Electrodiagnosis in disease of nerve and muscle. 3rd ed. New York: Oxford University Press, 2001.
Question 344: Physiology - EMG
Discussion:
An incremental response to tetanic stimulation indicates a presynaptic defect at the neuromuscular junction. In
an infant, the most likely reason for developing this is from intestinal botulism.
References:
Jones HR, Bolton CF, Harper CM. Pediatric clinical electromyography. Philadelphia: Lippincott, Wiliams &
Wilkins, 1996.
Question 355: Physiology - EMG
Discussion:
The blink reflex is elicited by stimulation of the supraorbital nerve. Stimulation elicits an R1 response on the
side of stimulation and R2 responses bilaterally. In a unilateral trigeminal lesion, the ipsilateral R1 and both R2
latencies will be prolonged; stimulation on the normal side will show normal latencies of all responses.
References:
95
2007 RITE Discussion & Reference Manual
References:
Preston DC, Shapiro BE. Electromyography and neuromuscular disorders: clinical-electrophysiologic
correlations. 2nd ed. Philadelphia: Elsevier, 2005; 59-64.
Question 360: Physiology - Basic Physiology
Discussion:
Ethosuximide, used to treat absence seizures, binds to calcium channels in the thalamus.
References:
Harden CL. New antiepileptic drugs. Neurology 1994;44:787-795.
Question 362: Physiology - EMG
Discussion:
Nerve impulses conduct faster at a higher body temperature. Lower temperature augments the amplitude of the
nerve and the muscle potential. Both motor and sensory fibers conduct substantially more slowly in the legs than
in the arms. The nerve impulse propagates faster in the proximal than in the distal nerve segments. A reduction
in the mean conduction rate of about 10% is seen at age of 60 and above. Aging also causes a diminution in
amplitude and changes in the shape of the evoked potential. Nerve conduction velocities are higher in large
myelinated fibers.
References:
Kimura, J. Electrodiagnosis in diseases of nerve and muscle: principles and practice. 2nd ed. Philadelphia:
Davis, 1989;94-97.
Question 377: Physiology - EEG
Discussion:
The electrical fields that generate EEG signals are the result of inhibitory and excititory postsynaptic potentials
(IPSPs and EPSPs) on the apical dendrites of cortical neurons. Pyramidal neurons contribute the plurality of the
signal.
References:
Swanson TH. Basic cellular and synaptic mechanisms underlying the electroencephalogram. In: Levin KH,
Lüders HO, editors. Comprehensive clinical neurophysiology. Philadelphia: WB Saunders, 2000;349-385.
Question 379: Physiology - EMG
Discussion:
Myotonic dystrophy is a multisystem disease that can cause cardiac conduction defects, early cataracts, frontal
balding, ptosis, facial weakness, hip girdle and finger flexor weakness as well as hypogammaglobulinemia,
endocrine and CNS abnormalities. Electrophysiologic studies demonstrate myotonic discharges, even when there
is minimal to no clinical myotonia elicitable.
References:
Day JW, Ranum LPW. Myotonic disorders. In: Katirji B, Kaminski HJ, Preston DC, et al, editors.
Neuromuscular disorders in clinical practice. Woburn, Mass: Butterworth-Heinemann, 2002;1078-1091.
Question 388: Physiology - Evoked Potentials
Discussion:
A reduced P100 amplitude of a pattern reversal evoked potential on one side is most likely due to decreased
visual acuity in that eye.
96
2007 RITE Discussion & Reference Manual
References:
Chiappa KH. Evoked potentials in clinical medicine. 3rd ed. New York: Lippincott-Raven, 1997.
Question 389: Physiology - EEG
Discussion:
The EEG record is generated by recording electrical potential differences between pairs of electrodes. The
universal convention is that negative potential differences are represented by a deflection above the baseline, ie,
up.
References:
Hamer HM, Lüders HO. Electrode montages and localization potentials in clinical electroencephalography. In:
Levin KH, Lüders HO, editors. Comprehensive clinical neurophysiology. Philadelphia: WB Saunders,
2000;358-386.
Question 391: Physiology - EMG
Discussion:
In Bell’s palsy, facial nerve conduction studies assess the facial nerve distal to the stylomastoid foramen only.
Blink reflexes allow assessment of the nerve proximal to the stylomastoid foramen, including the intraosseus
portion.
References:
Kimura J. Electrodiagnosis in disease of nerve and muscle. 3rd ed. New York: Oxford University Press, 2001.
Question 404: Physiology - Evoked Potentials
Discussion:
In upper extremity somatosensory evoked potential (SEP), after stimulation of the median or ulnar nerve in the
wrist, activity can be recorded at the elbow, erbs point cervical spine, and scalp. The N5 potential is recorded in
the median or ulnar nerve. The N9 potential is recorded at the Erb’s point. The N11 potential is recorded at the
dorsal root entry zone. The N13 potential is created by dorsal column of the cervical cord and N14 potential
represents activity in the nucleus cuneatus or medial lemniscus at the cervicomedullary junction. The activity
recorded over the frontal lobe is seen by the N30 peak.
References:
Daube JR, editor. Clinical neurophysiology. New York: Oxford University Press, 2002;159-160.
Question 405: Physiology - EMG
Discussion:
Damage to the long thoracic nerve can occur with neck surgical procedures, resulting in scapular winging due to
weakness of the serratus anterior muscle.
References:
Kimura J. Electrodiagnosis in disease of nerve and muscle. 3rd ed. New York: Oxford University Press, 2001.
Question 416: Physiology - Evoked Potentials
Discussion:
Brainstem auditory evoked potentials (BAEPs) have five measurable waves. Presumed anatomical location for
the source of these waves are as follows: wave I - distal action potential of CN VIII; wave II - ipsilateral
proximal CN VIII or cochlear nucleus; wave III - ipsilateral superior olivary nucleus; wave IV - nucleus or axon
97
2007 RITE Discussion & Reference Manual
of lateral lemniscus; wave V - inferior colliculus; wave VI - medial geniculate body; wave VII - thalamocortical
pathways. In brain death, peak I may still be present.
References:
Daube JR, editor. Clinical neurophysiology. Philadelphia: FA Davis, 1996.
Question 420: Physiology - Autonomic Studies
Discussion:
Painful neuropathies and significant reflex sympathetic dystrophy can both cause persistent or “hung up” sweat
responses with the quantitative sweat testing (QSART).
References:
Daube JR, editor. Clinical neurophysiology. Philadelphia: FA Davis, 1996;81-84.
Question 444: Physiology - EEG
Discussion:
In hepatic coma, the EEG often shows a triphasic wave pattern consisting of medium to high-voltage broad
triphasic waves that occur rhythmically and bilaterally synchronous and symmetrical fashion over the two
hemispheres. They have a fronto-occipital or occipitofrontal time lag [2]. The triphasic waves usually have a
frontal predominance[3].
References:
Kass D, Westmoreland B. Electroencephalography: general principles and adult electroncephalograms In:
Daube JR, editor. Clinical neurophysiology. New York: Oxford University Press, 2000;98.
Bickfor RG, Butt HR. Hepatic coma: the electroencephalographic pattern. J Clin Invest 1955;34:790-799.
Fisch BJ, Klass DW. The diagnostic specificity of triphasic wave patterns. Electroencephalogr Clin
Neurophysiol 1988;70:1-8.
98